Тест тосолов за рулем: сюрприз — стало лучше! — журнал За рулем

Содержание

Экспертиза «За рулем»: тест охлаждающих жидкостей на горение: тесты и обзоры

Журнал «За рулем»,16 августа 2016 г.
Многие продаваемые у нас охлаждающие жидкости могут стать причиной пожара. Мы в этом убедились, проверив на горючесть 12 образцов антифризов. Результаты — в тексте редакции.

Недавно по телевидению прошел сюжет про творог, который горит! Я не знаю, что туда подмешали. Но абсолютно уверен, что получить легковоспламеняющуюся охлаждающую жидкость намного проще. Достаточно при производстве вместо дорогого этиленгликоля использовать дешевый глицерин и добавить метанол для приведения вязкости продукта к приемлемому уровню. Причем этим занимаются многие производители. И никому не говорят, что температура кипения метанола — всего-навсего 64,5°C. Зачем, дескать, пугать людей?

Чем метанол опасен для моторов, мы писали неоднократно. Но никогда не заостряли внимание на том, что утечка в системе охлаждения может привести… к пожару. Попади метанол, скажем, на раскаленный коллектор — и привет: вспоминайте, как звонить с мобильника пожарным. Не верите? Мы провели небольшие испытания.

Закупив в подмосковном магазинчике дюжину различных охлаждающих жидкостей, именуемых тосолами и антифризами, мы напросились в гости в химлабораторию — измерить температуру кипения и определить таким образом наличие метанола. В серьезных испытаниях для этого обычно используют хроматограф, который легко покажет на экране состав проверяемой жидкости. Но наша цель была иной. Хроматограф — это нечто заумное, а электрическая плитка и термометр сделают проверку простой и наглядной. Особенно когда на плитке одновременно стоят два стаканчика с жидкостями, одна из которых вовсю кипит, а вторая и не думает. Именно такой эксперимент мы и провели. Тем, кто захочет его повторить, очень советуем раздобыть специальные термостойкие стаканчики, которые не станут трескаться в неподходящий момент.

Мы нагревали каждую приобретенную жидкость, фиксируя температуру начала кипения (появление первых пузырьков) и собственно температуру кипения. Полученные результаты приводим в нашей фотогалерее.

Вспомнив про горящий творог, мы попытались поочередно поджечь все проверяемые жидкости зажигалкой. Результат жутковатый: пять образцов из дюжины мгновенно вспыхнули. Более того, если на плитке одновременно находились два стаканчика — один со стандартным антифризом, а другой с метанольным коктейлем, то при попытке поджечь хорошую жидкость вспыхивал… соседний состав. Пары метанола очень не любят раскаленных поверхностей и открытого огня!

Результаты наших исследований относятся к конкретным образцам приобретенной нами продукции, а потому не могут служить основанием для выводов о деятельности той или иной компании. Вполне возможно, что некоторые производители повторят наш эксперимент — прежде всего те, чья продукция на поверку оказалась небезопасной.

Еще один простейший способ выявить жидкость низкого качества — использовать тест-полоски, определяющие наличие алкоголя в слюне. Во всех случаях индикаторы, искупавшись в метанольном коктейле, поменяли цвет.

А что делать потребителю? Кипятить товар перед покупкой ему точно не разрешат. Один из способов оградить себя от неприятностей — приобретать только те товары, которые хорошо зарекомендовали себя в наших экспертизах. Избегайте торговых развалов на обочинах дорог. Хотя и в магазинах (как следует из нашего эксперимента) не всегда продают качественный товар. Причем продавцы иногда понятия не имеют, какие неприятности сулит нормальная с виду канистра. Поэтому еще один совет: пользуйтесь услугами проверенного автосервиса. Сервисмены судят о качестве приобретенной партии товара, устраивая выборочную проверку. Это куда надежнее единичных закупок.

Sintec Euro G11, Россия

Температура начала кипения 99 ºС
Температура кипения 105 ºС
НЕ ГОРИТ!

Антифриз SINTEC EURO – охлаждающая жидкость последнего поколения, созданная на основе высококачественного моноэтиленгликоля и импортных ингибиторов коррозии. Обладает смазывающими свойствами, продлевающими ресурс водяного насоса. Не повреждает резиновые детали системы охлаждения.

Аntifreeze Maxim, Россия

Температура начала кипения 70 ºС
Температура кипения 84 ºС
ГОРИТ!

Antifreeze G11 Pilot, Россия

Температура начала кипения 70 ºС
Температура кипения 84 ºС
ГОРИТ!

Unix Antifreeze, Россия

Температура начала кипения 79 ºС
Температура кипения 88 ºС
ГОРИТ!

Nord Antifreeze, Россия

Температура начала кипения 106 ºС
Температура кипения 109 ºС
НЕ ГОРИТ!

ТCL Anti-Freeze, Япония

Температура начала кипения 100 ºС
Температура кипения 105 ºС
НЕ ГОРИТ!

Антифриз CoolStream Standard —40 ºС, Россия

Температура начала кипения 105 ºС
Температура кипения 109 ºС
НЕ ГОРИТ!


Тосол —40 Felix, Россия

Температура начала кипения 99 ºС
Температура кипения 105 ºС

НЕ ГОРИТ!

Тосол Айсберг, Россия

Температура начала кипения 103 ºС
Температура кипения 106 ºС
НЕ ГОРИТ!

Тосол Аляsка, Россия

Температура начала кипения 91 ºС
Температура кипения 93 ºС
ГОРИТ!

Тосол Дзержинский Maxim, Россия

Температура начала кипения 70 ºС
Температура кипения 89 ºС
ГОРИТ!

Тосол-север 40, Россия

Температура начала кипения 103 ºС
Температура кипения 106 ºС
НЕ ГОРИТ!

Источник: www. zr.ru

Антифриз тесты за рулем 2016


Тест автомобильных охлаждающих жидкостей ТОСОЛов и антифризов 2016

Наше издание регулярно готовит для своих читателей материалы, посвященные тестам автомобильных охлаждающих жидкостей. Последний опубликованный тест (http://www.autodela.ru/main/top/test/antifreeze_test_20_og) показал, что количество охлаждающих жидкостей, характеристики которых не соответствуют требованиям ГОСТ, из года в год не сокращается, особенно в сегменте бюджетных продуктов. Тенденция последнего времени выявила рост числа охлаждающих жидкостей с содержанием глицерина и метанола. Также были выявленные охлаждающие жидкости, содержащие смесь глицерина и этиленгликоля.

Давайте разберемся, чем опасно использование глицерина.

Глицерин предлагали использовать как компонент охлаждающей жидкости еще в начале ХХ века. Но проблема глицериновых охлаждающих жидкостей – высокая вязкость и недостаточная текучесть. Ее пытались решить за счет добавления этанола, метанола, солей, и т. д., пока не нашли полноценную замену: основой охладителя стал этиленгликоль. К 1937 голу антифризы на основе этиленгликоля практически вытеснили глицериновые и метаноловые.

При изготовлении охлаждающей жидкости с температурой замерзания –40 °С соотношение глицерина и воды должно составлять 65:35 % масс., в то время как для этиленгликоля и воды – 53:47 % масс., т е. при одинаковой температуре замерзания глицериновый антифриз содержит больше органического компонента (глицерина) и меньше воды, чем этиленгликолевый. Это приводит к дополнительному повышению плотности и вязкости, к снижению теплоемкости.

Но есть еще одна, более существенная проблема. Глицерин термически нестабилен, при длительном нагреве свыше 90 °С разлагается с образованием летучих веществ, в т.ч. акролеина. Продукты разложения токсичны, коррозионно агрессивны. Глицерин сильно пенится, и по этой причине отвод тепла ухудшается.

Из-за таких свойств, как высокая вязкость при минусовых температурах, при использовании глицерина в системе охлаждения двигателя все ее компоненты будут работать не в проектном режиме: многократно возрастет нагрузка на крыльчатку помпы, значительно увеличится механический износ от трения, что существенно сократит срок службы радиатора и помпы.

Но и это не все: при высоких температурах, которые возникают на поверхности цилиндров, глицерин разлагается, выпадая в осадок, забивая каналы охлаждения и значительно увеличивая коррозионную активность такой смеси. Следствием присутствия глицерина (и особенно метанола) является снижение температуры кипения до опасного уровня, не говоря о том, что скачкообразно растет кавитационный износ.

Подробнее 

ЗАЯВЛЕННЫЕ ОСОБЕННОСТИ

Произведено ООО ПКФ «ПРОМПЭК»

«Дзержинский ТОСОЛ А40М» предназначен для охлаждения двигателя внутреннего сгорания автотракторной техники отечественного и импортного производства при температуре окружающего воздуха до –40 °С. Совместима с охлаждающими жидкостями отечественного и импортного производства на основе этиленгликоля.

РЕЗУЛЬТАТЫ ТЕСТА

Лабораторные исследования показали, что «Дзержинский ТОСОЛ А40М» имеет очень низкую температуру замерзания –55 °С. Казалось бы, можно радоваться: как-никак большой запас относительно температуры кристаллизации.

Но при этом образец проваливает испытания по температуре кипения, которая вместо положенных ГОСТ 108 °С оказалась равной 96,5 °С.

Причину этого выявили в лаборатории. Охлаждающая жидкость «Дзержинский ТОСОЛ А40М» выполнена не на основе этиленгликоля, а на основе глицерина с добавлением метилового спирта, доля которого составила 5,9 %.

РЕЗЮМЕ

Использование в качестве основы для охлаждающей жидкости не этиленгликоля, а глицерина запрещено ГОСТ. Поэтому мы не рекомендуем использовать «Дзержинский ТОСОЛ А40М» в ДВС.

ЗАЯВЛЕННЫЕ ОСОБЕННОСТИ

Произведено ООО ПКФ «Завод дзержинских антифризов»

ZDA Antifreeze G11 предназначен для охлаждения двигателя внутреннего сгорания, а также для использования в качестве теплоносителя в теплообменных аппаратах и системах терморегулирования.

Состав: этиленгликоль, вода, присадки антикоррозионные, антипенные агенты, смазывающие добавки. Не содержит аминов, фосфатов, нитритов. Соответствует стандарту VW TL 774C (G11).

РЕЗУЛЬТАТЫ ТЕСТА

Лабораторные исследования показали, что ZDA Antifreeze G11 не уложился в порог температуры кристаллизации и вместо –40 °С замерз при температуре –37 °С. Зато он, в отличие от предыдущего состава, изготовлен из этиленгликоля, что положительным образом сказалось на температуре кипения, которая составила 108 °С.

РЕЗЮМЕ

Охлаждающую жидкость ZDA Antifreeze G11 мы не рекомендуем к использованию, так как она не уложилась в требования ГОСТ по температуре кристаллизации.

ЗАЯВЛЕННЫЕ ОСОБЕННОСТИ

Произведено ООО ПКФ «НИАГАРА»

Антифриз нового поколения NIAGARA Antifreeze Universal G11&G12 предназначен для использования в системах охлаждения современных высокофорсированных двигателей. Рекомендован для эксплуатации в тяжело нагруженных автомобилях. Содержит комбинированный пакет антикоррозионных и смазывающих присадок, обеспечивающий эффективную и долговечную защиту системы охлаждения двигателя от замерзания, перегрева и коррозии.

Состав: этиленгликоль, вода, функциональные присадки агенты, краситель.

РЕЗУЛЬТАТЫ ТЕСТА

Лабораторные исследования показали, что NIAGARA Antifreeze Universal G11&G12 превосходит требования ГОСТ. Температура кристаллизации состава –43,5 °С. Температура кипения также выше требований ТУ 111,5 °С. Лаборатория констатировала, что состав изготовлен на основе этиленгликоля.

РЕЗЮМЕ

Охлаждающая жидкость NIAGARA Antifreeze Universal G11&G12 превосходит требования ГОСТ и поэтому может быть рекомендован к применению.

ЗАЯВЛЕННЫЕ ОСОБЕННОСТИ

Произведено ООО ПКФ «НИАГАРА»

Карбоксилатный антифриз нового поколения NIAGARA RED G12+ предназначен для использования в системах охлаждения современных двигателей. Рекомендован для эксплуатации в тяжело нагруженных автомобилях. Содержит комбинированный пакет антикоррозионных и смазывающих присадок, обеспечивающий эффективную и долговечную защиту системы охлаждения двигателя от замерзания, перегрева и коррозии. На упаковке производитель обещает бесперебойную работу состава на протяжении 250 тыс. км пробега.

Состав: этиленгликоль, вода, функциональные присадки агенты, краситель.

РЕЗУЛЬТАТЫ ТЕСТА

Лабораторные исследования показали, что NIAGARA RED G12+ укладывается в требования ГОСТ по температуре кристаллизации (–40,5 °С). А вот температура кипения оказалась ниже требуемых 108 °С, правда, совсем на чуть-чуть, и составила 107,5 °С. В лаборатории констатировали, что состав изготовлен из этиленгликоля.

РЕЗЮМЕ

Охлаждающая жидкость NIAGARA RED G12+ буквально на полградуса не уложилась в установленную температуру закипания охлаждающей жидкости.

ЗАЯВЛЕННЫЕ ОСОБЕННОСТИ

Произведено OOO «СТРЭКСТЭН» Delfin group

Pilots Antifreeze G12 имеет веселенькую упаковку, по дизайну больше напоминающую японское средство для стирки. Однако перед нами охлаждающая жидкость для применения в двигателях внутреннего сгорания. Заявляется, что состав обеспечивает надежное охлаждения двигателя в температурном диапазоне –40 до +50 °С.

Состав: вода, полиолы, алифатические спирты, антикоррозионные и противопенные присадки, краситель.

РЕЗУЛЬТАТЫ ТЕСТА

Лабораторные исследования показали, что Pilots Antifreeze G12 не соответствует требованиям ГОСТ. Его температура кристаллизации всего –36 °С, что выше требований ТУ, при низкой температуре кипения (всего 94 °С). Подобное поведение характерно для составов с использованием глицерина. Если внимательно читать этикетку, то можно увидеть, что в составе не указан этиленгликоль, при этом указано наличие полиолов. Заглядываем в Сеть и находим, что это многоатомные спирты, к которым относится и глицерин. Что и выявил лабораторный анализ. Кстати, анализ также показал наличие метилового спирта в количестве 9,31 %.

РЕЗЮМЕ

Использование в качестве основы для охлаждающей жидкости не этиленгликоля, а глицерина и метилового спирта запрещено ГОСТ. Поэтому мы не рекомендуем использовать Pilots Antifreeze G12 в ДВС.

ЗАЯВЛЕННЫЕ ОСОБЕННОСТИ

Произведено OOO «Ви Кемикал»

Высококачественный, готовый к применению антифриз Vitex G 12 ultra предназначен для использования в системе охлаждения двигателей последнего поколения с алюминиевым блоком цилиндров. Усовершенствованная карбоксилатная основа антифриза Vitex G 12 Ultra G безопасна для металлических частей всех современных двигателей. Благодаря усовершенствованному составу карбоксилатной основы и уникальному антикоррозионному пакету присадок продукт увеличивает резерв работы двигателя, что позволяет не думать о замене антифриза до 200 000 км.

Мягкая формула антифриза не содержит в своем составе аминов, нитритов, фосфатов, силикатов и боратов, вызывающих образование геля и осадка. Поэтому использование антифриза Vitex G12 Ultra G предотвращает загрязнение системы охлаждения двигателя. Уникальный пакет присадок Ultra SAFE придает охлаждающей жидкости антикоррозионные и антивспенивающие свойства и обеспечивает ее безопасное применение.

Действуя на молекулярном уровне, пакет присадок проявляет защитные свойства только в тех местах двигателя, которые подверглись коррозионному воздействию. В точках повреждений образуется защитный слой, препятствующий образованию и развитию коррозии и обеспечивающий безопасную работу двигателя.

Произведено из концентрата Alpine Mitan GmbH (Германия), имеющего допуски Mercedes-Benz Blatt 325.3, MAN 324 SNF, VW TL 774 D/F.

Состав: вода, этиленгликоль, пакет присадок Ultra SAFE, антипенные агенты, смазывающие добавки.

РЕЗУЛЬТАТЫ ТЕСТА

Лабораторные исследования показали, что Vitex G 12 ultra превосходит требования ГОСТ. Температура кристаллизации состава оказалось –42,5 °С. Температура кипения также выше требований ТУ: 109,5 °С. Лаборатория констатировала, что состав изготовлен на основе этиленгликоля.

РЕЗЮМЕ

Охлаждающая жидкость Vitex G 12 ultra превосходит требования ГОСТ и поэтому рекомендуем к применению.

ЗАЯВЛЕННЫЕ ОСОБЕННОСТИ

Произведено OOO «Тосол-Синтез-Инвест»

Professional Antifreeze FELIX PROLONGERпредназначен для использования во всех легковых и грузовых автомобилях, в том числе высоконагруженных и форсированных, эксплуатируемых в тяжелых климатических и дорожных условиях.

По заявлению производителя, состав обеспечивает 100% защиту двигателя и деталей системы охлаждения от всех видов коррозии, перегрева и переохлаждения. Увеличенный пробег без замены охлаждающей жидкости – до 120 000 км. Обладает усиленной и пролонгированной защитой металлов (в том числе алюминия), сплавов, резины. Его применение исключает возможность образования накипи и отложений, при этом состав обладает высокими смазывающими свойствами и низким пенообразованием.

Эффективен при температурах окружающего воздуха от –45 до +50 °С. Обеспечивает быстрый прогрев двигателя до нужной температуры. Обладает превосходной теплопроводностью и теплоемкостью. Улучшает работу и продлевает срок службы помпы, термостата, радиатора.

Содержит запатентованный многофункциональный пакет антикоррозионных, антикавитационных, антипенных и смазывающих присадок. Изготовлен из моноэтиленгликоля высшего сорта и специально подготовленной деминерализованной воды высшей степени очистки. Неорганический антифриз класса G11 по классификации VW.

FELIX PROLONGERотвечает требованиям крупнейших автопроизводителей и международным стандартам качества: Audi: TL 774-C, BMW: BMW N 600 69. 0, German Army: TL 6850-0038/1, KHD: H-LV 0161 0188, MAN: MAN 324-NE, Mercedes-Benz: DBL 7700.20 page 325.0, MTU: MTL 5048, Opel/General Motors: B 040 0240, Saab: 6901599, Seat: TL 774-C, Scoda: TL 774-C, Volkswagen: TL 774-C (VW code G11), Volvo 1286083, Deutz AG: 0199-99-1115 and 0199-99-2091, GE Jenbacher: TA 1000-0201, Chrysler MS 7170, John Deere h44C1, ASTM D 3306, ASTM D 4340, SAE J 1034, BS 6580-1992, AFNOR R15-601.

РЕЗУЛЬТАТЫ ТЕСТА

Лабораторные исследования показали, что FELIX PROLONGER соответствует требованиям ГОСТ. Температура кристаллизации состава равна –40,0 °С. Температура кипения выше требований ТУ: 110 °С. Лаборатория констатировала, что состав изготовлен на основе этиленгликоля.

РЕЗЮМЕ

Охлаждающая жидкость FELIX PROLONGER превосходит требования ГОСТ и поэтому рекомендуется к применению.

ЗАЯВЛЕННЫЕ ОСОБЕННОСТИ

Произведено OАO «ТЕХНОФОРМ»

CoolStream Standard– универсальная охлаждающая жидкость зеленого цвета наоснове этиленгликоля, выпускаемая покарбоксилатнойOAT-технологии(Organic Acid Technology). Изготавливается изсуперконцентрата Corrosion Inhibitor BSB компании Arteco (Belgium) иявляется точной копией (ребрендом)AntifreezeBS-Coolant.

Обладает следующими преимуществами: нет блокирующего слоя (нет силикатов), улучшенная теплопередача, высокотемпературная стабильность ингибиторов, нет зависимости свойств отжесткости воды, для концентрата (нет фосфатов), необразуются осадки игели (нет силикатов), обеспечивает до 150 000 км пробега без замены.

Спецификации:BS 6580, ASTM D3306 BS 6580:1992, BS 6580:2010 [1] GB 29743-2013 NB/SH/T 0521-2010.

Допуски и первоначальная заправка:КАМАЗ, ГАЗ, МАЗ, АвтоВАЗ, ЛиАЗ, НЕфАЗ, ПАЗ, ММЗ, ЯМЗ, УАЗ.

Антифриз CoolStream Standard отвечает требованиям производителейтяжелонагруженнойтехники ииспользуется как антифриз постгарантийного обслуживания грузовиков.

Состав: вода, этиленгликоль, пакет присадок.

РЕЗУЛЬТАТЫ ТЕСТА

Лабораторные исследования показали, что CoolStream Standard не соответствует требованиям ГОСТ по температуре кристаллизации состава, которая оказалась выше, чем требуемые –40 °С, и оказалось составила – 37 °С. При этом температура кипения охлаждающей жидкости выше требований ТУ: 110 °С. Лаборатория констатировала, что состав изготовлен на основе этиленгликоля.

РЕЗЮМЕ

Охлаждающая жидкость CoolStream Standard не уложилась в требования ГОСТ по требуемой температуре кристаллизации.

ЗАЯВЛЕННЫЕ ОСОБЕННОСТИ

Произведено OОO «Гелена химавто»

Антифриз NORD – универсальная охлаждающая жидкость, предназначенная для заправки систем охлаждения автомобильных двигателей всех марок (отечественных и зарубежных), эксплуатируемых при температуре окружающего воздуха от –40 С до +50 °С. Нормативный срок службы: 4 года или 120 тыс. км пробега.

По заверениям производителя, состав обеспечивает отличное охлаждение двигателя, увеличенный срок службы водяного насоса, мощную антикоррозионную защиту радиатора. При применении антифриза NORD отсутствует кавитационная коррозия в каннах головки блока и блока цилиндров, состав облегчает поиск мест утечки за счет флуоресцирующего красителя.

Состав: вода, этиленгликоль, антикоррозионные, антипенные, моюще-диспергирующие, антифрикционные и стабилизирующие присадки.

РЕЗУЛЬТАТЫ ТЕСТА

Лабораторные исследования показали, что антифриз NORD не соответствует требованиям ГОСТ по температуре кристаллизации состава, которая оказалась выше, чем требуемые –40 °С, и составила –37 °С. При этом температура кипения охлаждающей жидкости также оказалось ниже требований ТУ: 107,5 °С. Лаборатория констатировала, что состав изготовлен из смеси моно- и диэтиленгликоля.

РЕЗЮМЕ

Охлаждающая жидкость NORD не уложилась в требования ГОСТ и поэтому не рекомендуется к применению в ДВС.

ЗАЯВЛЕННЫЕ ОСОБЕННОСТИ

Произведено OOO «Обнинскоргсинтез»

SINTEC EURO G11 – охлаждающая жидкость последнего поколения, созданная на основе высококачественного моноэтиленгликоля и импортных ингибиторов коррозии.

Применяется в системе охлаждения легковых автомобилей отечественного и зарубежного производства, грузовиков и других транспортных средств со средними и тяжелыми условиями эксплуатации. Обеспечивает правильный тепловой режим эксплуатации двигателя, предотвращает появление коррозии и отложений в системе охлаждения. Не допускает замерзания, перегрева, выкипания и появления воздушных пробок.

Обладает эффективными смазывающими свойствами, продлевающими ресурс водяного насоса. Не повреждает шланги, прокладки и уплотнения системы охлаждения.

Имеет допуски: DERWAYS, Uz DAEWOO, ФУЗО КАМАЗ Тракс Рус.

РЕЗУЛЬТАТЫ ТЕСТА

Лабораторные исследования показали, что SINTEC EURO G11 соответствует требованиям ГОСТ. Температура кристаллизации состава оказалось –40,5 °С. Температура кипения выше требований ТУ: 110,5 °С. Лаборатория констатировала, что состав изготовлен на основе этиленгликоля.

РЕЗЮМЕ

Охлаждающая жидкость SINTEC EURO G11 соответствует требованиям ГОСТ и поэтому рекомендуется к применению.

ЗАЯВЛЕННЫЕ ОСОБЕННОСТИ

Произведено OОO «Промсинтез»

Охлаждающая жидкость «Oil Right Тосол Дзержинский ОЖ-40» предназначена к применению в системах охлаждения двигателей внутреннего сгорания автомобилей.

Отличительные особенности состава, заявленные производителем: обладает высокими антикоррозионными свойствами; эффективно предохраняет систему охлаждения от замерзания зимой и от закипания летом; снижает вероятность образования отложений и выпадения осадка в системе охлаждения; способствует увеличению срока службы термостата, радиатора и водяного насоса.

Соответствует международным стандартам ASTM D 3306, SAE J 1034 и спецификациям автомобильных компаний Audi, BMW, Opel, Volvo, Volkswagen.

Состав: вода, этиленгликоль, попионы, алифатические спирты, антикоррозионные, противопенные присадки, краситель.

РЕЗУЛЬТАТЫ ТЕСТА

Лабораторные исследования показали, что «Oil Right Тосол Дзержинский ОЖ-40» не соответствует требованиям ГОСТ по температуре кристаллизации состава, которая выше, чем требуемые –40 °С, и составила –37 °С. Температура кипения охлаждающей жидкости также оказалось ниже требований ТУ: 92,5 °С.

Подобное поведение характерно для составов с использованием глицерина. Если внимательно читать этикетку, то в составе указано наличие полиолов, которые представляют собой многоатомные спирты, к коим относится и глицерин. Его наличие в составе показал и лабораторный анализ. При этом также используется смесь моно- и триэтиленгликоля.

РЕЗЮМЕ

Охлаждающая жидкость «Oil Right Тосол Дзержинский ОЖ-40» не уложилась в требования ГОСТ и поэтому не рекомендуется к применению в ДВС.

ЗАЯВЛЕННЫЕ ОСОБЕННОСТИ

Произведено OОO ПКФ «Промпэк»

Охлаждающая жидкость Antifreeze Extra -42 G11 предназначена для использования в системах охлаждения любых современных бензиновых и дизельных двигателей автомобилей всех марок иностранного и российского производства при температуре окружающего воздуха до –42 °С.

По заявлению производителя, состав идеален для высокофорсированных двигателей и двигателей, эксплуатирующихся в тяжелых условиях. Содержит оптимально сбалансированный пакет антиокислительных, антипенных и антифрикционных присадок.

Имеет повышенную активационную защиту припоя, алюминия, меди, латуни, стали и чугуна.

Антифриз Antifreeze Extra -42 G11 разработан с учетом требований ASTM D3306/4656 и VW TL 774C (G11).

Состав: вода, моноэтиленгликоль, пакет присадок, краситель.

РЕЗУЛЬТАТЫ ТЕСТА

Лабораторные исследования показали, что Antifreeze Extra -42 G11 не соответствует требованиям ГОСТ по температуре кристаллизации состава, которая ваше, чем требуемые –40 °С, и оказалось равной –39 °С. При этом температура кипения охлаждающей жидкости выше требований ТУ: 111 °С. Лаборатория констатировала, что состав изготовлен на основе этиленгликоля.

РЕЗЮМЕ

Охлаждающая жидкость Antifreeze Extra -42 G11 не уложилась в требования ГОСТ по температуре кристаллизации.

ЗАЯВЛЕННЫЕ ОСОБЕННОСТИ

Произведено OOO «Ви Кемикал»

Тосол Vitex А40D предназначен для охлаждения двигателей внутреннего сгорания, а также для использования в качестве теплоносителя в теплообменных аппаратах и системах терморегулирования. Совершенная рецептура изготовления тосола на основе ультранового комплекса присадок, позволяет двигателю автомобиля работать без перегрузок и защищает его от коррозии.

Тосол Vitex А-40D изготовлен из высококачественного сырья (моноэтиленгликоля) с применением специализированного пакета присадок. Антикоррозионные, антивспенивающие и защитные свойства присадок обеспечивают надежную защиту конструкционных материалов двигателей российского и иностранного производства.

Тосол Vitex А-40D полностью соответствует требованиям ГОСТ 28084-89 и ASTM. Благодаря высоким качественным показателям является экономичной альтернативой антифризу класса G11.

Обладает отличными защитными свойствами. Не содержит солей, образующих нежелательные осадки. Безопасен для пластика, резиновых изделий, лакокрасочных покрытий. Не содержит фосфатов, аминов, нитритов. Совместим с другими охлаждающими жидкостями на основе этиленгликоля.

РЕЗУЛЬТАТЫ ТЕСТА

Лабораторные исследования показали, что тосол Vitex А-40D соответствует требованиям ГОСТ. Температура кристаллизации состава оказалось –40 °С. Температура кипения выше требований ТУ: 110,5 °С. Лаборатория констатировала, что состав изготовлен на основе этиленгликоля.

РЕЗЮМЕ

Тосол Vitex А-40D соответствует требованиям ГОСТ и поэтому рекомендуется к применению в ДВС.

ЗАЯВЛЕННЫЕ ОСОБЕННОСТИ

Произведено OОO «Дзержинские антифризы»

Охлаждающая жидкость «Тосол Дзержинский А40М» предназначена для охлаждения двигателей внутреннего сгорания.

Состав изготовлен на основе этиленгликоля. Содержит антипенные присадки. Применяется при температуре окружающей среды не ниже –40 °С.

РЕЗУЛЬТАТЫ ТЕСТА

Лабораторные исследования показали, что «Тосол Дзержинский А40М» не соответствует требованиям ГОСТ по температуре кристаллизации состава, которая выше, чем требуемые –40 °С, и оказалось равной –30,5 °С. Температура кипения охлаждающей жидкости также оказалось ниже требований ТУ: 101 °С.

Подобное поведение характерно для составов с использованием глицерина, что и выявил лабораторный анализ. При этом в продукте используется и этиленгликоль. Также было выявлено содержание в охлаждающей жидкости метилового спирта в концентрации 4,45 %.

РЕЗЮМЕ

Охлаждающая жидкость «Тосол Дзержинский А40М» не уложилась в требования ГОСТ и поэтому не рекомендуется к применению в ДВС.

ЗАЯВЛЕННЫЕ ОСОБЕННОСТИ

Произведено OОO «Нова-нефтехим»

Охлаждающая жидкость «MR.AUTO Дзержинский antifrize Тосол-40» предназначена для охлаждения двигателей внутреннего сгорания, а также в качестве рабочей жидкости в других теплообменных аппаратах, эксплуатируемых при низких температурах.

Производитель заявляет об использовании технологии органического синтеза, наличии в охлаждающей жидкости оптимизированного пакета присадок, предназначенного для защиты системы охлаждения. Заявляется о гарантии качества продукта. Указано, что производство соответствует стандарту EURO ISO 9001.

РЕЗУЛЬТАТЫ ТЕСТА

Лабораторные исследования показали, что «MR.AUTO Дзержинский antifrize Тосол-40» не соответствует требованиям ГОСТ по температуре кристаллизации состава, которая выше, чем требуемые –40 °С, и составила –29 °С. Температура кипения охлаждающей жидкости также оказалось ниже требований ТУ: 94,5 °С.

Подобное поведение характерно для составов с использованием глицерина, что и показал лабораторный анализ. При этом также используется смесь моно- и диэтиленгликоля.

Также было выявлено содержание в охлаждающей жидкости метилового спирта в концентрации 7,79 %.

Возникает резонный вопрос: о каком использовании стандарта EURO ISO 9001 на производстве может идти речь при таком низком качестве выпускаемого продукта?

РЕЗЮМЕ

Охлаждающая жидкость «MR.AUTO Дзержинский antifrize Тосол-40» не уложилась в требования ГОСТ и поэтому не рекомендуется к применению в ДВС.

ЗАЯВЛЕННЫЕ ОСОБЕННОСТИ

Произведено OOO «СТРЭКСТЭН» Delfin group

Pilots Тосол ОЖ-40 – охлаждающая жидкость для применения в двигателях внутреннего сгорания. Заявляется, что состав обеспечивает надежное охлаждение двигателя в температурном диапазоне –40 до +50 °С.

Состав: вода, полиолы, алифатические спирты, антикоррозионные и противопенные присадки, краситель.

РЕЗУЛЬТАТЫ ТЕСТА

Лабораторные исследования показали, что Pilots Тосол ОЖ-40 не соответствует требованиям ГОСТ. Его температура кристаллизации всего –35,5 °С, что выше требований ТУ, при низкой температуре кипения всего 92 °С. Подобное поведение характерно для составов с использованием глицерина. Если внимательно читать этикетку, то в составе не указан этиленгликоль, при этом говорится о наличии полиолов. Это многоатомные спирты, к которым относится и глицерин (что и показал лабораторный анализ). Также установлено наличие метилового спирта в количестве 11,23 %.

РЕЗЮМЕ

Использование в качестве основы для охлаждающей жидкости не этиленгликоля, а глицерина (как и метилового спирта) запрещено ГОСТ. Поэтому мы не рекомендуем использовать Pilots Тосол ОЖ-40 в ДВС.

ЗАЯВЛЕННЫЕ ОСОБЕННОСТИ

Произведено OOO «Тосол НН»

«Тосол-А40М «БАРС» – охлаждающая жидкость для применения в системах охлаждения всех типов отечественных и импортных автомобилей. Обладает антикоррозионными свойствами, совместима с резиновыми и пластиковыми деталями в системе охлаждения. Состав не агрессивен к стали и цветным металлам.

Состав: вода, этиленгликоль, антикоррозионные ингибиторы и противопенные присадки.

На сайте производителя заявлено, что тосол «Барс» – это высококачественныйтосол-А40М классического сине-голубого цвета. Благодаря выбранному наилучшему соотношению цена–качество «Тосол-А40М«Барс»быстро заняла лидирующие позиции на рынке автохимии. Заявленный срок эксплуатации – 150 тыс. км пробега.

РЕЗУЛЬТАТЫ ТЕСТА

Лабораторные исследования показали, что «Тосол-А40М «БАРС» не соответствует требованиям ГОСТ. Его температура кристаллизации всего –26,5 °С. По этому параметру данный продукт – антилидер нашего теста: более высокой температуры кристаллизации не показал ни один образец. Температура кипения состава также ниже технических требований: 106,5 °С.

Лаборатория констатировала, что состав изготовлен на основе этиленгликоля.

РЕЗЮМЕ

Охлаждающая жидкость «Тосол-А40М «БАРС» не уложилась в требования ГОСТ и поэтому не рекомендуется к применению в ДВС.

ЗАЯВЛЕННЫЕ ОСОБЕННОСТИ

Произведено АO «Делфин Индастри» Delfin group

Антифризы LUXE обладают высокими эксплуатационными свойствами, обеспечивающими надежную защиту системы охлаждения откоррозии иобразования накипи. Поряду параметров превосходят все отечественные аналоги. Совместимы совсеми типами охлаждающих жидкостей наоснове этиленгликоля. Обеспечивают надежную работу системы охлаждения двигателя при температуре окружающей среды от–40до+50 °С.

Низкозамерзающая охлаждающая жидкостьпредназначенадля использования всистемах охлаждения двигателей внутреннего сгорания. Всостав зеленого антифриза LUXE входит флуоресцентный краситель, использование которого позволяет быстро определить места протечки. Для этого достаточно просветить предполагаемое место ультрафиолетовой лампой.

По заявлению производителя, состав отвечает требованиямVWTL774-C; Opel GMQL130100; MB-Approval 325. 0/325.2; BMW GS9400; MAN 324NF.

РЕЗУЛЬТАТЫ ТЕСТА

Лабораторные исследования показали, что LUXE Green Line ANTIFREEZE Long Life не соответствует требованиям ГОСТ по температуре кристаллизации, которая при разбавлении 1:1 составила –35,5 °С, что выше требований ТУ и выше значений, указанных на этикетке (– 38 °С). Температура кипения оказалась в норме: 110 °С.

Лаборатория констатировала, что состав изготовлен на основе этиленгликоля.

РЕЗЮМЕ

Охлаждающая жидкость LUXE Green Line ANTIFREEZE Long Life не уложилась в требования ГОСТ по требуемой температуре кристаллизации.

ЗАЯВЛЕННЫЕ ОСОБЕННОСТИ

Произведено ООO «Промсинтез»

«Тосол ОЖ-40» – низкозамерзающая охлаждающая жидкость, которая гарантирует надежную работу системы охлаждения двигателя при температурах окружающей среды от–40до+50 °С. Предназначена для использования в системах охлаждения двигателей внутреннего сгорания, а также в качестве рабочей жидкости в других теплообменных аппаратах. Обеспечивает увеличение срока службы охлаждающей жидкости, стойкую антикоррозионную защиту металлов и сплавов, инертность к резиновым и керамическим деталям. Предотвращает образование накипи.

Состав: вода, этиленгликоль, полиолы, алифатические спирты, антикоррозионные и противопенные присадки, краситель.

РЕЗУЛЬТАТЫ ТЕСТА

Лабораторные исследования показали, что «Тосол ОЖ-40» не соответствует требованиям ГОСТ. Его температура кристаллизации всего –27,5 °С, что выше требований ТУ, при низкой температуре кипения – всего 93,5 °С. Подобное поведение характерно для составов с использованием глицерина. Если внимательно читать этикетку, то в составе указано наличие полиолов. Это многоатомные спирты, к которым относится и глицерин (его наличие выявил лабораторный анализ). Лабораторное исследование также показало наличие метилового спирта в количестве 9,59 %.

РЕЗЮМЕ

Использование в качестве основы для охлаждающей жидкости не этиленгликоля, а глицерина запрещено ГОСТ, равно как и метилового спирта. Поэтому мы не рекомендуем использовать «Тосол ОЖ-40» в ДВС.

ЗАЯВЛЕННЫЕ ОСОБЕННОСТИ

Произведено ООO «Тектрон» Delfin group

«Аляsка Тосол 40» – низкозамерзающая охлаждающая жидкость, предназначенная для работы в системе охлаждения двигателя при температурах окружающей среды от–40до+50 °С.

Новейшая формула обеспечивает увеличение срока службы тосола, стойкую антикоррозионную защиту металлов и сплавов, инертность к резиновым и керамическим деталям. Предотвращает образование накипи.

Состав: вода, этиленгликоль, полиолы, алифатические спирты, антикоррозионные и противопенные присадки, краситель.

РЕЗУЛЬТАТЫ ТЕСТА

Лабораторные исследования показали, что «Аляsка Тосол 40» не соответствует требованиям ГОСТ. Температура кристаллизации –35,5 °С, что выше требований ТУ, при низкой температуре кипения – всего 95 °С. Подобное поведение характерно для составов с использованием глицерина. В составе указано наличие полиолов – многоатомных спиртов, к которым относится и глицерин. По результатам лабораторного анализа состав содержит глицерин, а также метиловый спирт в количестве 9,94 %.

РЕЗЮМЕ

Использование в качестве основы для охлаждающей жидкости не этиленгликоля, а глицерина запрещено ГОСТ, как и метилового спирта. Поэтому мы не рекомендуем использовать «Аляsка Тосол 40» в ДВС.

Тест 17 антифризов: замерзают, закипают и огнем полыхают

У жидкостей, которые используются в системах охлаждения двигателей, два функциональных назначения. Во-первых, в сильные морозы они не должны превращаться в лед, что и объясняет, почему охлаждающие жидкости называют также антифризами, ведь antifreeze в переводе с английского означает «против замерзания». Во-вторых, охлаждающая жидкость не должна преждевременно закипеть и стать по этой причине виновницей перегрева мотора.

В том, что риск перегрева, к которому может быть причастен антифриз, залитый в систему охлаждения, существует, мы убедились нынешним летом в ходе проверки температуры кипения 8 образцов разных марок, закупленных в нескольких торговых точках Минска. Более того, тогда же выяснилось, что пары некоторых кипящих антифризов способны воспламеняться от источника огня и продолжают гореть, о чем рассказали в статье «Шокирующий тест ABW.BY: антифризы горят ярким пламенем!».

Однако когда в свои права вступает зима, самое время проверить готовность антифризов не замерзать до того, как столбик термометра сравняется с температурой, заявленной производителями на ярлыках товарных емкостей с охлаждающими жидкостями. На этот раз мы решили не ограничиваться 8 антифризами, а брали на экспертизу все, что попадалось на глаза в торговых точках по продаже автомобильных запчастей и расходных материалов.

Исключение — концентраты, а также готовые жидкости, за качество которых голосовала авторитетная торговая марка и высокая стоимость. Цена, по словам продавцов, была причиной низкого спроса на подобные продукты, а нас интересовали антифризы, доступные широким массам потребителей и, стало быть, на прилавках не залеживающиеся. В результате набрали 17 образцов.

Но в товарных емкостях на экспертизу они не поехали. Мы обезличили участников теста, для чего перелили их в одинаковые бутылки и присвоили каждому соответствующий номер.

В таком виде антифризы были доставлены в лабораторию, сотрудников которой мы попросили ответить на три вопроса:

  • при какой температуре замерзнет каждый образец?
  • какова температура его кипения?
  • способны ли испарения кипящей жидкости воспламеняться и продолжать гореть?

Если следовать букве ГОСТ 28084-89 и технического регламента ТС 030/2012, то в соответствии с этими документами антифризы должны оцениваться не по полному замерзанию, а по температуре начала кристаллизации.

При этом по Приложению I ТР ТС 030/2012 для охлаждающих низкозамерзающих жидкостей температура начала кристаллизации должна быть не выше -35°С.

Наши образцы проверялись с помощью прибора АТКт-02, работающего по гостовской методике.

Прибор аттестован БелГИМ. Погрешность измерения составляет ±1°С. Это означает, что если проба антифриза начнет кристаллизоваться, например, при -35°С, то фактическая температура жидкости в это время имеет значение в интервале от -34°С до -36°С.

Согласно методике в пробирку для проверки заливается 20 мл жидкости.

Далее пробирка с образцом помещается в криостат прибора.

Сверху гнездо криостата закрывается крышкой с термометром и мешалкой, перемешивающей содержимое пробирки.

Работает аппарат АТКт-02 в автоматическом режиме.

На дисплей прибора выводятся температуры пробы, камеры криостата и хладагента в текущий момент времени. При необходимости можно получить распечатку параметров.

Об окончании испытаний прибор сигнализирует звуковым сигналом.

Работа прибора основана на фиксации изменения пропускания света через жидкость в связи с появлением кристалликов льда. Что визуально представляет собой начало кристаллизации, можно увидеть по тому, во что превратилось содержимое пробирки.

На температуру начала кристаллизации все образцы подвергались испытанию по три раза, что позволило для каждого антифриза усреднить этот показатель и сделать его более достоверным. Что касается температуры замерзания, как ее понимают рядовые потребители, то, по мнению работников лаборатории, она ниже температуры начала кристаллизации на 2 градуса, а если учесть погрешность измерения, то можно уверенно говорить, что антифриз замерзнет полностью при температуре, на 3 градуса более низкой, чем температура начала кристаллизации.

В отличие от летних испытаний на воспламеняемость, когда для наглядности температура кипения оценивалась лишь приблизительно с помощью термометра, на этот раз ее определяли опять-таки по гостовской методике оценки температуры начала перегонки.

Проверке на температуру кипения каждый образец подвергался дважды. Но что готовые антифризы могут гореть, разработчики ГОСТ 28084-89 понятия не имели. Поэтому методики проверки на воспламеняемость нет — в лаборатории просто вносили в испарения закипевшей на электроплитке охлаждающей жидкости зажженную лучину и смотрели, возгорится ли пламя. Как это делается, подробнее можно прочитать в указанной выше статье.

Что получилось в итоге?

Образец №1 — антифриз Starex производства ООО «Нижнекамскнефтеоргсинтез» (Нижнекамск, Россия). Что означает надпись «-40» на этикетке, осталось загадкой, ведь отсутствие единицы измерения превращает ее в абстрактную величину, рядом с которой с одинаковым успехом могли бы стоять как °С, так и кг, л, м и даже амперы с вольтами. Температура начала кристаллизации оказалась ниже, чем требует регламент ТС. Что также плохо — пары Starex горят, хотя на этикетке есть надпись «Пожаровзрывобезопасно». Судя по всему, она, как и «-40» ни к чему не обязывает.

Образец №2 — антифриз EuroLine. Не только хорошо, но и честно, так как при заявленной производителем PPH Chemia Bomar (Скорогощ, Польша) начальной температуре кристаллизации -37°С мы получили -39°С. Кстати, EuroLine оказался единственным антифризом, на этикетке которого с обратной стороны емкости говорится именно о начальной температуре кристаллизации.

Образец №3 — антифриз Vaxoil производства COOO «Вакс-Авто» (Калинковичи, Беларусь). К температуре начала кристаллизации претензий нет. Температуру кипения хотелось бы выше. А к предупреждению на этикетке при работе с жидкостью не курить следует отнестись всерьез не только в связи с указанием «Осторожно! Ядовито», но и потому, что при испытании на воспламеняемость паров выяснилось, что они горят.

Образец №4 — тосол А-40М Arctic производства ООО «БЛЮ СТАР» (Дзержинск, Россия). Плохо все, в том числе и повторяемость результата на возгорание паров, ведь эта охлаждающая жидкость была среди 8 образцов, которые мы испытывали летом.

Образец №5 — антифриз ONZ OIL Optimal производства СООО «М-Стандарт» (Минский район, Беларусь), изготовленный по заказу ООО «Онза-Авто» (Минск, Беларусь). К параметрам вопросов нет, смущает только внушительный список спецификаций производителей, приведенный на этикетке. На упаковках ни одной из представленных выше охлаждающих жидкостей мы не видели ничего подобного. Это маркетинг или антифриз действительно был сертифицирован согласно допускам означенных в списке автомобильных компаний?

Образец №6 — антифриз Sintec ЗАО «Обнинскоргсинтез» (Обнинск, Россия). Температуру кипения хотелось бы чуть выше, ведь этикетка с обратной стороны емкости уверяет, что жидкость можно использовать до +110°С. Больше придраться не к чему. Летние испытания на возгораемость паров образец такой охлаждающей жидкости тоже прошел без замечаний.

Образец №7 — антифриз Venol производства Venol Motor Oil GmbH (Франкфурт-на-Одере, Германия). Когда на этикетке указано -35°С без уточнения, о чем конкретно идет речь, вопрос несоответствия температуры начала кристаллизации требованиям регламента ТС выглядит риторическим.

Образец №8 — антифриз Sibiria производства OOO «Дзержинский завод органического синтеза» (Дзержинск, Россия). Вновь вызывает смущение этикетка с внушительным списком автомобильных марок, которым продукт рекомендован. Кристаллизоваться антифриз начал при -37°С, а закипел при 108°С. Но не горит — это факт.

Образец №9 — антифриз MaxLane производства COOO «Белполиуретаны» (Могилев, Беларусь). На лицевой этикетке крупно написано: «Без метанола».

В это верим, ибо пары кипящей жидкости не загорелись, но давали ли согласие все автомобильные компании, перечисленные в этикетке с обратной стороны емкости, на применение MaxLane в своих двигателях — вопрос.

Образец №10 — тосол «Сибирь» А-40М, изготовленный ООО НПО «Органик-Прогресс» (Дзержинск, Россия). Среди участников экспертизы, которую мы проводили летом, был антифриз «Сибирь» этого же производителя. Не похоже, что между тосолом и антифризом есть разница: пары тосола тоже воспламенились, температуры начала кристаллизации и кипения антифриза оставляют желать лучшего.

Образец №11 — антифриз Pilot производства ООО «ТРЭКСТЭН» (Пушкино, Россия). Температура начала кристаллизации вполне себе, но по температуре кипения и способности гореть между этим образцом и «красным» Pilot, который приобретался для летней экспертизы, разницы не находим.

Образец №12 — антифриз Felix, выпущенный ООО «Тосол-Синтез-Инвест» (Дзержинск, Россия). Вопросов нет, за исключением ссылок на автопроизводителей, проходящих под вывеской Recommended for. Это рекомендация производителя Felix или автомобильных компаний?

Образец №13 — антифриз производства NordTec ПТЧУП «КиликияПлюс» (Минск, Беларусь). Заявление на этикетке, что NordTec используется в системе охлаждения от -40°С, учитывая определенную при испытаниях температуру начала кристаллизации -30°С, кажется излишне смелым, но других претензий нет.

Образец №14 — изготовитель Сергей Боярских. Что образцы были обезличенными, мне показалось мало. Втихаря я разбавил пополам один из купленных антифризов водой из крана и назвал продукт собственного приготовления «камнем преткновения». Когда он действительно превратился в камень при -10°С, в лаборатории поинтересовались, могу ли я раскрыть марку антифриза и его состав, заявленный производителем, ибо с образцом №14 происходит что-то непонятное. Так тайное стало явным, после чего мне посоветовали убрать этот образец из протокола экспертизы как не имеющий смысла. Так что в таблице результатов испытаний его нет, но по поводу смысла позволю себе не согласиться с коллегами. Бывает, что в дороге приходится доводить уровень охлаждающей жидкости до нормы и случается, что, кроме воды, больше долить в систему нечего. Что может произойти, если такую смесь не слить до морозов, «камень преткновения» продемонстрировал в лучшем виде.

Образец №15 — антифриз GreenCool производства СООО «М-Стандарт» (Минский район, Беларусь). Вопросов нет.

Образец №16 — антифриз «Гродно-Азот» АМ35, выпущенный ПТУП «АзотХимФортис» (Гродно, Беларусь). Вопросов нет, за исключением, может быть, несовременного вида тары, но ведь важно не то, как выглядит упаковка, а то, что в нее залито. Или все-таки важно и то и другое?

Образец №17 — антифриз X-Freeze производства ООО «Синтез-Пак» (Дзержинск, Россия). Вопросов нет.

Образец №18 — антифриз Arctic, выпущенный ООО «АкцентХим» (Дзержинск, Россия), стал вторым и незапланированным «камнем преткновения» экспертизы. Кристаллизоваться он начинал при -19°С, закипел при 96°С, но вопреки ожиданию не воспламенился. Из-за таких результатов мне тоже посоветовали не вносить образец №18 в протокол, а подвергнуть его дополнительным углубленным исследованиям. Насчет исследований посмотрим, а пока я решил, что читателям куда важнее знать, что Arctic трижды, а повторюсь, что на температуру кристаллизации все образцы проверялись по три раза, начал замерзать при температуре, которая в наших условиях эксплуатации куда более вероятна, чем 35-градусные морозы, на которых настаивает регламент ТС.

Наш вердикт

Если продается, значит, можно использовать — не так ли думают покупатели, увидев тот или иной товар на торговом прилавке? Однако результаты, показанные шестью из 17 взятых на экспертизу образцов, не отвечают требованиям Приложения I ТР ТС 030/2012 к температуре начала кристаллизации. Кроме того, семь образцов закипели рановато, испарения пяти из них воспламенились, что вызывает еще большие сомнения в целесообразности применения этих охлаждающих жидкостей в двигателе. Возникает и вопрос: как подобные продукты оказываются на прилавке?

Источник: https://www.abw.by/news/193857/

Тест ТОСОЛов и антифризов 2016

Наше издание регулярно готовит для своих читателей материалы, посвященные тестам автомобильных охлаждающих жидкостей. Последний опубликованный тест (http://www.autodela.ru/main/top/test/antifreeze_test_20_og) показал, что количество охлаждающих жидкостей, характеристики которых не соответствуют требованиям ГОСТ, из года в год не сокращается, особенно в сегменте бюджетных продуктов. Тенденция последнего времени выявила рост числа охлаждающих жидкостей с содержанием глицерина и метанола. Также были выявленные охлаждающие жидкости, содержащие смесь глицерина и этиленгликоля.

Давайте разберемся, чем опасно использование глицерина.

Глицерин предлагали использовать как компонент охлаждающей жидкости еще в начале ХХ века. Но проблема глицериновых охлаждающих жидкостей – высокая вязкость и недостаточная текучесть. Ее пытались решить за счет добавления этанола, метанола, солей, и т. д., пока не нашли полноценную замену: основой охладителя стал этиленгликоль. К 1937 голу антифризы на основе этиленгликоля практически вытеснили глицериновые и метаноловые.

При изготовлении охлаждающей жидкости с температурой замерзания –40 °С соотношение глицерина и воды должно составлять 65:35 % масс., в то время как для этиленгликоля и воды – 53:47 % масс., т е. при одинаковой температуре замерзания глицериновый антифриз содержит больше органического компонента (глицерина) и меньше воды, чем этиленгликолевый. Это приводит к дополнительному повышению плотности и вязкости, к снижению теплоемкости.

Но есть еще одна, более существенная проблема. Глицерин термически нестабилен, при длительном нагреве свыше 90 °С разлагается с образованием летучих веществ, в т.ч. акролеина. Продукты разложения токсичны, коррозионно агрессивны. Глицерин сильно пенится, и по этой причине отвод тепла ухудшается.

Из-за таких свойств, как высокая вязкость при минусовых температурах, при использовании глицерина в системе охлаждения двигателя все ее компоненты будут работать не в проектном режиме: многократно возрастет нагрузка на крыльчатку помпы, значительно увеличится механический износ от трения, что существенно сократит срок службы радиатора и помпы.

Но и это не все: при высоких температурах, которые возникают на поверхности цилиндров, глицерин разлагается, выпадая в осадок, забивая каналы охлаждения и значительно увеличивая коррозионную активность такой смеси. Следствием присутствия глицерина (и особенно метанола) является снижение температуры кипения до опасного уровня, не говоря о том, что скачкообразно растет кавитационный износ.

Дзержинский ТОСОЛ А40М – охлаждающая жидкость, тест

ЗАЯВЛЕННЫЕ ОСОБЕННОСТИ

Произведено ООО ПКФ «ПРОМПЭК»

«Дзержинский ТОСОЛ А40М» предназначен для охлаждения двигателя внутреннего сгорания автотракторной техники отечественного и импортного производства при температуре окружающего воздуха до –40 °С. Совместима с охлаждающими жидкостями отечественного и импортного производства на основе этиленгликоля.

РЕЗУЛЬТАТЫ ТЕСТА

Лабораторные исследования показали, что «Дзержинский ТОСОЛ А40М» имеет очень низкую температуру замерзания –55 °С. Казалось бы, можно радоваться: как-никак большой запас относительно температуры кристаллизации. Но при этом образец проваливает испытания по температуре кипения, которая вместо положенных ГОСТ 108 °С оказалась равной 96,5 °С.

Причину этого выявили в лаборатории. Охлаждающая жидкость «Дзержинский ТОСОЛ А40М» выполнена не на основе этиленгликоля, а на основе глицерина с добавлением метилового спирта, доля которого составила 5,9 %.

РЕЗЮМЕ

Использование в качестве основы для охлаждающей жидкости не этиленгликоля, а глицерина запрещено ГОСТ. Поэтому мы не рекомендуем использовать «Дзержинский ТОСОЛ А40М» в ДВС.

ZDA Antifreeze G11 – охлаждающая жидкость, тест

ЗАЯВЛЕННЫЕ ОСОБЕННОСТИ

Произведено ООО ПКФ «Завод дзержинских антифризов»

ZDA Antifreeze G11 предназначен для охлаждения двигателя внутреннего сгорания, а также для использования в качестве теплоносителя в теплообменных аппаратах и системах терморегулирования.

Состав: этиленгликоль, вода, присадки антикоррозионные, антипенные агенты, смазывающие добавки. Не содержит аминов, фосфатов, нитритов. Соответствует стандарту VW TL 774C (G11).

РЕЗУЛЬТАТЫ ТЕСТА

Лабораторные исследования показали, что ZDA Antifreeze G11 не уложился в порог температуры кристаллизации и вместо –40 °С замерз при температуре –37 °С. Зато он, в отличие от предыдущего состава, изготовлен из этиленгликоля, что положительным образом сказалось на температуре кипения, которая составила 108 °С.

РЕЗЮМЕ

Охлаждающую жидкость ZDA Antifreeze G11 мы не рекомендуем к использованию, так как она не уложилась в требования ГОСТ по температуре кристаллизации.

NIAGARA Antifreeze Universal G11&G12 – охлаждающая жидкость, тест

ЗАЯВЛЕННЫЕ ОСОБЕННОСТИ

Произведено ООО ПКФ «НИАГАРА»

Антифриз нового поколения NIAGARA Antifreeze Universal G11&G12 предназначен для использования в системах охлаждения современных высокофорсированных двигателей. Рекомендован для эксплуатации в тяжело нагруженных автомобилях. Содержит комбинированный пакет антикоррозионных и смазывающих присадок, обеспечивающий эффективную и долговечную защиту системы охлаждения двигателя от замерзания, перегрева и коррозии.

Состав: этиленгликоль, вода, функциональные присадки агенты, краситель.

РЕЗУЛЬТАТЫ ТЕСТА

Лабораторные исследования показали, что NIAGARA Antifreeze Universal G11&G12 превосходит требования ГОСТ. Температура кристаллизации состава –43,5 °С. Температура кипения также выше требований ТУ 111,5 °С. Лаборатория констатировала, что состав изготовлен на основе этиленгликоля.

РЕЗЮМЕ

Охлаждающая жидкость NIAGARA Antifreeze Universal G11&G12 превосходит требования ГОСТ и поэтому может быть рекомендован к применению.

NIAGARA RED G12+ – охлаждающая жидкость, тест

ЗАЯВЛЕННЫЕ ОСОБЕННОСТИ

Произведено ООО ПКФ «НИАГАРА»

Карбоксилатный антифриз нового поколения NIAGARA RED G12+ предназначен для использования в системах охлаждения современных двигателей. Рекомендован для эксплуатации в тяжело нагруженных автомобилях. Содержит комбинированный пакет антикоррозионных и смазывающих присадок, обеспечивающий эффективную и долговечную защиту системы охлаждения двигателя от замерзания, перегрева и коррозии. На упаковке производитель обещает бесперебойную работу состава на протяжении 250 тыс. км пробега.

Состав: этиленгликоль, вода, функциональные присадки агенты, краситель.

РЕЗУЛЬТАТЫ ТЕСТА

Лабораторные исследования показали, что NIAGARA RED G12+ укладывается в требования ГОСТ по температуре кристаллизации (–40,5 °С). А вот температура кипения оказалась ниже требуемых 108 °С, правда, совсем на чуть-чуть, и составила 107,5 °С. В лаборатории констатировали, что состав изготовлен из этиленгликоля.

РЕЗЮМЕ

Охлаждающая жидкость NIAGARA RED G12+ буквально на полградуса не уложилась в установленную температуру закипания охлаждающей жидкости.

Pilots Antifreeze G12 – охлаждающая жидкость, тест

ЗАЯВЛЕННЫЕ ОСОБЕННОСТИ

Произведено OOO «СТРЭКСТЭН» Delfin group

Pilots Antifreeze G12 имеет веселенькую упаковку, по дизайну больше напоминающую японское средство для стирки. Однако перед нами охлаждающая жидкость для применения в двигателях внутреннего сгорания. Заявляется, что состав обеспечивает надежное охлаждения двигателя в температурном диапазоне –40 до +50 °С.

Состав: вода, полиолы, алифатические спирты, антикоррозионные и противопенные присадки, краситель.

РЕЗУЛЬТАТЫ ТЕСТА

Лабораторные исследования показали, что Pilots Antifreeze G12 не соответствует требованиям ГОСТ. Его температура кристаллизации всего –36 °С, что выше требований ТУ, при низкой температуре кипения (всего 94 °С). Подобное поведение характерно для составов с использованием глицерина. Если внимательно читать этикетку, то можно увидеть, что в составе не указан этиленгликоль, при этом указано наличие полиолов. Заглядываем в Сеть и находим, что это многоатомные спирты, к которым относится и глицерин. Что и выявил лабораторный анализ. Кстати, анализ также показал наличие метилового спирта в количестве 9,31 %.

РЕЗЮМЕ

Использование в качестве основы для охлаждающей жидкости не этиленгликоля, а глицерина и метилового спирта запрещено ГОСТ. Поэтому мы не рекомендуем использовать Pilots Antifreeze G12 в ДВС.

Vitex G12 ultra – охлаждающая жидкость, тест

ЗАЯВЛЕННЫЕ ОСОБЕННОСТИ

Произведено OOO «Ви Кемикал»

Высококачественный, готовый к применению антифриз Vitex G12 ultra предназначен для использования в системе охлаждения двигателей последнего поколения с алюминиевым блоком цилиндров. Усовершенствованная карбоксилатная основа антифриза Vitex G 12 Ultra G безопасна для металлических частей всех современных двигателей. Благодаря усовершенствованному составу карбоксилатной основы и уникальному антикоррозионному пакету присадок продукт увеличивает резерв работы двигателя, что позволяет не думать о замене антифриза до 200 000 км.

Мягкая формула антифриза не содержит в своем составе аминов, нитритов, фосфатов, силикатов и боратов, вызывающих образование геля и осадка. Поэтому использование антифриза Vitex G12 Ultra G предотвращает загрязнение системы охлаждения двигателя. Уникальный пакет присадок Ultra SAFE придает охлаждающей жидкости антикоррозионные и антивспенивающие свойства и обеспечивает ее безопасное применение.

Действуя на молекулярном уровне, пакет присадок проявляет защитные свойства только в тех местах двигателя, которые подверглись коррозионному воздействию. В точках повреждений образуется защитный слой, препятствующий образованию и развитию коррозии и обеспечивающий безопасную работу двигателя.

Произведено из концентрата Alpine Mitan GmbH (Германия), имеющего допуски Mercedes-Benz Blatt 325.3, MAN 324 SNF, VW TL 774 D/F.

Состав: вода, этиленгликоль, пакет присадок Ultra SAFE, антипенные агенты, смазывающие добавки.

РЕЗУЛЬТАТЫ ТЕСТА

Лабораторные исследования показали, что Vitex G 12 ultra превосходит требования ГОСТ. Температура кристаллизации состава оказалось –42,5 °С. Температура кипения также выше требований ТУ: 109,5 °С. Лаборатория констатировала, что состав изготовлен на основе этиленгликоля.

РЕЗЮМЕ

Охлаждающая жидкость Vitex G 12 ultra превосходит требования ГОСТ и поэтому рекомендуем к применению.

Felix Prolonger – охлаждающая жидкость, тест

ЗАЯВЛЕННЫЕ ОСОБЕННОСТИ

Произведено OOO «Тосол-Синтез-Инвест»

Professional Antifreeze FELIX PROLONGER предназначен для использования во всех легковых и грузовых автомобилях, в том числе высоконагруженных и форсированных, эксплуатируемых в тяжелых климатических и дорожных условиях.

По заявлению производителя, состав обеспечивает 100% защиту двигателя и деталей системы охлаждения от всех видов коррозии, перегрева и переохлаждения. Увеличенный пробег без замены охлаждающей жидкости – до 120 000 км. Обладает усиленной и пролонгированной защитой металлов (в том числе алюминия), сплавов, резины. Его применение исключает возможность образования накипи и отложений, при этом состав обладает высокими смазывающими свойствами и низким пенообразованием.

Эффективен при температурах окружающего воздуха от –45 до +50 °С. Обеспечивает быстрый прогрев двигателя до нужной температуры. Обладает превосходной теплопроводностью и теплоемкостью. Улучшает работу и продлевает срок службы помпы, термостата, радиатора.

Содержит запатентованный многофункциональный пакет антикоррозионных, антикавитационных, антипенных и смазывающих присадок. Изготовлен из моноэтиленгликоля высшего сорта и специально подготовленной деминерализованной воды высшей степени очистки. Неорганический антифриз класса G11 по классификации VW.

FELIX PROLONGER отвечает требованиям крупнейших автопроизводителей и международным стандартам качества: Audi: TL 774-C, BMW: BMW N 600 69.0, German Army: TL 6850-0038/1, KHD: H-LV 0161 0188, MAN: MAN 324-NE, Mercedes-Benz: DBL 7700.20 page 325.0, MTU: MTL 5048, Opel/General Motors: B 040 0240, Saab: 6901599, Seat: TL 774-C, Scoda: TL 774-C, Volkswagen: TL 774-C (VW code G11), Volvo 1286083, Deutz AG: 0199-99-1115 and 0199-99-2091, GE Jenbacher: TA 1000-0201, Chrysler MS 7170, John Deere h44C1, ASTM D 3306, ASTM D 4340, SAE J 1034, BS 6580-1992, AFNOR R15-601.

РЕЗУЛЬТАТЫ ТЕСТА

Лабораторные исследования показали, что FELIX PROLONGER соответствует требованиям ГОСТ. Температура кристаллизации состава равна –40,0 °С. Температура кипения выше требований ТУ: 110 °С. Лаборатория констатировала, что состав изготовлен на основе этиленгликоля.

РЕЗЮМЕ

Охлаждающая жидкость FELIX PROLONGER превосходит требования ГОСТ и поэтому рекомендуется к применению.

CoolStream Standard – охлаждающая жидкость, тест

ЗАЯВЛЕННЫЕ ОСОБЕННОСТИ

Произведено OАO «ТЕХНОФОРМ»

CoolStream Standard– универсальная охлаждающая жидкость зеленого цвета наоснове этиленгликоля, выпускаемая покарбоксилатнойOAT-технологии(Organic Acid Technology). Изготавливается изсуперконцентрата Corrosion Inhibitor BSB компании Arteco (Belgium) иявляется точной копией (ребрендом)AntifreezeBS-Coolant.

Обладает следующими преимуществами: нет блокирующего слоя (нет силикатов), улучшенная теплопередача, высокотемпературная стабильность ингибиторов, нет зависимости свойств отжесткости воды, для концентрата (нет фосфатов), необразуются осадки игели (нет силикатов), обеспечивает до 150 000 км пробега без замены.

Спецификации:BS 6580, ASTM D3306 BS 6580:1992, BS 6580:2010 [1] GB 29743-2013 NB/SH/T 0521-2010.

Допуски и первоначальная заправка:КАМАЗ, ГАЗ, МАЗ, АвтоВАЗ, ЛиАЗ, НЕфАЗ, ПАЗ, ММЗ, ЯМЗ, УАЗ.

Антифриз CoolStream Standard отвечает требованиям производителейтяжелонагруженнойтехники ииспользуется как антифриз постгарантийного обслуживания грузовиков.

Состав: вода, этиленгликоль, пакет присадок.

РЕЗУЛЬТАТЫ ТЕСТА

Лабораторные исследования показали, что CoolStream Standard не соответствует требованиям ГОСТ по температуре кристаллизации состава, которая оказалась выше, чем требуемые –40 °С, и оказалось составила – 37 °С. При этом температура кипения охлаждающей жидкости выше требований ТУ: 110 °С. Лаборатория констатировала, что состав изготовлен на основе этиленгликоля.

РЕЗЮМЕ

Охлаждающая жидкость CoolStream Standard не уложилась в требования ГОСТ по требуемой температуре кристаллизации.

NORD – охлаждающая жидкость, тест

ЗАЯВЛЕННЫЕ ОСОБЕННОСТИ

Произведено OОO «Гелена химавто»

Антифриз NORD – универсальная охлаждающая жидкость, предназначенная для заправки систем охлаждения автомобильных двигателей всех марок (отечественных и зарубежных), эксплуатируемых при температуре окружающего воздуха от –40 С до +50 °С. Нормативный срок службы: 4 года или 120 тыс. км пробега.

По заверениям производителя, состав обеспечивает отличное охлаждение двигателя, увеличенный срок службы водяного насоса, мощную антикоррозионную защиту радиатора. При применении антифриза NORD отсутствует кавитационная коррозия в каннах головки блока и блока цилиндров, состав облегчает поиск мест утечки за счет флуоресцирующего красителя.

Состав: вода, этиленгликоль, антикоррозионные, антипенные, моюще-диспергирующие, антифрикционные и стабилизирующие присадки.

РЕЗУЛЬТАТЫ ТЕСТА

Лабораторные исследования показали, что антифриз NORD не соответствует требованиям ГОСТ по температуре кристаллизации состава, которая оказалась выше, чем требуемые –40 °С, и составила –37 °С. При этом температура кипения охлаждающей жидкости также оказалось ниже требований ТУ: 107,5 °С. Лаборатория констатировала, что состав изготовлен из смеси моно- и диэтиленгликоля.

РЕЗЮМЕ

Охлаждающая жидкость NORD не уложилась в требования ГОСТ и поэтому не рекомендуется к применению в ДВС.

SINTEC EURO G11 – охлаждающая жидкость, тест

ЗАЯВЛЕННЫЕ ОСОБЕННОСТИ

Произведено OOO «Обнинскоргсинтез»

SINTEC EURO G11 – охлаждающая жидкость последнего поколения, созданная на основе высококачественного моноэтиленгликоля и импортных ингибиторов коррозии.

Применяется в системе охлаждения легковых автомобилей отечественного и зарубежного производства, грузовиков и других транспортных средств со средними и тяжелыми условиями эксплуатации. Обеспечивает правильный тепловой режим эксплуатации двигателя, предотвращает появление коррозии и отложений в системе охлаждения. Не допускает замерзания, перегрева, выкипания и появления воздушных пробок.

Обладает эффективными смазывающими свойствами, продлевающими ресурс водяного насоса. Не повреждает шланги, прокладки и уплотнения системы охлаждения.

Имеет допуски: DERWAYS, Uz DAEWOO, ФУЗО КАМАЗ Тракс Рус.

РЕЗУЛЬТАТЫ ТЕСТА

Лабораторные исследования показали, что SINTEC EURO G11 соответствует требованиям ГОСТ. Температура кристаллизации состава оказалось –40,5 °С. Температура кипения выше требований ТУ: 110,5 °С. Лаборатория констатировала, что состав изготовлен на основе этиленгликоля.

РЕЗЮМЕ

Охлаждающая жидкость SINTEC EURO G11 соответствует требованиям ГОСТ и поэтому рекомендуется к применению.

Oil Right Тосол Дзержинский ОЖ-40 – охлаждающая жидкость, тест

ЗАЯВЛЕННЫЕ ОСОБЕННОСТИ

Произведено OОO «Промсинтез»

Охлаждающая жидкость «Oil Right Тосол Дзержинский ОЖ-40» предназначена к применению в системах охлаждения двигателей внутреннего сгорания автомобилей.

Отличительные особенности состава, заявленные производителем: обладает высокими антикоррозионными свойствами; эффективно предохраняет систему охлаждения от замерзания зимой и от закипания летом; снижает вероятность образования отложений и выпадения осадка в системе охлаждения; способствует увеличению срока службы термостата, радиатора и водяного насоса.

Соответствует международным стандартам ASTM D 3306, SAE J 1034 и спецификациям автомобильных компаний Audi, BMW, Opel, Volvo, Volkswagen.

Состав: вода, этиленгликоль, попионы, алифатические спирты, антикоррозионные, противопенные присадки, краситель.

РЕЗУЛЬТАТЫ ТЕСТА

Лабораторные исследования показали, что «Oil Right Тосол Дзержинский ОЖ-40» не соответствует требованиям ГОСТ по температуре кристаллизации состава, которая выше, чем требуемые –40 °С, и составила –37 °С. Температура кипения охлаждающей жидкости также оказалось ниже требований ТУ: 92,5 °С.

Подобное поведение характерно для составов с использованием глицерина. Если внимательно читать этикетку, то в составе указано наличие полиолов, которые представляют собой многоатомные спирты, к коим относится и глицерин. Его наличие в составе показал и лабораторный анализ. При этом также используется смесь моно- и триэтиленгликоля.

РЕЗЮМЕ

Охлаждающая жидкость «Oil Right Тосол Дзержинский ОЖ-40» не уложилась в требования ГОСТ и поэтому не рекомендуется к применению в ДВС.

Antifreeze Extra -42 G11 – охлаждающая жидкость, тест

ЗАЯВЛЕННЫЕ ОСОБЕННОСТИ

Произведено OОO ПКФ «Промпэк»

Охлаждающая жидкость Antifreeze Extra -42 G11 предназначена для использования в системах охлаждения любых современных бензиновых и дизельных двигателей автомобилей всех марок иностранного и российского производства при температуре окружающего воздуха до –42 °С.

По заявлению производителя, состав идеален для высокофорсированных двигателей и двигателей, эксплуатирующихся в тяжелых условиях. Содержит оптимально сбалансированный пакет антиокислительных, антипенных и антифрикционных присадок.

Имеет повышенную активационную защиту припоя, алюминия, меди, латуни, стали и чугуна.

Антифриз Antifreeze Extra -42 G11 разработан с учетом требований ASTM D3306/4656 и VW TL 774C (G11).

Состав: вода, моноэтиленгликоль, пакет присадок, краситель.

РЕЗУЛЬТАТЫ ТЕСТА

Лабораторные исследования показали, что Antifreeze Extra -42 G11 не соответствует требованиям ГОСТ по температуре кристаллизации состава, которая ваше, чем требуемые –40 °С, и оказалось равной –39 °С. При этом температура кипения охлаждающей жидкости выше требований ТУ: 111 °С. Лаборатория констатировала, что состав изготовлен на основе этиленгликоля.

РЕЗЮМЕ

Охлаждающая жидкость Antifreeze Extra -42 G11 не уложилась в требования ГОСТ по температуре кристаллизации.

Тосол Vitex А40D – охлаждающая жидкость, тест

ЗАЯВЛЕННЫЕ ОСОБЕННОСТИ

Произведено OOO «Ви Кемикал»

Тосол Vitex А40D предназначен для охлаждения двигателей внутреннего сгорания, а также для использования в качестве теплоносителя в теплообменных аппаратах и системах терморегулирования. Совершенная рецептура изготовления тосола на основе ультранового комплекса присадок, позволяет двигателю автомобиля работать без перегрузок и защищает его от коррозии.

Тосол Vitex А-40D изготовлен из высококачественного сырья (моноэтиленгликоля) с применением специализированного пакета присадок. Антикоррозионные, антивспенивающие и защитные свойства присадок обеспечивают надежную защиту конструкционных материалов двигателей российского и иностранного производства.

Тосол Vitex А-40D полностью соответствует требованиям ГОСТ 28084-89 и ASTM. Благодаря высоким качественным показателям является экономичной альтернативой антифризу класса G11.

Обладает отличными защитными свойствами. Не содержит солей, образующих нежелательные осадки. Безопасен для пластика, резиновых изделий, лакокрасочных покрытий. Не содержит фосфатов, аминов, нитритов. Совместим с другими охлаждающими жидкостями на основе этиленгликоля.

РЕЗУЛЬТАТЫ ТЕСТА

Лабораторные исследования показали, что тосол Vitex А-40D соответствует требованиям ГОСТ. Температура кристаллизации состава оказалось –40 °С. Температура кипения выше требований ТУ: 110,5 °С. Лаборатория констатировала, что состав изготовлен на основе этиленгликоля.

РЕЗЮМЕ

Тосол Vitex А-40D соответствует требованиям ГОСТ и поэтому рекомендуется к применению в ДВС.

Дзержинский ТОСОЛ А40М – охлаждающая жидкость, тест

ЗАЯВЛЕННЫЕ ОСОБЕННОСТИ

Произведено OОO «Дзержинские антифризы»

Охлаждающая жидкость «Тосол Дзержинский А40М» предназначена для охлаждения двигателей внутреннего сгорания.

Состав изготовлен на основе этиленгликоля. Содержит антипенные присадки. Применяется при температуре окружающей среды не ниже –40 °С.

РЕЗУЛЬТАТЫ ТЕСТА

Лабораторные исследования показали, что «Тосол Дзержинский А40М» не соответствует требованиям ГОСТ по температуре кристаллизации состава, которая выше, чем требуемые –40 °С, и оказалось равной –30,5 °С. Температура кипения охлаждающей жидкости также оказалось ниже требований ТУ: 101 °С.

Подобное поведение характерно для составов с использованием глицерина, что и выявил лабораторный анализ. При этом в продукте используется и этиленгликоль. Также было выявлено содержание в охлаждающей жидкости метилового спирта в концентрации 4,45 %.

РЕЗЮМЕ

Охлаждающая жидкость «Тосол Дзержинский А40М» не уложилась в требования ГОСТ и поэтому не рекомендуется к применению в ДВС.

MR.AUTO Дзержинский antifrize Тосол-40 – охлаждающая жидкость, тест

ЗАЯВЛЕННЫЕ ОСОБЕННОСТИ

Произведено OОO «Нова-нефтехим»

Охлаждающая жидкость «MR.AUTO Дзержинский antifrize Тосол-40» предназначена для охлаждения двигателей внутреннего сгорания, а также в качестве рабочей жидкости в других теплообменных аппаратах, эксплуатируемых при низких температурах.

Производитель заявляет об использовании технологии органического синтеза, наличии в охлаждающей жидкости оптимизированного пакета присадок, предназначенного для защиты системы охлаждения. Заявляется о гарантии качества продукта. Указано, что производство соответствует стандарту EURO ISO 9001.

РЕЗУЛЬТАТЫ ТЕСТА

Лабораторные исследования показали, что «MR.AUTO Дзержинский antifrize Тосол-40» не соответствует требованиям ГОСТ по температуре кристаллизации состава, которая выше, чем требуемые –40 °С, и составила –29 °С. Температура кипения охлаждающей жидкости также оказалось ниже требований ТУ: 94,5 °С.

Подобное поведение характерно для составов с использованием глицерина, что и показал лабораторный анализ. При этом также используется смесь моно- и диэтиленгликоля.

Также было выявлено содержание в охлаждающей жидкости метилового спирта в концентрации 7,79 %.

Возникает резонный вопрос: о каком использовании стандарта EURO ISO 9001 на производстве может идти речь при таком низком качестве выпускаемого продукта?

РЕЗЮМЕ

Охлаждающая жидкость «MR.AUTO Дзержинский antifrize Тосол-40» не уложилась в требования ГОСТ и поэтому не рекомендуется к применению в ДВС.

Pilots Тосол ОЖ-40 – охлаждающая жидкость, тест

ЗАЯВЛЕННЫЕ ОСОБЕННОСТИ

Произведено OOO «СТРЭКСТЭН» Delfin group

Pilots Тосол ОЖ-40 – охлаждающая жидкость для применения в двигателях внутреннего сгорания. Заявляется, что состав обеспечивает надежное охлаждение двигателя в температурном диапазоне –40 до +50 °С.

Состав: вода, полиолы, алифатические спирты, антикоррозионные и противопенные присадки, краситель.

РЕЗУЛЬТАТЫ ТЕСТА

Лабораторные исследования показали, что Pilots Тосол ОЖ-40 не соответствует требованиям ГОСТ. Его температура кристаллизации всего –35,5 °С, что выше требований ТУ, при низкой температуре кипения всего 92 °С. Подобное поведение характерно для составов с использованием глицерина. Если внимательно читать этикетку, то в составе не указан этиленгликоль, при этом говорится о наличии полиолов. Это многоатомные спирты, к которым относится и глицерин (что и показал лабораторный анализ). Также установлено наличие метилового спирта в количестве 11,23 %.

РЕЗЮМЕ

Использование в качестве основы для охлаждающей жидкости не этиленгликоля, а глицерина (как и метилового спирта) запрещено ГОСТ. Поэтому мы не рекомендуем использовать Pilots Тосол ОЖ-40 в ДВС.

Тосол-А40М «БАРС» – охлаждающая жидкость, тест

ЗАЯВЛЕННЫЕ ОСОБЕННОСТИ

Произведено OOO «Тосол НН»

«Тосол-А40М «БАРС» – охлаждающая жидкость для применения в системах охлаждения всех типов отечественных и импортных автомобилей. Обладает антикоррозионными свойствами, совместима с резиновыми и пластиковыми деталями в системе охлаждения. Состав не агрессивен к стали и цветным металлам.

Состав: вода, этиленгликоль, антикоррозионные ингибиторы и противопенные присадки.

На сайте производителя заявлено, что тосол «Барс» – это высококачественныйтосол-А40М классического сине-голубого цвета. Благодаря выбранному наилучшему соотношению цена–качество «Тосол-А40М«Барс»быстро заняла лидирующие позиции на рынке автохимии. Заявленный срок эксплуатации – 150 тыс. км пробега.

РЕЗУЛЬТАТЫ ТЕСТА

Лабораторные исследования показали, что «Тосол-А40М «БАРС» не соответствует требованиям ГОСТ. Его температура кристаллизации всего –26,5 °С. По этому параметру данный продукт – антилидер нашего теста: более высокой температуры кристаллизации не показал ни один образец. Температура кипения состава также ниже технических требований: 106,5 °С.

Лаборатория констатировала, что состав изготовлен на основе этиленгликоля.

РЕЗЮМЕ

Охлаждающая жидкость «Тосол-А40М «БАРС» не уложилась в требования ГОСТ и поэтому не рекомендуется к применению в ДВС.

LUXE Green Line ANTIFREEZE Long Life – охлаждающая жидкость, тест

ЗАЯВЛЕННЫЕ ОСОБЕННОСТИ

Произведено АO «Делфин Индастри» Delfin group

Антифризы LUXE обладают высокими эксплуатационными свойствами, обеспечивающими надежную защиту системы охлаждения откоррозии иобразования накипи. Поряду параметров превосходят все отечественные аналоги. Совместимы совсеми типами охлаждающих жидкостей наоснове этиленгликоля. Обеспечивают надежную работу системы охлаждения двигателя при температуре окружающей среды от–40до+50 °С.

Низкозамерзающая охлаждающая жидкостьпредназначенадля использования всистемах охлаждения двигателей внутреннего сгорания. Всостав зеленого антифриза LUXE входит флуоресцентный краситель, использование которого позволяет быстро определить места протечки. Для этого достаточно просветить предполагаемое место ультрафиолетовой лампой.

По заявлению производителя, состав отвечает требованиям VWTL774-C; Opel GMQL130100; MB-Approval 325.0/325.2; BMW GS9400; MAN 324NF.

РЕЗУЛЬТАТЫ ТЕСТА

Лабораторные исследования показали, что LUXE Green Line ANTIFREEZE Long Life не соответствует требованиям ГОСТ по температуре кристаллизации, которая при разбавлении 1:1 составила –35,5 °С, что выше требований ТУ и выше значений, указанных на этикетке (– 38 °С). Температура кипения оказалась в норме: 110 °С.

Лаборатория констатировала, что состав изготовлен на основе этиленгликоля.

РЕЗЮМЕ

Охлаждающая жидкость LUXE Green Line ANTIFREEZE Long Life не уложилась в требования ГОСТ по требуемой температуре кристаллизации.

Тосол ОЖ-40 – охлаждающая жидкость, тест

ЗАЯВЛЕННЫЕ ОСОБЕННОСТИ

Произведено ООO «Промсинтез»

«Тосол ОЖ-40» – низкозамерзающая охлаждающая жидкость, которая гарантирует надежную работу системы охлаждения двигателя при температурах окружающей среды от–40до+50 °С. Предназначена для использования в системах охлаждения двигателей внутреннего сгорания, а также в качестве рабочей жидкости в других теплообменных аппаратах. Обеспечивает увеличение срока службы охлаждающей жидкости, стойкую антикоррозионную защиту металлов и сплавов, инертность к резиновым и керамическим деталям. Предотвращает образование накипи.

Состав: вода, этиленгликоль, полиолы, алифатические спирты, антикоррозионные и противопенные присадки, краситель.

РЕЗУЛЬТАТЫ ТЕСТА

Лабораторные исследования показали, что «Тосол ОЖ-40» не соответствует требованиям ГОСТ. Его температура кристаллизации всего –27,5 °С, что выше требований ТУ, при низкой температуре кипения – всего 93,5 °С. Подобное поведение характерно для составов с использованием глицерина. Если внимательно читать этикетку, то в составе указано наличие полиолов. Это многоатомные спирты, к которым относится и глицерин (его наличие выявил лабораторный анализ). Лабораторное исследование также показало наличие метилового спирта в количестве 9,59 %.

РЕЗЮМЕ

Использование в качестве основы для охлаждающей жидкости не этиленгликоля, а глицерина запрещено ГОСТ, равно как и метилового спирта. Поэтому мы не рекомендуем использовать «Тосол ОЖ-40» в ДВС.

Аляsка Тосол 40 – охлаждающая жидкость, тест

ЗАЯВЛЕННЫЕ ОСОБЕННОСТИ

Произведено ООO «Тектрон» Delfin group

«Аляsка Тосол 40» – низкозамерзающая охлаждающая жидкость, предназначенная для работы в системе охлаждения двигателя при температурах окружающей среды от–40до+50 °С.

Новейшая формула обеспечивает увеличение срока службы тосола, стойкую антикоррозионную защиту металлов и сплавов, инертность к резиновым и керамическим деталям. Предотвращает образование накипи.

Состав: вода, этиленгликоль, полиолы, алифатические спирты, антикоррозионные и противопенные присадки, краситель.

РЕЗУЛЬТАТЫ ТЕСТА

Лабораторные исследования показали, что «Аляsка Тосол 40» не соответствует требованиям ГОСТ. Температура кристаллизации –35,5 °С, что выше требований ТУ, при низкой температуре кипения – всего 95 °С. Подобное поведение характерно для составов с использованием глицерина. В составе указано наличие полиолов – многоатомных спиртов, к которым относится и глицерин. По результатам лабораторного анализа состав содержит глицерин, а также метиловый спирт в количестве 9,94 %.

РЕЗЮМЕ

Использование в качестве основы для охлаждающей жидкости не этиленгликоля, а глицерина запрещено ГОСТ, как и метилового спирта. Поэтому мы не рекомендуем использовать «Аляsка Тосол 40» в ДВС.

Журнал «АВТОДЕЛА»

Охлаждающие жидкости… горят! Экспертиза «За рулем».

Недавно по телевидению прошел сюжет про творог, который горит! Я не знаю, что туда подмешали. Но абсолютно уверен, что получить легковоспламеняющуюся охлаждающую жидкость намного проще. Достаточно при производстве вместо дорогого этиленгликоля использовать дешевый глицерин и добавить метанол для приведения вязкости продукта к приемлемому уровню. Причем этим занимаются многие производители. И никому не говорят, что температура кипения метанола — всего-навсего 64,5°C.  Зачем, дескать, пугать людей?

Чем метанол опасен для моторов, мы писали неоднократно. Но никогда не заостряли внимание на том, что утечка в системе охлаждения может привести… к пожару. Попади метанол, скажем, на раскаленный коллектор — и привет: вспоминайте, как звонить с мобильника пожарным. Не верите? Мы провели небольшие испытания.

Закупив в подмосковном магазинчике дюжину различных охлаждающих жидкостей, именуемых тосолами и антифризами, мы напросились в гости в химлабораторию — измерить температуру кипения и определить таким образом наличие метанола. В серьезных испытаниях для этого обычно используют хроматограф, который легко покажет на экране состав проверяемой жидкости. Но наша цель была иной. Хроматограф — это нечто заумное, а электрическая плитка и термометр сделают проверку простой и наглядной. Особенно когда на плитке одновременно стоят два стаканчика с жидкостями, одна из которых вовсю кипит, а вторая и не думает. Именно такой эксперимент мы и провели. Тем, кто захочет его повторить, очень советуем раздобыть специальные термостойкие стаканчики, которые не станут трескаться в неподходящий момент.

Мы нагревали каждую приобретенную жидкость, фиксируя температуру начала кипения (появление первых пузырьков) и собственно температуру кипения. Полученные результаты приводим в нашей фотогалерее.

Вспомнив про горящий творог, мы попытались поочередно поджечь все проверяемые жидкости зажигалкой. Результат жутковатый: пять образцов из дюжины мгновенно вспыхнули. Более того, если на плитке одновременно находились два стаканчика — один со стандартным антифризом, а другой с метанольным коктейлем, то при попытке поджечь хорошую жидкость вспыхивал… соседний состав. Пары метанола очень не любят раскаленных поверхностей и открытого огня!

Результаты наших исследований относятся к конкретным образцам приобретенной нами продукции, а потому не могут служить основанием для выводов о деятельности той или иной компании. Вполне возможно, что некоторые производители повторят наш эксперимент — прежде всего те, чья продукция на поверку оказалась небезопасной.

Еще один простейший способ выявить жидкость низкого качества — использовать тест-полоски, определяющие наличие алкоголя в слюне. Во всех случаях индикаторы, искупавшись в метанольном коктейле, поменяли цвет.

А что делать потребителю? Кипятить товар перед покупкой ему точно не разрешат. Один из способов оградить себя от неприятностей — приобретать только те товары, которые хорошо зарекомендовали себя в наших экспертизах. Избегайте торговых развалов на обочинах дорог. Хотя и в магазинах (как следует из нашего эксперимента) не всегда продают качественный товар. Причем продавцы иногда понятия не имеют, какие неприятности сулит нормальная с виду канистра. Поэтому еще один совет: пользуйтесь услугами проверенного автосервиса. Сервисмены судят о качестве приобретенной партии товара, устраивая выборочную проверку. Это куда надежнее единичных закупок.

Sintec Euro G11, Россия

Температура начала кипения 99 ºС Температура кипения 105 ºС

НЕ ГОРИТ!

Антифриз SINTEC EURO – охлаждающая жидкость последнего поколения, созданная на основе высококачественного моноэтиленгликоля и импортных ингибиторов коррозии.  Обладает смазывающими свойствами, продлевающими ресурс водяного насоса. Не повреждает резиновые детали системы охлаждения.

Аntifreeze Maxim, Россия

Температура начала кипения 70 ºС Температура кипения 84 ºС

ГОРИТ!

Antifreeze G11 Pilot, Россия

Температура начала кипения 70 ºС Температура кипения 84 ºС

ГОРИТ!

Unix Antifreeze, Россия

Температура начала кипения 79 ºС Температура кипения 88 ºС

ГОРИТ!

Nord Antifreeze, Россия

Температура начала кипения 106 ºС Температура кипения 109 ºС

НЕ ГОРИТ!

ТCL Anti-Freeze, Япония

Температура начала кипения 100 ºС Температура кипения 105 ºС

НЕ ГОРИТ!

Антифриз CoolStream Standard —40 ºС, Россия

Температура начала кипения 105 ºС Температура кипения 109 ºС

НЕ ГОРИТ!

Тосол —40 Felix, Россия

Температура начала кипения 99 ºС Температура кипения 105 ºС

НЕ ГОРИТ!

Тосол Айсберг, Россия

Температура начала кипения 103 ºС Температура кипения 106 ºС

НЕ ГОРИТ!

Тосол Аляsка, Россия

Температура начала кипения 91 ºС Температура кипения 93 ºС

ГОРИТ!

Тосол Дзержинский Maxim, Россия

Температура начала кипения 70 ºС Температура кипения 89 ºС

ГОРИТ!

Тосол-север 40, Россия

Температура начала кипения 103 ºС Температура кипения 106 ºС

НЕ ГОРИТ!

Источник: www. zr.ru

Тесты (За Рулем, Авторевю, Главная Дорога и др.)

Чугунным моторам и латунным радиаторам дре­мучих автомобилей подобная смесь не грозила кор­розией. А вот в более современных двигателях горя­чий антифриз начал выгрызать куски металла. Поэто­му химики создали новую охлаждающую жидкость. Первые три буквы ее названия взяли с таблички над дверью отдела: «Технология органического синтеза». Окончание «ол» позаимствовали из химической тер­минологии. Так появился «Тосол».

Название превратилось в имя нарицательное. Од­нако в эпоху безграмотности ситуация изменилась: этим именем начали называть любые охлаждающие жидкости для машин отечественного производства. «Тосол» и «антифриз» стали чуть ли не синонимами определений «плохой» и «хороший». К сожалению, та­кое разделение охлаждающих жидкостей поддержа­ли все — от оптовиков и розничных торговцев до ав­томобилистов. В то, что сегодня называют тосолом, производители чаще всего добавляют присадки, обеспечивающие разве что минимальные антикор­розионные свойства. И это вполне объяснимо: деше­во и вполне достаточно для Жигулей. А вот что зали­вают в яркие и привлекательные канистры с надпися­ми «Антифриз», мало кому известно. И тем более не­известно, как оно отразится на здоровье современно­го двигателя дорогой иномарки.

Избежать неприятностей вам помогут наши экс­пертизы За Рулем.

Давайте заглянем внутрь двигателя. Поршни носятся вверх-вниз, вращается коленчатый вал. Подобная «гимнастика» неизбежно вызывает колебания стенок цилиндров: гильзы начинают вибрировать. Охлаждающая жидкость испытывает перепады давления, периодически вскипая. При этом на наружных стенках гильз возникают воздушные пузырьки, которые, лопаясь, порождают эффект кавитации. И если жидкость легкокипящая (с глицерином и метанолом), кавитация резко усиливается. Пузырьки взрываются, выгрызая из гильз кусочки металла. После этого — два пути: капремонт или помойка. Аналогичным образом от кавитации страдает крыльчатка насоса охлаждающей жидкости.

Кадр посвящается тем, кто полагает, будто все охлаждающие жидкости разливаются из одной бочки. Ничего подобного! Синяя уже кипит, а вот красная и не думает.

Кроме того, если в системе охлаждения возникают паровые пробки, то охлаждающая жидкость перестает циркулировать. Двигатель начинает перегреваться, метанол из охлаждающей жидкости выкипает, остается водный раствор глицерина с температурой кристаллизации от -20 до -12 °С. И такой «антифриз» вполне может замерзнуть.

Страшилка для тех, кто верит сказкам про безвредность охлаждающих жидкостей с метанолами и глицеринами. В прежней жизни это была качественная чугунная гильза. Но борьбу с кавитацией она проиграла нокаутом. Вместе с двигателем, естественно.

Что получилось в тесте?

В тесте участвовало около трех десятков жидкостей — и почти половина не выдержала проверку! Мы измеряли у каждой жидкости температуру кипения и пытались ее поджечь.

Фрагмент испытаний охлаждающих жидкостей на коррозию. Полная проверка занимает 336 часов при температуре 88 °С. Каждая жидкость проходит проверку на шести различных образцах: медь, припой, латунь, сталь, чугун, алюминий. В конце теста оценивают изменение массы каждой пластинки, а также изучают ее внешний вид.

По науке наличие метанола, как и иных веществ, оценивают на хроматографах. Но мы специально ставили наглядный эксперимент. Результаты плачевные: почти половина проверенных охлаждающих жидкостей может отправить двигатель в капремонт.

Тех, кому захочется повторить наши эксперименты, предупреждаем: использовать следует только химические стаканчики из тонкого стекла. Граненый может лопнуть! Перчатки тоже не помешают.

Чуть больше года назад (ЗР, № 8, 2016) мы провели аналогичную экспертизу — тогда из дюжины протестированных охлаждающих жидкостей половина оказалась пожароопасной. Три из них — Pilot, Unix и Аляsка — вновь попали к нам на проверку. И опять та же картина: горят синим пламенем. Видимо, производители за это время не успели (или не захотели) изменить технологию и наладить выпуск негорючих охлаждающих жидкостей. Самым страшным для них будет наказание рублем — если люди перестанут покупать их продукцию. Равно как и другие охлаждающие жидкости, не выдержавшие испытания.

Результаты теста — в фотогалерее. Они относятся только к конкретным образцам приобретенной нами продукции, а потому не могут служить основанием для каких-либо выводов о деятельности той или иной компании в целом.

Как выбрать хорошую охлаждающую жидкость? Самый простой и надежный вариант — воспользоваться результатами наших исследований и приобретать только ту продукцию, которая успешно выдержала испытание.

Вы продлите жизнь вашему двигателю и сможете избежать дорогостоящего ремонта.

01.11.2017 18:41

Опасные тосолы и антифризы — тест

Согласно исследованиям, которые провела «Ассоциация производителей и потребителей автотоваров», более 70% антифризов и тосолов российского производства не соответсвуют ГОСТу.

По запросу ассоциации, исследовательский центр ОАО «НИИК» провел в начале прошлого зимнего сезона испытания самых популярных тосолов и антифризов российских производителей по основным параметрам:

  1. плотность;
  2. температура кипения;
  3. температура кристаллизации;
  4. содержание метанола.

По результатам исследования, большинство жидкостей провалили тест на температуру замерзания, которая наблюдалась у них при более высоком значении, чем -40°С, что является эталоном для охлаждающих жидкостей. Также ряд антифризов и тосолов не прошел проверку на температуру кипения (≥108°С по ГОСТу) и плотность вещества (1,065-1,085 по ГОСТ 28084-89), продемонстрировав гораздо худшие результаты, чем заявлено на упаковке.

Результаты теста антифризов и тосоловДля просмотра изображения в полном размере кликните по картинке

Показательным является также тот факт, что во многих продуктах были обнаружены сторонние вещества, снижающие смазывающие свойства жидкости. Особенно стоит отметить факт, что 40% тестируемых жидкостей имели в своем составе яд-метанол, запрещенный к применению в автохимии из-за его реакции с алюминиевыми деталями.

Даже если тосол или антифриз в начале эксплуатации выдерживает -35°С, то спустя непродолжительное время температура замерзания поднимется до -25°С.

Остерегайтесь подделок и некачественных товаров. Выбирайте проверенные запчасти и эксплуатационные материалы, рекомендуйте своим клиентам лучшие предложения.

Читайте также о пользе и вреде антифриза.

За рулем тесты антифризов 2019


ТОП 9 лучших антифризов за 2019 год

Картинка антифриза и авто

Самое главное в чем вы должны разбираться, это классы антифриза и его разновидности! В современном мире существует лишь 3-и класса антифриза, это G11, G12, G13 каждый из которых имеет подгруппы со знаком «+», а именно G12+, G12++, G13+.

Детальнее рассмотрим каждый класс охлаждающей жидкости более детально:

  • G11 — для данного класса характерен очень короткий срок службы из всех представленных, 2 — 3 года. Окрашивают их в зеленый, синий и желтый;
  • G12 — срок их эксплуатации около 5 лет. Карбоновая кислота главная присадка в составе именно она наделяет данный класс присущими только ему свойствами. Преимущественно окрашивают в красный;
  • G12+ — более усовершенствованный класс G12, их ключевая особенность это возможность смешивания с другим классом антифриза;
  • G13 — самый современный и экологичный класс антифриза появившийся в 2012 году в составе которых полипропиленгликоль, сроком их службы около 7 — 8 лет. Окрашивают в оранжевый или желтый;
  • G13+ — усовершенствованная разновидность предыдущего класса срок службы которого составляет около 10 лет.

Почему антифризы разного цвета? Антифриз окрашивают как минимум по 3-м причинам!

  • С целью безопасности — сама по себе она бесцветна и что бы отличить ее от обычной жидкости или бензина и т.д., их окрашивают в яркие цвета;
  • Определение утечки в системе охлаждения — согласитесь, было бы гораздо сложнее определить не только место утечки но и утечку самой жидкости, если бы она была бесцветной;
  • Класс допуска к автомобилю — в основном это относится к Европейским странам, цвет антифриза указывает на его класс, например G11 чаще всего имеет зеленый, синий и желтый, G12 имеет красный, а G13 желтый или оранжевый.

Рекомендации которые следует соблюдать

Что бы ваш автомобиль служил вам верно, без поломок и коррозии в системе охлаждения, вы должны соблюдать 5-ть главных правил при выборе любого класса антифриза!

  1. Приобретайте антифриз класса соответствующего вашему автомобилю;
  2. Не старайтесь сэкономить на покупке, экономия может повлечь серьезные затраты;
  3. Всегда уточняйте температурные допуски жидкости;
  4. Совершайте покупки в крупных сетевых магазинах, антифриз часто подделывают, а устранение поломки в системе охлаждения стоит очень дорого;
  5. Никогда не смешивайте разные классы антифриза, только в безвыходных ситуациях.

Топ 9 самых лучших антифризов

При составлении рейтинга ориентировались на:

  • количество подделок;
  • стоимости;
  • качества;
  • температуру вспышки и застывания.

Во всех современных автомобилях допуск к охлаждающей жидкости не ниже класса G12!

9 место — антифриз AGA Z40

Цвет жидкости: окрашена в красный.

Подходит для двигателей: легковых, грузовых авто и форсированных.

Срок службы: от 3 до 5 лет.

Средняя цена: 850 руб за 5 литров.

Особенности: достаточно эффективно себя показывает на форсированных двигателях, способен дополнительно защитить сальники, помпы и другие элементы. В основе этиленгликоль.

Преимущества

  • можно найти почти в любом авто магазине;
  • доступная цена;
  • приемлемая защита от коррозии.

Недостатки

  • кристализируется при температуре выше заявленной производителем;
  • много подделок.
8 место — антифриз AGA Z42

Цвет жидкости: окрашена в зеленый.

Подходит для двигателей: прекрасно показывает себя в турбированных моторах легковых и грузовых авто. Температурный диапазон от -43 градусов до +123 градусов по C°.

Срок службы: от 3 до 5 лет

Средняя цена: 850 руб за 5 литров.

Особенности: сохраняет свои свойства при резких перепадах температуры, у данного антифриза есть особенность которой нет у других, это флуоресцирующий краситель который позволяет найти даже мельчайшую утечку.

Преимущества

  • оказывает хорошую защиту двигателя от перегрева на протяжении всего срока службы;
  • хорошая защита охлаждающей системы от коррозии;
  • защищает охлаждающую систему от кавитации.

Недостатки

  • во время эксплуатации может оставлять осадок.
7 место — антифриз AGA Z65

Цвет жидкости: окрашена в желтый.

Подходит для двигателей: легковых и грузовых авто работающих на бензиновом и дизельном топливе, а так же турбированных.

Срок службы: от 4 до 6 лет.

Средняя цена: 900 руб за 5 литров.

Особенности: данный антифриз превосходно сохраняет свои свойства даже при экстремальных температурах от -66 градусов до +133 градусов по C°.

Преимущества

  • срок службы до 6 лет;
  • превосходная защита охлаждающей системы от коррозии;

Недостатки

  • во время эксплуатации может оставлять осадок.
6 место — антифриз Sintec Unlimited

Цвет жидкости: окрашена в фиолетовый

Подходит для двигателей: легковых и грузовых авто, для двигателей работающих на дизельном и бензиновом топливе.

Срок службы: от 5 до 7 лет.

Средняя цена: 800 руб за 5 литров.

Особенности: уникальный состав на основе этиленгликоля произведенного по биполярной технологии. По данным тестов, антифриз начинает замерзать при -46 градусов C°!

Преимущества

  • благодаря качественным присадкам обладает хорошими антикоррозийными свойствами;
  • температура кристализации чуть ниже чем заявлял производитель;
  • срок службы до 7 лет.

Недостатки

  • за такую цену их просто нет.
5 место — антифриз Felix Energy

Одни из немногих антифризов Российского производства который считается одним из лучших в классе g12!

Цвет жидкости: окрашена в красны либо розовый

Подходит для двигателей: легковых, грузовых, форсированных и турбированных.

Срок службы: от 5 до 7 лет.

Средняя цена: 750 руб за 5 литров.

Особенности: карбосиликатная основа, способен сохранять свои свойства при температуре от -48 градусов до +131 градуса по C°. Данный антифриз образует тонкую пленку на всех очагах ржавчины.

Преимущества

  • срок службы до 7 лет;
  • подходит для любых двигателей внутреннего сгорания;
  • одни из лучших показателей температуры закипания;
  • обладает антикоррозийными свойствами;
  • доступная цена.

Недостатки

  • обладает единственным недостатком, кристализуется при температуре чуть выше заявленной производителем.
4 место — антифриз Toyota Long Life Coolant Red Concentrate

Лучший из всех концентратов антифриза в нашем списке!

Цвет жидкости: окрашена в красный.

Подходит для двигателей: чугунных, алюминиевых и других сплавов.

Срок службы: от 2 до 5 лет.

Средняя цена: 500 руб за 1 литр.

Особенности: категорически запрещено смешивать с другими классами и марками антифриза. Отлично выдержbвает минусовую температуру до -38 градусов по C°.

Преимущества

  • хорошо защищает любые сплавы от коррозии;
  • хорошая теплоотдача;
  • не оказывает негативного влияния на резинки и патрубки.

Недостатки

3 место — антифриз Liqui Moly Motorbike Langzeit Kuhlerfrostschutz

Цвет жидкости: окрашена в красный.

Подходит для двигателей: мотобайков и авто.

Срок службы: от 3 до 5 лет.

Средняя цена: 650 руб за 1 литр.

Особенности: производитель утверждает что он совместим с многими другими видами антифриза на карбоксилатной основе, в его основе этиленгликоль.

Преимущества

  • хорошая защита от коррозии;
  • хорошая теплоотдача;
  • лучшее что можно найти для алюминиевых моторов;
  • подходит для мотоциклов.

Недостатки

2 место — антифриз Liqui Moly Langzeit Kuhlerfrostschutz

Цвет жидкости: окрашена в синий.

Подходит для двигателей: чугунных, алюминиевых и других сплавов.

Срок службы: от 2 до 4 лет.

Средняя цена: 2000 руб за 5 литров.

Особенности: отлично сохраняет свои свойства в обширном диапазоне температур от -40 до +110 градусов по C°.

Преимущества

  • хорошая теплоотдача;
  • образует внутри охлаждающей системы защитную пленку;
  • способен продлить срок службы насоса;
  • не оказывает негативного влияния на резинки и патрубки.

Недостатки

  • цена;
  • температура закипания ниже чем у большинства образцов.
1 место — антифриз TCL Power Coolant Green

Японский производитель как всегда на высоте! Это самый лучший антифриз в нашем тесте!

Цвет жидкости: окрашена в зеленый.

Подходит для двигателей: чугунных, алюминиевых и других сплавов.

Срок службы: от 4 до 6 лет.

Средняя цена: 700 руб за 2 литра.

Особенности: один из лучших Японских антифризов на карбоксилатной основе, в отличии от других в его состав добавлены специальные присадки способствующие минимизировать трения между деталями.

Преимущества

  • широкий диапазон рабочей температуры;
  • образует внутри охлаждающей системы стойкую защитную пленку;
  • во время эксплуатации абсолютно не образуется осадок;
  • очень качественное сырье.

Недостатки

  • при очень низкой температуре становится вязким.
Расскажи о статье своим друзьям: ведь она может быть полезна не только тебе

Топ 5 лучших антифризов для авто в 2019 году

На прилавках автомагазинов можно встретить огромное количество антифризов различных марок, так как эта продукция пользуется большим спросом. Чтобы подобрать для своего авто качественную охлаждающую жидкость необходимо учесть несколько параметров, в частности:

  • Популярность производителя;
  • Долговечность;
  • Отзывы экспертов, профессиональных водителей, обычных автовладельцев;
  • Соотношение цена/качество;
  • Ну и самое главное – эффективность антифриза.

Именно эти параметры мы учитывали при составлении представленного здесь рейтинга лучших антифризов для авто. Если вы не знаете, какой марки антифриз лучше заливать, то изучите наш топ и, возможно, вы сможете определиться с производителем и купить для своего автомобиля качественную охлаждающую жидкость.

Что заливаете в систему охлаждения?

ТосолАнтифриз

Не лишним будет разобраться и в классификации антифризов, которая была разработана крупнейшими европейскими производителями. Так, все охлаждающие жидкости имеют одну из следующих маркировок:

  • G11 – синяя или зеленая ОЖ, которая подходит для устаревших двигателей. Срок службы такого антифриза не большой – всего 2 года.
  • G12 – оптимальный вариант для б/у автомобилей. Срок службы составляет до 5 лет. Обычно окрашен в фиолетовый, розовый или красный цвет.
  • G12+. Антифриз с такой маркировкой рекомендуется к применению в современных автомобилях, в том числе и новых.
5. SINTEC LUX G12

Данный антифриз относится к карбоксилатным – такие ОЖ считаются самыми качественными, современными и эффективными. Жидкость производится на заводах Обнинскоргсинтез – одного из крупнейших российских производителей.

Срок службы этого антифриза составляет порядка 5 лет. Жидкость надежно защищает от коррозии, низких температур, перегрева системы охлаждения. Согласно отзывам автовладельцев и проведенным тестам, антифриз SINTEC LUX G12 не только эффективно выполняет свои основные задачи, но и очищает систему, противостоит образованию отложений, не пенится.

ОЖ имеет ярко красный цвет, но сама жидкость прозрачна. Цвет создают красители, которые добавляют для того, чтобы жидкость можно было отличить от другой или найти утечку и устранить ее.

Ринат, 34 года

Очень качественный антифриз. Если не знаете, какая охлаждающая жидкость лучше для вашего авто, то рекомендую именно SINTEC LUX G12. Использую его уже третий год. Хорошо держит температуру, отлично ведет себя в минус -35, не пенится и вообще полностью устраивает во всем. Лучшее соотношение цены и качества.

Кстати тем, кто выбирает для своего автомобиля моторное масло, предлагаем изучить рейтинг лучших в 2018 году.
4. AGA Z40

На четвертом месте антифриз, который производит российская компания «Торговый дом AGA». Продукция этого производителя широко известна и пользуется большим спросом как на родине, так и за ее пределами.

При изготовлении ОЖ используются самые современные технологии и компоненты. По своим свойствам антифриз не уступает топовым карбоксилатным охлаждающим жидкостям. AGA Z40 одинаково эффективно работает в температурном режиме от -40°С до + 123 °С. Надежно защищает систему от коррозии, перегрева и плюс ко всему смазывает металлические поверхности. Срок службы этой ОЖ составляет 5 лет, она не потеряет первоначальных свойств при пробеге до 150 тыс км.

Антифриз поставляется на прилавки только в готовом к использованию виде. Разводить ОЖ можно дистиллированной водой не более чем на 10%.

Артем, 28 лет

Второй год пользуюсь этим антифризом. Работаю на своем автомобиле в условиях крайнего севера. Жидкость качественная, прекрасно держит температуру, наверное, самая достойная в своей ценовой категории.

Охлаждающая жидкость, разработанная по новейшей технологии. Создана для защиты от коррозии, замерзания, перегрева двигателя. Рекомендуется для использования в современных силовых агрегатах, которые подвергаются высоким нагрузкам.

Антифриз создан на основе карбоксилатной технологии, благодаря чему было достигнуто максимально эффективное охлаждение двигателя. Жидкость создает на поверхности тонкую защитную пленку, которая улучшает теплообмен, уменьшает расход присадок и в целом увеличивает срок службы (до 5 лет).

ЛУКОЙЛ Red G12 окрашен в ярко красный цвет, чтобы можно было обнаружить места протечек и отличить жидкость от воды. К слову, ориентироваться на цвет при выборе не стоит, так как создается красителями лишь для перечисленных выше целей.

Охлаждающие карбоксилатным жидкости, можно смешивать с антифризами, которые произведены по аналогичной технологии. А вот смешивать ОЖ разных производителей не рекомендуется, так как это проводит к потере некоторых свойств.

Евгений, 43 года

Если не знаете, какой фирмы антифриз лучше выбрать рекомендую ЛУКОЙЛ. Одно из лучших предложений в данном ценовом сегменте. Одинаково эффективно ведет себя при экстремально низких температурах и в жару. Срок службы составляет 5 лет, поэтому залил и забыл. Пользуюсь им уже 3 года.

Эффективный антифриз, выпускаемый российской компанией «ХимАВТО». Содержит в своем составе этиленгликоль и пакет высокотехнологичных, идеально сбалансированных присадок. Обеспечивает надежную защиту двигателя от перегрева и переохлаждения, работая в широком температурном диапазоне от -40 до +112 градусов.

Уникальный состав охлаждающей жидкости Норд обеспечивает эффективную защиту всех деталей системы. При этом антифриз совместим со всеми типами металлов, а также других материалов, из которых могут быть выполнены элементы охлаждающей системы (резина, пластик, сплавы). ОЖ создает на поверхности тончайшую пленку, которая предотвращает возникновение коррозии, замедляет износ.

Жидкость имеет маркировку G12 и выпускается в канистрах объемом 1, 3, 5 и 10 литров в готовом для использования виде.

Михаил, 55 лет

Хорошая, отечественная ОЖ, ничем не уступающая зарубежным аналогам. Отлично справляется со своей функцией.

P.S. Покупайте антифриз у официальных дилеров, а не на рынке, иначе есть риск нарваться на подделку.

Итак, первое место в нашем рейтинге занимает антифриз от российского производителя «Тосол-Синтез». На сегодняшний день это единственный антифриз отечественного производства, который прошел весь цикл стендовых и лабораторных испытаний, соответствует всем международным стандартам.

ОЖ производят по кабоксилатной технологии, предусматривающей использование специальных добавок, которые противостоят коррозии, создавая тонкую защитную пленку. Кроме того, эта пленка предотвращает преждевременный износ компонентов системы охлаждения, продлевает их ресурс.

Антифриз FELIX Carbox не теряет своих свой при пробеге до 250 тысяч км. Жидкость устойчива к пенообразованию и возникновению коррозии, предотвращает образование воздушных пробок и кавитации. К особенностям следует отнести еще и то, что антифризх смазывает трущиеся детали системы, снижает трение и тем самым продлевает срок службы как отдельных элементов, так и всей системы в целом.

Георгий, 44 года

Считаю, что это лучшая марка антифриз для автомобиля и не верю, что самое качественное поставляют из Европы. Эта жидкость проявила себя только с положительных сторон – летом надежно держит рабочую температуру и не закипает, зимой не кристаллизуется даже при -40 градусов.

Итак, благодаря нашему рейтингу мы выявили лучший антифриз 2018 года и надеемся, что вы подберете для своего автомобиля самую качественную охлаждающую жидкость, которое эффективно и долгое время будет выполнять свою задачу.

Как выбрать антифриз: полезное видео в тему

 Загрузка …

Охлаждающие жидкости… горят! Экспертиза ЗР

17 августа 2016 года

Недавно по телевидению прошел сюжет про творог, который горит! Я не знаю, что туда подмешали. Но абсолютно уверен, что получить легковоспламеняющуюся охлаждающую жидкость намного проще. Достаточно при производстве вместо дорогого этиленгликоля использовать дешевый глицерин и добавить метанол для приведения вязкости продукта к приемлемому уровню. Причем этим занимаются многие производители. И никому не говорят, что температура кипения метанола — всего-навсего 64,5°C. Зачем, дескать, пугать людей?

Чем метанол опасен для моторов, мы писали неоднократно. Но никогда не заостряли внимание на том, что утечка в системе охлаждения может привести… к пожару. Попади метанол, скажем, на раскаленный коллектор — и привет: вспоминайте, как звонить с мобильника пожарным. Не верите? Мы провели небольшие испытания.

Это и есть хроматограф. Справа на мониторе можно узреть всё, что намешали изготовители. Глицерин, метанол — кто больше?Это и есть хроматограф. Справа на мониторе можно узреть всё, что намешали изготовители. Глицерин, метанол — кто больше?

Закупив в подмосковном магазинчике дюжину различных охлаждающих жидкостей, именуемых тосолами и антифризами, мы напросились в гости в химлабораторию — измерить температуру кипения и определить таким образом наличие метанола. В серьезных испытаниях для этого обычно используют хроматограф, который легко покажет на экране состав проверяемой жидкости. Но наша цель была иной. Хроматограф — это нечто заумное, а электрическая плитка и термометр сделают проверку простой и наглядной. Особенно когда на плитке одновременно стоят два стаканчика с жидкостями, одна из которых вовсю кипит, а вторая и не думает. Именно такой эксперимент мы и провели. Тем, кто захочет его повторить, очень советуем раздобыть специальные термостойкие стаканчики, которые не станут трескаться в неподходящий момент.

Такой стаканчик закипает через 8–9 минут.Такой стаканчик закипает через 8–9 минут.

Мы нагревали каждую приобретенную жидкость, фиксируя температуру начала кипения (появление первых пузырьков) и собственно температуру кипения. Полученные результаты приводим в нашей фотогалерее, а самые эффектные кадры — на видео.

Вспомнив про горящий творог, мы попытались поочередно поджечь все проверяемые жидкости зажигалкой. Результат жутковатый: пять образцов из дюжины мгновенно вспыхнули. Более того, если на плитке одновременно находились два стаканчика — один со стандартным антифризом, а другой с метанольным коктейлем, то при попытке поджечь хорошую жидкость вспыхивал… соседний состав. Пары метанола очень не любят раскаленных поверхностей и открытого огня!

Гвоздь программы: две охлаждающие жидкости на одной плитке. Справа — хорошая, слева — плохая. Пытаемся поджечь ту, что справа, однако вместо нее вспыхивают пары метанола слева.Гвоздь программы: две охлаждающие жидкости на одной плитке. Справа — хорошая, слева — плохая. Пытаемся поджечь ту, что справа, однако вместо нее вспыхивают пары метанола слева.

Результаты наших исследований относятся к конкретным образцам приобретенной нами продукции, а потому не могут служить основанием для выводов о деятельности той или иной компании. Вполне возможно, что некоторые производители повторят наш эксперимент — прежде всего те, чья продукция на поверку оказалась небезопасной.

Еще один простейший способ выявить жидкость низкого качества — использовать тест-полоски, определяющие наличие алкоголя в слюне. Во всех случаях индикаторы, искупавшись в метанольном коктейле, поменяли цвет.Еще один простейший способ выявить жидкость низкого качества — использовать тест-полоски, определяющие наличие алкоголя в слюне. Во всех случаях индикаторы, искупавшись в метанольном коктейле, поменяли цвет.

А что делать потребителю? Кипятить товар перед покупкой ему точно не разрешат. Один из способов оградить себя от неприятностей — приобретать только те товары, которые хорошо зарекомендовали себя в наших экспертизах. Избегайте торговых развалов на обочинах дорог. Хотя и в магазинах (как следует из нашего эксперимента) не всегда продают качественный товар. Причем продавцы иногда понятия не имеют, какие неприятности сулит нормальная с виду канистра. Поэтому еще один совет: пользуйтесь услугами проверенного автосервиса. Сервисмены судят о качестве приобретенной партии товара, устраивая выборочную проверку. Это куда надежнее единичных закупок.

Аntifreeze Maxim, Россия Antifreeze G11 Pilot, РоссияAntifreeze G11 Pilot, Россия Unix Antifreeze, Россия Nord Antifreeze, Россия

Температура начала кипения 70 ºСТемпература кипения 84 ºС

ГОРИТ!

Температура начала кипения 70 ºСТемпература кипения 84 ºС

ГОРИТ!

Температура начала кипения 79 ºСТемпература кипения 88 ºС

ГОРИТ!

Температура начала кипения 106 ºСТемпература кипения 109 ºС

НЕ ГОРИТ!

Sintec Euro G11, Россия ТCL Anti-Freeze, Япония Антифриз CoolStream Standard —40 ºС, РоссияАнтифриз CoolStream Standard —40 ºС, Россия Тосол —40 Felix, Россия

Температура начала кипения 99 ºСТемпература кипения 105 ºС

НЕ ГОРИТ!

Температура начала кипения 100 ºСТемпература кипения 105 ºС

НЕ ГОРИТ!

Температура начала кипения 105 ºСТемпература кипения 109 ºС

НЕ ГОРИТ!

Температура начала кипения 99 ºСТемпература кипения 105 ºС

НЕ ГОРИТ!

Тосол Айсберг, Россия Тосол Аляsка, Россия Тосол Дзержинский Maxim, РоссияТосол Дзержинский Maxim, Россия Тосол-север 40, Россия

Температура начала кипения 103 ºСТемпература кипения 106 ºС

НЕ ГОРИТ!

Температура начала кипения 91 ºСТемпература кипения 93 ºС

ГОРИТ!

Температура начала кипения 70 ºСТемпература кипения 89 ºС

ГОРИТ!

Температура начала кипения 103 ºСТемпература кипения 106 ºС

НЕ ГОРИТ!

Охлаждающие жидкости. .. горят! Экспертиза ЗРОхлаждающие жидкости… горят! Экспертиза ЗРОшибка в тексте? Выделите её мышкой! И нажмите: Ctrl + Enter

Вызывают огонь на себя: 13 вопросов об антифризах

22 июня 2016 года

Антифриз или охлаждающая жидкость?

Правильный термин — охлаждающая жидкость (ОЖ). Однако часто ее называют антифризом, поэтому в рамках этой статьи условно будем считать их синонимами. Подробнее об этом — чуть ниже.

В чем разница между антифризами и тосолами?

Тосол отличается от антифриза, как щука от рыбы. Тосол — это один из антифризов. Так раньше называли отечественную охлаждающую жидкость, а со временем ее название стало именем нарицательным. Самое распространенное заблуждение — полагать, что тосолы предназначены для отечественных машин, а антифризы — для иномарок.

Сколько типов ОЖ сегодня на авторынке? Чем они отличаются друг от друга?

Все антифризы состоят из базовых компонентов — воды и этиленгликоля, а также добавленных в них функциональных присадок. По составу присадок антифризы в настоящее время делятся на четыре основных типа: карбоксилатные (ОАТ), гибридные (hybrid), лобридные (lobrid) и традиционные (traditional).

Карбоксилатные:

  • Havoline XLC (Chevron, США)
  • Glysantin G30 (BASF, Германия)
  • G‑Energy Antifreeze SNF (Газпромнефть-СМ, Россия)
  • CoolStream Premium (Техноформ, Россия)
  • Lukoil Coolant SF (Лукойл, Россия)
  • Sintec G12+ (Обнинскоргсинтез, Россия).

Содержат ингибиторы коррозии на основе органических (карбоновых) кислот. В иностранной литературе обозначаются как сarboxylate coolants, OAT (organic acid technology), SNF (silicate nitrite free), а также G12+ (спецификации Фольксвагена). Такие ингибиторы не образуют защитного слоя по всей поверхности системы — они адсорбируются лишь в очагах возникновения коррозии. Срок службы — более пяти лет. Хорошо защищают металлы от коррозии и кавитации, обеспечивая оптимальное охлаждение двигателя.

Гибридные:

  • Glysantin G48 (BASF, Германия)
  • Mobil Antifreeze Extra (Mobil, США)
  • G‑Energy Antifreeze NF (Газпромнефть-СМ, Россия)
  • Lukoil Coolant Plus (Лукойл, Россия).

Помимо органических (карбоксилатных) ингибиторов содержат также неорганические ингибиторы: силикаты (европейская технология), нитриты (американская технология) или фосфаты (японская и корейская технологии). Обозначения: hybrid coolants, HOAT (hybrid organic acid technology), NF (nitrite free), а также G11 (спецификации Фольксвагена). Срок службы — от трех до пяти лет. Это фирменные антифризы для автомобилей марок BMW, Volvo и Mercedes-Benz.

Лобридные:

  • Glysantin G40 (BASF, Германия)
  • Cat ELC (Chevron, США)
  • G‑Energy Antifreeze HD (Газпромнефть-СМ, Россия)
  • Lukoil Coolant SOT (Лукойл, Россия)

Появились в 2008 году. Органическая основа дополнена небольшим количеством минеральных ингибиторов. Для них еще не установлено общепринятое обозначение. Европейские разработчики называют их lobrid coolants или Si-OAT (силикатно-карбоксилатные), американские — N‑OAT (нитритно-карбоксилатные), японские и корейские разработчики — Р‑OAT (фосфатно-карбоксилатные). Срок службы — как у карбоксилатных: более пяти лет.

Традиционные:

  • Cat DEAC (Chevron, США)
  • Fleetguard ES Compleat (Cummins Filtration, США)
  • Газпромнефть Тосол (Газпромнефть-СМ, Россия)
  • Тосол Felix (Тосол-Синтез, Россия).

Ингибиторами коррозии в этих антифризах служат неорганические вещества — силикаты, фосфаты, бораты, нитриты, амины, нитраты и их комбинации. Обозначаются терминами traditional coolants, conventional coolants, IAT (inorganic acid technology). Считаются морально устаревшими. Их, за редким исключением, не применяют при первой (заводской) заправке автомобилей. Это связано с тем, что неорганические ингибиторы имеют небольшой срок службы и не выдерживают температур выше 105 °C. Срок службы — примерно два года. В этом секторе — наибольшее количество подделок и суррогатов, особенно под названием Тосол.

И еще…

И еще немножко про Фольксваген. На практике для антифризов часто используют краткие обозначения, принятые в спецификации TL 774 группы Volkswagen: карбоксилатные — G12 (TL 774‑D, выведено из употребления в 2005 году) и G12+ (TL 774‑F), гибридные — G11 (TL 774‑C), лобридные — G12++ (TL 774‑G) и G13 (TL 774‑J).

К сожалению, по этикетке в большинстве случаев невозможно понять, к какому типу относится та или иная ОЖ.

Вызывают огонь на себя: 13 вопросов об антифризах

Страницы

← предыдущаяследующая →

12

Ошибка в тексте? Выделите её мышкой! И нажмите: Ctrl + Enter

Антифриз тесты за рулем 2020


Антифриз Sintec Multifreeze — универсальная ОЖ — журнал За рулем

Sintec Multifreeze — универсальный антифриз, совместимый с охлаждающими жидкостями любых типов. У него нет аналога среди российских продуктов, из зарубежных наиболее близкий ему по параметрам — антифриз Pеак Global LifeTime, выпущенный американской компанией Old World Industries.

Самый популярный вопрос, задаваемый автомобилистами про охлаждающие жидкости: «Можно ли смешивать один антифриз с другим?». Ведь часто требуется долить немного антифриза в систему охлаждения автомобиля. А владелец, как правило, понятия не имеет, какая именно жидкость залита там изначально. При этом на полках магазинов полным-полно канистр с антифризами — всевозможных названий, цветов, да еще и сделанных по разным технологиям. И при смешивании разнородных продуктов у итоговой смеси возможны нарушения эксплуатационных свойств, что может привести к снижению ресурса оборудования (например, водяного насоса).

Так проверяли коррозионное воздействие смесей антифриза Sintec Multifreeze и охлаждающих жидкостей всех типов на разные металлы. Все полученные показатели соответствуют требованиям ГОСТ.

Так проверяли коррозионное воздействие смесей антифриза Sintec Multifreeze и охлаждающих жидкостей всех типов на разные металлы. Все полученные показатели соответствуют требованиям ГОСТ.

Радикальный подход — заменить всю охлаждающую жидкость целиком. Для покупателей подержанных автомобилей это оправданное решение: неизвестно, что за жижа плещется в системе охлаждения. А если автомобиль свежий или антифриз недавно меняли? Современные жидкости рассчитаны на 200 000–250 000 км, и сливать их раньше времени нецелесообразно.

Так как же подобрать антифриз? По цвету? По названию? Трудности порождены тем, что различные изготовители применяют в производстве аж четыре основные технологии. И на прилавках соседствуют охлаждающие жидкости, называемые традиционными, карбоксилатными (ОАТ, organic acid technology), гибридными и лобридными. Цвет у них может быть любым: его определяет не технология производства, а добавляемый краситель. Иными словами, чтобы не навредить машине, нужно точно знать, антифриз какого типа залит в систему охлаждения, и цвет антифриза при этом не всегда принципиален. А вот смешивать одну зеленую жидкость с другой зеленой, не будучи уверенным, что они сделаны по одной технологии, не следует.Самый эффективный способ применения антифризов, по принципу «не навреди», сулят универсальные продукты. Так, новинка бренда #Sintec — Multifreeze — выделяется из традиционной линейки. Результаты лабораторных испытаний подтверждают, что #Sintec Multifreeze можно использовать как основную охлаждающую жидкость — это позволит в будущем не беспокоиться по поводу долива или замены охлаждающей жидкости. Кроме того, #Sintec Multifreeze можно добавлять в систему охлаждения любого автомобиля, какая бы жидкость ни использовалась в нем ранее. Разумеется, речь идет только о заведомо качественных продуктах.

Совместимость антифриза #Sintec Multifreeze с охлаждающими жидкостями иных производителей подтверждена комплексными испытаниями. В продукте используются новейшие пакеты присадок, в которых сочетаются преимущества карбоксилатых и силикатных антифризов. Это позволяет смешивать этот продукт с любыми антифризами, не опасаясь за работоспособность системы охлаждения. Проверяли не только сам #Sintec Multifreeze, но и продукты, полученные в результате его смешивания в пропорциях 90:10, 10:90 и 50:50 с карбоксилатными (G12), лобридными (G13) и традиционными (G11) антифризами разных цветов, произведенными именитыми компаниями по четырем основным технологиям. Испытания показали отсутствие химических реакций и выпадения осадка при смешивании с другими продуктами. Кроме этого, полученные смеси полностью сохраняют антикоррозионные свойства ОЖ.

www.zr.ru

Антифриз — читайте материалы с тегом Антифриз — сайт «За рулем» www.zr.ru

Рабочая температура современных двигателей и правда гораздо выше, чем у моторов, разработанных лет двадцать тому назад. И это сказывается на их надежности. Но не все так просто.

27569 просмотров

Сколько стоит обновить все эксплуатационные жидкости в Дастере и как их правильно выбрать? «За рулем» дает подробные рекомендации.

30756 просмотров

Чем больше товаров на прилавке, тем сложнее выбрать нужный. С этой точки зрения изобилие в чем-то хуже дефицита, когда покупателю нужно не думать, а хватать то, что есть. Типичный пример — покупка охлаждающей жидкости (ОЖ): обычно ее называют антифризом. Разные производители, симпатичные канистры, многообразие цветов и — главное! — различный химический состав продукта. Какую жидкость выбрать для своей машины?

НА ПРАВАХ РЕКЛАМЫ

6419 просмотров

Половина продающихся антифризов непригодны для применения — такими оказались результаты исследования двухгодичной давности. Проверив 17 свежеприобретенных образцов, эксперты выяснили, насколько изменилась ситуация.

12571 просмотр

Sintec Multifreeze — универсальный антифриз, совместимый с охлаждающими жидкостями любых типов. У него нет аналога среди российских продуктов, из зарубежных наиболее близкий ему по параметрам — антифриз Pеак Global LifeTime, выпущенный американской компанией Old World Industries.

НА ПРАВАХ РЕКЛАМЫ

15903 просмотра

Часто автомобилисты задают вопрос, чем отличается тосол от антифриза и что лучше использовать в системе охлаждения двигателя? В чем существенная разница между тосолом и антифризом и как их отличить друг от друга? Можно ли смешивать антифриз и тосол? На эти вопросы об охлаждающих жидкостях для вашего автомобиля вы сможете найти ответы в этой статье.

НА ПРАВАХ РЕКЛАМЫ

8513 просмотра

Эффективное и совершенно бесплатное решение проблемы, с которой сталкиваются многие автовладельцы, предложил читатель «За рулем».

16998 просмотров

«На здоровье не экономят», — гласит народная мудрость. И мы ответственно заявляем, что это выражение актуально не только для вашего собственного организма, но и для «здоровья» вашего авто. Ведь, согласитесь, проще один раз купить хороший шерстяной шарф, чем потом бегать по аптекам и покупать дорогие лекарства, которые в итоге обойдутся втридорога. Вот и с автомобилем та же история.

НА ПРАВАХ РЕКЛАМЫ

11389 просмотров

Читатель «За рулем» — опытный автомобилист — нашел способ, позволяющий продлить жизнь системе охлаждения автомобиля. Наверняка у вас тоже есть свои маленькие хитрости. Поделитесь ими — авторам всех опубликованных советов полагаются хорошие призы.

20591 просмотр

Подопытным стал внедорожник Mercedes-Benz ML 500 первого поколения. Результат эксперимента оказался вполне ожидаемым.

11671 просмотр

www.zr.ru

Карбоксилатные антифризы: Микстура № 12 — журнал За рулем

Последний писк антифризной моды — карбоксилатные составы, их часто обозначают индексом 12 в названии. Но можно ли верить обещаниям, которыми щедро усыпаны этикетки на бутылках с незамерзающими жидкостями? Проверку с пристрастием устроил Алексей Воробьев-Обухов.

Держать в секрете составы присадок нынче смысла нет: химические лаборатории освоили газовую хроматографию, и теперь достаточно пары капель жидкости, чтобы разложить ее на ингредиенты. Мы воспользовались помощью химиков — и получили в распоряжение формулы девяти карбоксилатных антифризов. На самом деле главная тайна — технология приготовления раствора, а ее так просто не вычислишь!

СОДЕРЖАНИЕ НЕОРГАНИЧЕСКИХ ВЕЩЕСТВ В АНТИФРИЗАХ

Нам, журналистам, впору было схватиться за голову от обилия органических кислот с никогда ранее не встречавшимися названиями в отчете из химической лаборатории. Но специалисты успокоили: все они хороши в том смысле, что имеют право (и должны) присутствовать в карбоксилатных антифризах. Эти-то кислоты как раз и защищают систему охлаждения от агрессивного этиленгликоля. А вот силикатов, боратов, фосфатов и нитритов в таких составах быть категорически не должно, ну разве что в незначительных количествах — в пределах пары миллиграммов на литр (это оговорено требованиями автопроизводителей да и прямо указано практически на всех этикетках).

Итак, стало ясно, по какому принципу нам расставить участников экспертизы. Те, анализы которых показали присутствие боратов, автоматически опустились вниз. Остальные же выстроились согласно прейскуранту. В конце концов, на дворе кризис, надо экономить.

Антифризы

Но только с умом: если ваш автомобиль новый, на гарантии, выбирайте антифриз с допуском именно вашего автопроизводителя и именно для конкретной модели. Даже если на этикетках лихо обещают пригодность для почти бесконечного списка машин, не стоит относиться к этому серьезно. Во-первых, такого не может быть в принципе, а во-вторых, зачем вам, случись что, оплачивать ремонт из своего кармана? Выбор для обладателей авто со стажем шире, но и здесь есть смысл взять антифриз, проверенный не только ЗР (помните, мы писали выше о важности технологии приготовления раствора?), но и прошедший обкатку в автопарке именитых автопроизводителей! И, наконец, вряд ли стоит рисковать и заливать жидкость непонятного состава, лишь заявленную как G12, но содержащую агрессивные по отношению к алюминию бораты, которых, повторим, там быть не должно — как по смыслу термина «карбоксилатный», так и согласно прямым заверениям на этикетках.

Антифризы

Разве что ненадолго, в экстренной ситуации…

НАША СПРАВКА: КАК ДОШЛИ ДО ЦИФРЫ ТАКОЙ?

Автомобилистам давным-давно известно: если воду разбавить этиленгликолем, то зимой ее можно будет не сливать из охлаждающей системы. Точность пропорций не столь уж важна: в любом случае такая смесь на сильном морозе не замерзает, не угрожает разрывом блока цилиндров и радиатора, потому что превращается в вязкую шугу с мелкими кристалликами льда. Для древних автомобилей с чугунными моторами и латунными радиаторами такой антифриз был к тому же безопасен в отношении коррозии.

Все стало хуже с началом широкого внедрения алюминиевых сплавов: горячий раствор этиленгликоля быстро разъедал радиаторы, интенсивнее всего — их тонкостенные трубки. Тогда в состав антифризов стали вводить добавки на основе неорганических солей — они образовывали на металлических поверхностях слой, устойчивый к этиленгликолю. Так работали и всем известный «Тосол», и ряд более поздних, так называемых гибридных антифризов, один из которых был обозначен фирмой «Фольксваген» как G11. Однако срок службы пакета неорганических присадок не превышал пары лет.

Продлить живучесть антифризов помогла органическая химия: вместо минералов в пакеты присадок стали включать кислоты со сложными названиями. Реагируя с оксидом алюминия, они создают на поверхности деталей защитную пленку, исправно работающую лет пять, а то и дольше! И вновь на «Фольксвагене» придумали ставший нарицательным индекс — G12. Хотя правильнее называть такого рода антифризы карбоксилатными.

Спросим чит

www.zr.ru

тесты, рейтинги и советы по выбору лучшей марки

Антифриз – незаменимая жидкость для автомобилистов зимой, так как она не замерзает при низкой температуре. Но его нужно правильно выбрать, руководствуясь рядом критериев. Эта жидкость нужна обеспечения нормальной работы систем охлаждения, предотвращения повреждения деталей. В составе этиленгликоль, дистиллированная вода (почти 80% всего объема) и присадки.

Классификация по цвету

Различают несколько видов жидкости: зеленый, красный и синий, желтый и фиолетовый. Последние два очень редко применяют, а синий (тосол) выпускают только в РФ. Присадки – важная часть в составе тосола. Им придают определенный цвет, по которому мы можем классифицировать жидкость. Какой марки антифриз лучше заливать, зависит от заявленных свойств, которые можно понять по цвету.

На заметку!

Тосол держит температуру от -30 до -40 оС. С его помощью происходит защита патрубков и трубок. Срок службы очень маленький (всего 2-3 года, а потом его свойство пропадает).

Еще один из недостатков – при температуре 110-115 оС он закипает. Его не желательно заливать в иномарки, однако отечественные автомобили с ним справляются. Рекомендовано менять его полностью раз в три года.

Характеристики антифризов по цвету:

  1. Зеленый (G11) борется с причинами возникновения коррозии, обволакивает внутренние стенки охладительного тракта. Однако, через некоторое время появляется налет в системе охлаждения, который забивает мелкие каналы. Замена происходит через 2-3 года. Отвод тепла снижен из-за пленки на стенках. Отзывы на антифриз Полярник подтверждают заявленные свойства жидкостей.
  2. У красного (G12) отвод тепла значительно лучше, появление налета из-за осыпания отсутствует, локализует очаги ржавчины. Однако, профилактики системы охлаждения нет. Рейтинг красных антифризов G12 показал, что у них отличная защита радиаторов из алюминия намного ниже, чем у медных или латунных.
  3. Лобридный или фиолетовый (G13) в составе вместо этиленгликоля имеет пропиленгликоль, а значит менее ядовитый и имеет угрозу для окружающей среды.
  4. Желтый (G13) – универсальные марки качественных антифризов (фото), подходят любой системе. Имеют большой срок службы.

Если в автомобиле больше меди и латуни, то заливают красный, если алюминия и его сплавов, то – зеленый. Рейтинг антифризов составляют с учетом этого критерия.

Читайте также: Пять лучших термокружек для вашего авто

Как выбрать антифриз для автомобиля

При выборе нужного антифриза не стоит опираться только на его цвет. Выбирают те, которые указаны автопроизводителем. Если вы решили проигнорировать советы производителя, и воспользоваться аналогом, то ознакомьтесь с с этикеткой. Обратите внимание на:

  • название;
  • дату изготовления;
  • срок годности;
  • где его применять.

Тест антифризов G12 проводят ежегодно. Выбирать можно и по марке. На антифриз Ниагара G12 отзывов много, можно взвесить все плюсы и минусы. Но специфической классификации там нет – у каждой страны свое буквенное обозначение. Смешивать жидкости G11 и G12 запрещено. Многие тосолы проходили тест антифризов «За рулем 2018».

Рейтинг марок

Существует много хладагентов, которые отличаются классификацией, цветом и качеством. Лучший антифриз для автомобиля выбрать трудно, большинство из них хорошо выполняют свои функции, однако есть и те, которые наоборот могут навредить автомобилю. Такие жидкости самые дешевые, и их стоит исключить из выбора.

На заметку!

Сложно выделить лучшего производителя, и какую фирму выбрать, так как у многих есть и удачные варианты, и малоэффективные.

Многие утверждают, что зеленый антифриз лучше. Но при этом не называют конкретную марку. Поэтому такие утверждения голословны, не подтверждены фактами. В тоже время по фактам продаж, марки антифриза красного цвета самые востребованные у автомобилистов. Это объясняется хорошим качеством при относительно невысокой цене. Рейтинг марок поможет подобрать подходящий тосол. Отзывы на красный антифриз Арктика G12 положительные, но это не значит, что он лучший, нужно изучить и другие составы.

Рейтинг универсальных марок по мнению потребителей:

  1. Антифриз Chemipro, по отзывам он отлично справляется со своими задачами.
  2. Кастрол. Антифриз от производителя в России. Зарекомендовал себя как качественный и недорогой. Допущен к использованию на многих марках автомобилей.
  3. COOLSTREAM Premium. Сравнение антифризов показало его многочисленные преимущества.

Какой антифриз лучше купить решает автовладелец. При затруднениях лучше посоветоваться в автосервисе. Там дадут рекомендации отталкиваясь от требований конкретного автомобиля.

Гибридные

Такой антифриз имеет более прочный набор присадок. Название появилось из-за комбинирования карбоксилатов (соли карбоновых кислот) и солей неорганических кислот. Воздействие на причины появления коррозии выше, чем у обычных присадок, но неорганические составляющие очень быстро истощаются, из-за чего срок эксплуатации уменьшается. Рейтинг производителей антифризов поможет подобрать нужную марку.

Читайте также: Советы для автомобилистов: как подниматься и спускаться с возвышенности зимой

Felix Prolonger G11

Попал в ТОП антифризов для автомобиля, так как подходит для легковых и грузовых автомобилей. Обеспечивает быстрый прогрев мотора, уменьшает трату горючего, а срок действия насоса и радиатора повышается, сравнительно с другими силикатными жидкостями. Преимущества:

  • высокая защита двигателя;
  • эффективная забота о металлических элементах от коррозии (в том числе и алюминиевых), переохлаждения и перегрева;
  • хорошие смазывающие и очищающие свойства;
  • отсутствие образования накипи;
  • низкая цена;
  • надежность.

Главный недостаток – токсичность, и защитные свойства хладагента уступают качествам карбоксилатных и лобридных марок. Самый лучший антифриз по мнению владельцев недорогих машин 1995-2000 годов выпуска.

CoolStream Hybrid Extra

Разработан на основе этиленгликоля и силикатно-карбоксилатных ингибиторов. Применяют почти во всех двигателях. Плюсы: защита от накипи, ржавчины и температурных влияний. У жидкости высокая эффективность, низкая цена, сочетаемость с прочими жидкостями.

На заметку!

Отсутствие фосфорных соединений снижает образование наслоений в гильзах, трубопроводах и радиаторах.

Состав, не содержащий фосфатных соединений, обеспечивает высокую стабильность даже с водой высокой степени жёсткости. Минусы пока отсутствуют. В сравнении с антифризом Роснефть немного выигрывает.

Sintec Ultra G11

Отличный показатель стойкости к перепадам температур. Субстанция не кристаллизуется на морозе, что гарантирует целостность силовой установки. Обладает низкой коррозионой активностью и малой ценой. Смесь насыщена противоизносными веществами, стимулирующими покрытие.

Из минусов:

  • малое число щелочи;
  • короткий срок службы.

Еще один недостаток – осыпание защитной пленки спустя 2-3 года эксплуатации, что негативно сказывается на работоспособности силовой установки.

Карбоксилатные

Это хладагенты с присадками-ингибиторами на карбоксилатной основе. Хорошие замедлители коррозионных процессов. Они точечно воздействуют на пораженный участок, покрывая защитной пленкой место химического воздействия. Отсутствуют активные примеси, которые могут способствовать жестким отложениям.

Liqui Moly Langzeit Kuhlerfrostschutz GTL 12 Plus

Разработан специально для мощных алюминиевых двигателей. В составе отсутствуют силикаты и фосфаты, но антифриз запрещено смешивать с жидкостями, которые не имеют силикаты. Есть возможность работать длительный период без замены. К минусам относят высокую цену (но из этого выплывает качество) и, в некоторых случаях, неприемлемый объем тары, ведь выпускают только 5-ти литровые бутыли.

Читайте также: Как легко разжечь костер с использованием автомобиля

Sintec LUX G12

Используют в основном в алюминиевых двигателях с большой нагрузкой. С помощью этого антифриза система защищается от замерзания, перегрева, окисления и образования отходов в радиаторе, насосе и моторе. Достоинства:

  • максимальное качество;
  • минимальная активность к образованию ржавчины и отсутствие отложений.

У этого тосола малая терпкость, высокая температура сгорания.

Felix Carbox G12

Работает при температуре от -45 до +50 оС. Основные плюсы: высокая температура закипания, защита от различных типов коррозии, включая кавитационную, многофункциональная антикоррозионная присадка, малая цена. Отсутствие в составе:

  • аминов;
  • фосфатов;
  • боратов;
  • силикатов.

Минусы: может кристаллизоваться при температуре выше 50 градусов, более короткий интервал замены по сравнению с другими жидкостями.

Лобридные

Такой антифриз занял промежуточное положение между гибридным и карбоксилатным типом. Ряд присадок состоит в основном из карбоксилатов с добавлением силикатов или фосфатов. Это намного совершенствует технические характеристики по сравнению с гибридным. Принципиальное отличие антифриза от всех существующих – неограниченный срок службы, при условии заливки в новый двигатель.

Sintec UNLIMITED G12++

С помощью силикатов появляется антикоррозийная пленка, а карбоновые компоненты действуют точечно в слабых и легко уязвимых местах. Достоинства: можно соединять с хладагентами других видов, не содержит вредных составляющих.

На заметку!

У этого тосола высокая температура кипения, долгий срок использования, устойчивость к образованию осадков, механическому износу.

Недостатки: солидный ценник и большое количество суррогата.

Ravenol LTC – Protect G12++ Premix -40

Не содержит бораты, нитриты, фосфаты. Обеспечивает долговременную, почти 5-ти летнюю, антикоррозионную защиту алюминиевого двигателя в холодную и жаркую пору. Угрозы для окружающей среды не представляет. В составе имеет лиловый флуоресцентный краситель, с помощью которого можно обнаружить возможные места утечки антифриза используя дефектоскоп. Минус – очень высокая цена.

Тосолы G13

Современные безвредные антифризы на основе полипропиленгликоля. Жидкость более экологична, поэтому ее стоимость намного выше, чем у аналогов. У G13 – неограниченный срок эксплуатации, если он залит в новую машину. G13 можно смешивать с видами типа G12, G12+ и G12++.

CoolStream G13

Преимущества – использование качественных присадок и доступная цена. Защищает все виды металла, высокие очищающие, антипенные свойства. Недостаток – требует мер предосторожности.

Motul Inugel G13

Обеспечивает защиту от перегрева или замерзания двигателя, дополнительную профилактику от отложений, коррозии деталей мотора, накипи. Применяют для остывания систем транспорта. Плюсы: отсутствие нитритов, фосфатов, аминов, боратов.

На заметку!

Тосол обеспечивает предотвращение кавитации.

В составе горькие добавки, чтобы избежать случайного отравления. Жидкость полностью безопасна для резиновых, пластиковых деталей, и прокладок. Минус – очень высокая цена.

Выбор антифриза происходит в зависимости от того, насколько важным является транспортное средство. Можно купить дорогой вариант, и он послужит лучшим защитным средством, или же поскупиться и приобрести дешевый продукт или даже подделку. При этом срок жизни автомобиля укоротится в геометрической прогрессии. И все же, выбрав нужный хладагент, нужно понимать, какие типы можно смешивать, а какие вовсе не стоит.

blognovichok.ru

Тест 35 антифризов

Антифриз не менее важен для мотора, чем масло. Он сохраняет идеальный для работы двигателя температурный режим, защищая от чрезмерного изнашивания деталей, коксования масла и отложения нагара. А вот некачественный антифриз может не просто привести к «закипанию» масла, а натурально «съесть» двигатель, не хуже чем червяк вкусное яблоко – коррозия способна разрушить его так, что останется только ставить новый.

Покупаем «кота в мешке»

И тут возникает вполне законный вопрос – как отличить кондиционную охлаждающую жидкость от некондиционной? Заранее отметаем варианты приобретения антифриза в сомнительных местах (у обочины на МКАДе, например). Но опыт показывает, что и в магазине покупка негодного продукта – не редкость. Как же из двух десятков канистр выбрать ту, которая будет служить двигателю верой и правдой? Сертификация проблему не решает – сертификат есть у ВСЕХ, а именно – сертификат соответствия заявленных технических условий производства охлаждающей жидкости реальным. Этот сертификат лишь косвенно соотносится с ГОСТом. Состав ВСЕХ антифризов (судя по этикетке) – одинаков: этиленгликоль, вода, пакет присадок. Что за присадки – покупателю никто не скажет; это «секрет фирмы». А ведь их там может не быть вообще! Или же это могут быть присадки, вредные для какого-то из металлов двигателя, обладающие агрессивными коррозионными свойствами. Таким образом, потребитель получает «кота в мешке», и покупка антифриза становится лотереей.

Выход есть

Выход из этой плачевной ситуации все же есть. Это независимые тесты согласно ГОСТу в имеющих государственную сертификацию лабораториях. Результаты этих тестов, публикуемые в прессе, должны быть своего рода «путеводной ниточкой» при покупке охлаждающих жидкостей (и не только!). Каждый потребитель вправе позвонить в редакцию и ознакомиться с протоколами тестов, чтобы быть уверенным, что полученные результаты – не «заказные», не очередное средство в конкурентной борьбе и одурачивании покупателей.

Представленный здесь тест антифризов проводился двумя лабораториями, имеющими аккредитацию Госстандарта: это Химико-Аналитическая лаборатория РГУ Нефти и газа им. Губкина и сертификационный центр «Констанд». Инициатор проведения теста – Московская ассоциация предприятий технического обслуживания и ремонта автомототранспортных средств (МАПТО). Главным направлением деятельности МАПТО является оказание содействия в создании в г. Москве комплексной системы технического обслуживания и ремонта автомототранспортных средств, предназначенной для дальнейшего повышения качества обслуживания и полного удовлетворения потребности государственных, общественных, коммерческих организаций и граждан в качественном обслуживании и ремонте автотранспорта.

По пяти предусмотренным ГОСТом параметрам оценено 35 охлаждающих жидкостей, которые были куплены в магазинах Москвы.

Что именно мы тестировали

Тест проводился по пяти параметрам ГОСТа 28084-89 (из десяти).

Плотность при 20°С определяет соотношение гликоля и воды. По ГОСТу плотность должна быть 1,065—1,085 для охлаждающих жидкостей –40 и 1,085—1,100 для охлаждающих жидкостей –65. Если воды будет больше нормы, двигатель попросту замерзнет, если же переборщили с этиленгликолем, ухудшится теплоотдача, двигатель будет перегреваться. Определяется плотность ареометром при 20 С.

Температура начала кристаллизации тоже зависит от соотношения вода/гликоль. Началом кристаллизации считается появление облачка мути, видимое невооруженным взглядом. До замерзания антифриза, конечно, еще далеко, но в ГОСТе требования четкие: —40°С для охлаждающей жидкости –40 и —65°С – для охлаждающей жидкости –65.

Водородный показатель (pH) согласно ГОСТу должен быть в пределах 7,5—11,0. Он позволяет судить об агрессивности препарата к металлам и определяется пакетом присадок, в отличие от двух предыдущих параметров, которые зависели от соотношения гликоль/вода. Измеряется pН-метром.

Щелочность. Со временем этиленгликоль может окисляться до уксусной или щавелевой кислоты, и поэтому для нейтрализации необходим буфер (иначе препарат может приобрести сильнокислотные свойства и будет разъедать металлы двигателя). ГОСТом декларирован показатель 10. Щелочность определяют потенциометрическим титрованием соляной кислотой.

Фракционные данные – это температура начала перегонки и массовая доля жидкости, перегоняемой до температуры 150°С (в процентах). Это испытание в ГОСТе не является обязательным и производится по требованию. Начало перегонки наступает при 100°С и до достижения 150°С должно отогнаться не более 5/40/50% жидкости (для концентрата, —65 и –40 антифризов). По сути, тест определяет количество воды в составе (напомним, что в концентрате массовая доля воды не должна превышать 5%, в охлаждающей жидкости –65 рекомендуется соотношение 35 на 65, в охлаждающей жидкости –40 – 56 на 44).

Тестировались не все параметры

Тест охватил очень широкий спектр антифризов, но он не претендует на полноту, поскольку по ряду важнейших параметров образцы не оценивались (акцент был сделан на соотношение гликоль/вода и общие параметры состава, такие, как pH и щелочность). Логически этот тест является первым этапом более обширного исследования, которое охватывало бы остальные параметры ГОСТа: внешний вид, коррозионное воздействие на металлы, вспениваемость, набухание резин и устойчивость в жесткой воде (только для концентрата).

Проблемы обычно возникают с коррозионным воздействием на металлы – нередко оно в десятки раз превышает требования ГОСТа. Этот параметр определяется пакетом присадок – дело в том, что смесь этиленгликоля и воды очень агрессивна, и специальные присадки призваны защитить металлы от коррозии. Хотя бывает, что эти же самые вещества негативно влияют на работу двигателя. Так, имеющиеся в некоторых рецептурах нитриты, нитраты и бораты призваны защищать черные металлы, бензоат натрия – цветные металлы, силикаты – алюминий. Однако в требованиях спецификации Ford хладагент для легковых автомобилей и легких грузовиков указано, что в составе антифризов не должны присутствовать силикаты, фосфаты, бораты и хлориды. Такие же требования предъявляют Texaco к антифризам (например, havoline), в составе которых не допускается присутствие нитратов, нитритов, боратов, фосфатов и силикатов. Некоторые зарубежные производители (например Shell с антифризом Shell SF) предлагают использовать соли карбоновых кислот – карбоксилаты; ожидается, что они смогут продлить срок службы двигателя.

Важным показателем является вспениваемость. Пены должно быть как можно меньше, и исчезать она обязана за три секунды. Если пены много, то система закупоривается и двигатель перегревается. Причина избытка пены – некачественный этиленгликоль.

Время собирать камни

Даже при неполном тесте, не включавшем проверку на коррозионное воздействие на металлы, не соответствующие ГОСТу, оказались 11 продуктов из 35 – то есть практически треть. Большинство имеют слишком высокую температуру начала кристаллизации – то есть вместо положенных —40°С показывают, скажем, —30°С. Для Москвы и теплых регионов это не столь существенно, как другой показатель – щелочность. Отсутствие или недостаточная щелочность говорят о том, что со временем препарат будет становиться все более и более агрессивным по отношению к металлам. Некоторые средства не соответствуют даже по рН!

Европейского качества недостаточно!

Результаты теста комментирует Ирина Николаевна Белокурова, работавшая в знаменитом отделе ТОС (Технология Органического Синтеза) ГСНИИОХТ, где появился первый ТОС-ОЛ (-ол – окончание для спирта), принимавшая непосредственное участие в разработке ГОСТа 28084-89.

Действительно, после теста на коррозионное воздействие на металлы «выживших» охлаждающих жидкостей станет значительно меньше. Да и допуск протестированных параметров в ГОСТе 28084-89 значительно «мягче», чем следовало бы. Например, плотность. Разрабатывая ГОСТ, мы ориентировались и на страны СЭВ (Восточной Европы), где климат значительно теплее нашего. Минимально допустимая плотность в ГОСТе – 1,065 рассчитана на —30°С. Для Европы с ее мягкими зимами этого достаточно, а вот для России – маловато. Оптимальная плотность – где-то по серединке: 1,075—1,080.

То же самое можно сказать и про рН: 11,0 слишком много, ведь алюминий не выдерживает уже при 9,5. А вот минимальный показатель щелочности можно, пожалуй, понизить с 10 до 9.

Удобрения в антифризе

Что же добавляют в антифризы недобросовестные производители? Во-первых, это некачественные присадки. Были случаи, когда в охлаждающую жидкость прямо из мешка добавлялиЕ удобрения. Нитраты все-таки! О том, какими последствиями такой экспромт обойдется двигателю, они не озаботились.

Но, оказывается, схитрить можно не только с «секретным» пакетом присадок, но и с основой антифриза – моноэтиленгликолем. В норме он должен быть высшего или первого сорта, что определяется содержанием основного вещества и примесей. «Сэкономить» можно, использовав в качестве основы более дешевый «грязный» этиленгликоль, содержащий большое количество примесей самого разного характера и происхождения. Некоторые производители вместо моноэтиленгликоля используют диэтиленгликоль (ДЭГ) – да, температура замерзания получается та же, двигатель охлаждается исправно. Но дело в том, что ДЭГ – менее стабильное соединение, он быстрее окисляется и более агрессивен по отношению к металлам.

Иногда в антифризе можно обнаружить до 20—30% эфиров. Опять же, нужные температуры замерзания они обеспечивают, но являются еще более токсичными, чем этиленгликоль.

В тесте приняли участие:

НАИМЕНОВАНИЕПРОИЗВОДИТЕЛЬСООТВЕТСТВИЕ ТРЕБОВАНИЯМ ГОСТ
Тосол 40 (жидкость охлаждающая)ООО «Агат-авто»щёлочность
Тосол А40-МЗАО «ПСК Цитадель»
Антифриз «Сапфир»ООО «Эмрос»
Антифриз концентрат «Экстра»Hi-GearТемпература начала кристализации
Антифриз HoltsООО «РусХолтс»
Тосол «Сапфир» -40ООО «Эмрос»Температура начала кристализации, водородный показатель рН, фракционные данные
Тосол Север -40ООО «Гелена ХимАвто»Температура начала кристализации, фракционные данные
Тосол (жидкость охлаждающая) Север-40ООО «Гелена ХимАвто»
Тосол (жидкость охлаждающая) Север-40ООО «Гелена ХимАвто»
Тосол 40 (жидкость охлаждающая)ООО «Исньский»
Антифриз «ТехКом -36»ОАО «Кусковский хим. завод»Температура начала кристализации, фракционные данные
Антифриз «Nord»ООО «Гелена ХимАвто»
Антифриз (жидкость охлаждающая) Sintec ОЖ-40ООО «Обнинскоргсинтез»
Тосол (жидкость охлаждающая) ТС ОЖ-40ООО «Тосол-Синтез»
Тосол А-40М «Техком»ОАО «Кусковский хим. завод»
Тосол (жидкость охлаждающая) ОЖ-40ООО «Полар-Авто»Фракционный состав
Антифриз 40 (жидкость охлаждающая)ООО «Полар-Авто»
Антифриз «Nordix»ЗАО «Дизельнефтепродукт»
Тосол (жидкость охлаждающая) ОЖ-40ООО «Волга-Ойл»Фракционный состав
Тосол «Арктон»ООО ПКФ «Аметист+»
Антифриз «Арктон»ООО ПКФ «Аметист+»
Тосол «Cool Keeper»ООО «ТЭКС»
Антифриз «BBF»ЗАО «Химпромпровет»
Тосол «Spectrol»ПГ «Спектр Авто»Температура начала кристализации, водородный показатель рН
Антифриз «За рулём»ЗАО «ТД Товары за рулём»
Тосол «Luxoil»ООО «Пушкинский завод»
Тосол 40 (жидкость охлаждающая)ООО «Агат-авто»щёлочность
Тосол «Dixis»ООО «Гелиос-Инт»
Антифриз «Dixis»ООО «Гелиос-Инт»
Супер тосол «Consol»ООО «Виал Ойл»
Антифриз «Consol»ООО «Виал Ойл»
Антифриз «ТехКом -63»ОАО «Кусковский хим. завод»Температура начала кристализации
Тосол «Helis Best»ООО «ТЭКС»
Тосол «Росполимер-А40М»ООО «Химинвест»Температура начала кристализации
ЛЕГЕНДА:Соответсвует требованиям ГОСТНе соответсвует требованиям ГОСТ

Тесты журнала ПОТРЕБИТЕЛЬ АвтоДела. www.autodela.ru

autolubricants.info

современные моторы обречены на перегрев — журнал За рулем

Рабочая температура современных двигателей и правда гораздо выше, чем у моторов, разработанных лет двадцать тому назад. И это сказывается на их надежности.
Но не все так просто.

В девяностые и нулевые на наших дорогах часто можно было наблюдать автомобили с перегревшимся двигателем. Их было видно издалека по клубам пара из-под капота. В основном это были немолодые отечественные машины, а причина перегрева часто крылась в некачественных комплектующих: некондиционные термостаты, насосы охлаждающей жидкости, дефектные шланги и радиаторы. Двигатели теряли охлаждающую жидкость и закипали.

Современные автомобили куда надежнее, но и сложнее конструктивно. Перегрев в привычном понимании этого слова случается реже, но настолько внезапно, что водитель не успевает среагировать и что-либо предпринять, а последствия перегрева для мотора зачастую фатальны. Да и в целом нынешние моторы куда «горячее» предшественников.

Какая температура двигателя нормальная?

Материалы по теме

Эффективность работы двигателя внутреннего сгорания повышается с ростом температуры. Казалось бы, зачем тогда нужна система охлаждения? Проблема в том, что современные конструкционные материалы, как и смазывающие вещества, не способны работать при слишком высоких температурах. Однако мотористы стараются сделать двигатели максимально эффективными. Если еще пару десятилетий назад считалась нормальной рабочая температура в 80°С, то теперь показатель — около 105°С. Мотор с такими характеристиками экономичнее, и у него ниже токсичность отработавших газов (за исключением окислов азота).

А что же нам показывает штатный указатель температуры? Начнем с того, что у части машин его просто нет. Таковы некоторые комплектации Kia Rio прошлого поколения, Nissan Note, Honda Jazz, Lada Granta первых годов выпуска и другие. Присутствует лишь индикатор перегрева, срабатывающий, как правило, слишком поздно. У других автомобилей очень условные индикаторы в виде «кирпичиков», управляемые бортовым компьютером. Информативность у таких указателей низкая. Например, у Лады Ларгус, будь температура антифриза 80°C или 105°C, на дисплее четыре «кирпичика».

Почему мотор греется?

www.zr.ru

Как выбрать антифриз — полезные советы — журнал За рулем

Выбирая охлаждающую жидкость для двигателя своего автомобиля, водители обычно ориентируются на рекомендации пусть и бывалых, но все же не очень компетентных коллег — или не очень грамотных в своем деле продавцов.
Не вникая в суть вопроса, они выбирают антифриз «по цвету» или «по классификации G», или самый дешевый и, чаще всего, промахиваются с выбором.

Откуда родились мифы об этих показателях качества антифриза? И почему они не соответствуют действительности?

Все цвета радуги

Первый, самый частый (и, пожалуй, самый категоричный) совет, который дают доброжелатели при выборе антифриза, — «Смотри на цвет и доливай такой же»! Есть даже негласная народная классификация: красный антифриз — это высший сорт, а срок его службы составляет до 5 лет; зеленый — среднее качество и около 3 лет службы; синий — антифриз низкого качества, такой заливают на год, от силы два.

Примитивная логика этих рассуждений может привести к серьезным проблемам с двигателем, вплоть до выхода его из строя. Потому что цвет антифриза не имеет ни малейшего отношения к его техническим показателям. И смешивая коктейль из двух жидкостей неизвестного состава, можно получить на выходе все что угодно — вплоть до коктейля Молотова.

Антифризы одного цвета могут оказаться принципиально несмешиваемыми между собой. Например, охлаждающие жидкости Volkswagen Group G11 и Kia/Hyundai A-110 имеют зеленый цвет. Но смешать их — это значит грубо нарушить требования автопроизводителей!

Изначально антифриз, как и любая другая сервисная жидкость, бесцветен. Производитель, в меру своей предприимчивости и фантазии, а также идя навстречу пожеланиям маркетологов, может добавить в основной состав любые красители, включая флуоресцентные. Это удобно (легче обнаруживаются места протечек), эффектно и, конечно, позволяет расширить линейку товаров. Если речь идет о крупном производстве, то чаще всего цвет антифриза — это выбор производителя сервисной жидкости, согласованный с производителем авто.

Так, например, российская компания «Техноформ» в партнерстве с компанией Arteco (Бельгия), одним из международных лидеров в производстве присадок и охлаждающих жидкостей, производит один и тот же антифриз — Coolstream Premium — сразу для нескольких автозаводов в разных цветах. И с точки зрения маркетинга, и с точки зрения учета удобнее, чтобы автозаводы получали антифриз разного цвета: для одного зарубежного автопроизводителя — оранжевый, для других — желтый, розовый, синий и т.д. При этом все это — один и тот же премиальный антифриз Coolstream, имеющий один и тот же состав и являющийся полным ребрендом (аналогом) лучших зарубежных антифризов (Havoline).

Точка G 

Такой метод подбора антифриза, наверно, придумали те, кто верит, что все лучшее, качественное и передовое — на Западе. И в этом тоже есть доля правды… Но и доля мифотворчества присутствует. Совет покупать для иномарок антифризы, на этикетке которых указана маркировка G11, G12, G12+ («и чем больше цифра, и чем больше плюсов — тем лучше!»), стал следствием высокой популярности антифризов, которые выпускает автоконцерн Volkswagen AG: VW coolant G 11 и VW coolant G 12. Действительно, к качеству здесь не придерешься. Жаль, что гаражные эксперты ленятся интересоваться, что же стоит за этой маркировкой.

На самом деле маркировку с литерой G немецкий автоконцерн разработал исключительно для внутреннего применения — для заправки автомобилей, которые выпускает Volkswagen (а это — Audi, Seat, Skoda, Bentley, Bugatti, Lamborghini, Porsche, Ducati, Volkswagen).

  • Маркировка G11 ставится на гибридных антифризах, которые соответствуют спецификации VW TL 774-C (не используется с 2011 года).
  • Маркировка G12 означает карбоксилатные антифризы, соответствующие спецификации VW TL 774-D (не используется с 2004 года).
  • Маркировка G12+ пришл

www.zr.ru

Опасные тосолы и антифризы — тест

Согласно исследованиям, которые провела «Ассоциация производителей и потребителей автотоваров», более 70% антифризов и тосолов российского производства не соответсвуют ГОСТу.

По запросу ассоциации, исследовательский центр ОАО «НИИК» провел в начале прошлого зимнего сезона испытания самых популярных тосолов и антифризов российских производителей по основным параметрам:

  1. плотность;
  2. температура кипения;
  3. температура кристаллизации;
  4. содержание метанола.

По результатам исследования, большинство жидкостей провалили тест на температуру замерзания, которая наблюдалась у них при более высоком значении, чем -40°С, что является эталоном для охлаждающих жидкостей. Также ряд антифризов и тосолов не прошел проверку на температуру кипения (≥108°С по ГОСТу) и плотность вещества (1,065-1,085 по ГОСТ 28084-89), продемонстрировав гораздо худшие результаты, чем заявлено на упаковке.

Результаты теста антифризов и тосолов
Для просмотра изображения в полном размере кликните по картинке

Показательным является также тот факт, что во многих продуктах были обнаружены сторонние вещества, снижающие смазывающие свойства жидкости. Особенно стоит отметить факт, что 40% тестируемых жидкостей имели в своем составе яд-метанол, запрещенный к применению в автохимии из-за его реакции с алюминиевыми деталями.

Даже если тосол или антифриз в начале эксплуатации выдерживает -35°С, то спустя непродолжительное время температура замерзания поднимется до -25°С.

Остерегайтесь подделок и некачественных товаров. Выбирайте проверенные запчасти и эксплуатационные материалы, рекомендуйте своим клиентам лучшие предложения.

Читайте также о пользе и вреде антифриза.

info-parts.ru

Тест антифризов За Рулем 2017

Чугунным моторам и латунным радиаторам дре­мучих автомобилей подобная смесь не грозила кор­розией. А вот в более современных двигателях горя­чий антифриз начал выгрызать куски металла. Поэто­му химики создали новую охлаждающую жидкость. Первые три буквы ее названия взяли с таблички над дверью отдела: «Технология органического синтеза». Окончание «ол» позаимствовали из химической тер­минологии. Так появился «Тосол».

Название превратилось в имя нарицательное. Од­нако в эпоху безграмотности ситуация изменилась: этим именем начали называть любые охлаждающие жидкости для машин отечественного производства. «Тосол» и «антифриз» стали чуть ли не синонимами определений «плохой» и «хороший». К сожалению, та­кое разделение охлаждающих жидкостей поддержа­ли все — от оптовиков и розничных торговцев до ав­томобилистов. В то, что сегодня называют тосолом, производители чаще всего добавляют присадки, обеспечивающие разве что минимальные антикор­розионные свойства. И это вполне объяснимо: деше­во и вполне достаточно для Жигулей. А вот что зали­вают в яркие и привлекательные канистры с надпися­ми «Антифриз», мало кому известно. И тем более не­известно, как оно отразится на здоровье современно­го двигателя дорогой иномарки.

Избежать неприятностей вам помогут наши экс­пертизы За Рулем.

Давайте заглянем внутрь двигателя. Поршни носятся вверх-вниз, вращается коленчатый вал. Подобная «гимнастика» неизбежно вызывает колебания стенок цилиндров: гильзы начинают вибрировать. Охлаждающая жидкость испытывает перепады давления, периодически вскипая. При этом на наружных стенках гильз возникают воздушные пузырьки, которые, лопаясь, порождают эффект кавитации. И если жидкость легкокипящая (с глицерином и метанолом), кавитация резко усиливается. Пузырьки взрываются, выгрызая из гильз кусочки металла. После этого — два пути: капремонт или помойка. Аналогичным образом от кавитации страдает крыльчатка насоса охлаждающей жидкости.

Кадр посвящается тем, кто полагает, будто все охлаждающие жидкости разливаются из одной бочки. Ничего подобного! Синяя уже кипит, а вот красная и не думает.

Кроме того, если в системе охлаждения возникают паровые пробки, то охлаждающая жидкость перестает циркулировать. Двигатель начинает перегреваться, метанол из охлаждающей жидкости выкипает, остается водный раствор глицерина с температурой кристаллизации от -20 до -12 °С. И такой «антифриз» вполне может замерзнуть.

Страшилка для тех, кто верит сказкам про безвредность охлаждающих жидкостей с метанолами и глицеринами. В прежней жизни это была качественная чугунная гильза. Но борьбу с кавитацией она проиграла нокаутом. Вместе с двигателем, естественно.

 

Что получилось в тесте?

 

В тесте участвовало около трех десятков жидкостей — и почти половина не выдержала проверку! Мы измеряли у каждой жидкости температуру кипения и пытались ее поджечь.

Фрагмент испытаний охлаждающих жидкостей на коррозию. Полная проверка занимает 336 часов при температуре 88 °С. Каждая жидкость проходит проверку на шести различных образцах: медь, припой, латунь, сталь, чугун, алюминий. В конце теста оценивают изменение массы каждой пластинки, а также изучают ее внешний вид.

По науке наличие метанола, как и иных веществ, оценивают на хроматографах. Но мы специально ставили наглядный эксперимент. Результаты плачевные: почти половина проверенных охлаждающих жидкостей может отправить двигатель в капремонт.

Тех, кому захочется повторить наши эксперименты, предупреждаем: использовать следует только химические стаканчики из тонкого стекла. Граненый может лопнуть! Перчатки тоже не помешают.

Чуть больше года назад (ЗР, № 8, 2016) мы провели аналогичную экспертизу — тогда из дюжины протестированных охлаждающих жидкостей половина оказалась пожароопасной. Три из них — Pilot, Unix и Аляsка — вновь попали к нам на проверку. И опять та же картина: горят синим пламенем. Видимо, производители за это время не успели (или не захотели) изменить технологию и наладить выпуск негорючих охлаждающих жидкостей. Самым страшным для них будет наказание рублем — если люди перестанут покупать их продукцию. Равно как и другие охлаждающие жидкости, не выдержавшие испытания.

Результаты теста — в фотогалерее. Они относятся только к конкретным образцам приобретенной нами продукции, а потому не могут служить основанием для каких-либо выводов о деятельности той или иной компании в целом.

Как выбрать хорошую охлаждающую жидкость? Самый простой и надежный вариант — воспользоваться результатами наших исследований и приобретать только ту продукцию, которая успешно выдержала испытание.

Вы продлите жизнь вашему двигателю и сможете избежать дорогостоящего ремонта.

 

oil-sales.ru

Экспертиза «За рулем»: тест охлаждающих жидкостей на горение: тесты и обзоры

Журнал «За рулем»,16 августа 2016 г.
Многие продаваемые у нас охлаждающие жидкости могут стать причиной пожара.
Мы в этом убедились, проверив на горючесть 12 образцов антифризов. Результаты — в тексте редакции.

Недавно по телевидению прошел сюжет про творог, который горит! Я не знаю, что туда подмешали. Но абсолютно уверен, что получить легковоспламеняющуюся охлаждающую жидкость намного проще. Достаточно при производстве вместо дорогого этиленгликоля использовать дешевый глицерин и добавить метанол для приведения вязкости продукта к приемлемому уровню. Причем этим занимаются многие производители. И никому не говорят, что температура кипения метанола — всего-навсего 64,5°C. Зачем, дескать, пугать людей?

Чем метанол опасен для моторов, мы писали неоднократно. Но никогда не заостряли внимание на том, что утечка в системе охлаждения может привести… к пожару. Попади метанол, скажем, на раскаленный коллектор — и привет: вспоминайте, как звонить с мобильника пожарным. Не верите? Мы провели небольшие испытания.

Закупив в подмосковном магазинчике дюжину различных охлаждающих жидкостей, именуемых тосолами и антифризами, мы напросились в гости в химлабораторию — измерить температуру кипения и определить таким образом наличие метанола. В серьезных испытаниях для этого обычно используют хроматограф, который легко покажет на экране состав проверяемой жидкости. Но наша цель была иной. Хроматограф — это нечто заумное, а электрическая плитка и термометр сделают проверку простой и наглядной. Особенно когда на плитке одновременно стоят два стаканчика с жидкостями, одна из которых вовсю кипит, а вторая и не думает. Именно такой эксперимент мы и провели. Тем, кто захочет его повторить, очень советуем раздобыть специальные термостойкие стаканчики, которые не станут трескаться в неподходящий момент.

Мы нагревали каждую приобретенную жидкость, фиксируя температуру начала кипения (появление первых пузырьков) и собственно температуру кипения. Полученные результаты приводим в нашей фотогалерее.

Вспомнив про горящий творог, мы попытались поочередно поджечь все проверяемые жидкости зажигалкой. Результат жутковатый: пять образцов из дюжины мгновенно вспыхнули. Более того, если на плитке одновременно находились два стаканчика — один со стандартным антифризом, а другой с метанольным коктейлем, то при попытке поджечь хорошую жидкость вспыхивал… соседний состав. Пары метанола очень не любят раскаленных поверхностей и открытого огня!

Результаты наших исследований относятся к конкретным образцам приобретенной нами продукции, а потому не могут служить основанием для выводов о деятельности той или иной компании. Вполне возможно, что некоторые производители повторят наш эксперимент — прежде всего те, чья продукция на поверку оказалась небезопасной.

Еще один простейший способ выявить жидкость низкого качества — использовать тест-полоски, определяющие наличие алкоголя в слюне. Во всех случаях индикаторы, искупавшись в метанольном коктейле, поменяли цвет.

А что делать потребителю? Кипятить товар перед покупкой ему точно не разрешат. Один из способов оградить себя от неприятностей — приобретать только те товары, которые хорошо зарекомендовали себя в наших экспертизах. Избегайте торговых развалов на обочинах дорог. Хотя и в магазинах (как следует из нашего эксперимента) не всегда продают качественный товар. Причем продавцы иногда понятия не имеют, какие неприятности сулит нормальная с виду канистра. Поэтому еще один совет: пользуйтесь услугами проверенного автосервиса. Сервисмены судят о качестве приобретенной партии товара, устраивая выборочную проверку. Это куда надежнее единичных закупок.

Sintec Euro G11, Россия

Температура начала кипения 99 ºС
Температура кипения 105 ºС
НЕ ГОРИТ!

Антифриз SINTEC EURO – охлаждающая жидкость последнего поколения, созданная на основе высококачественного моноэтиленгликоля и импортных ингибиторов коррозии. Обладает смазывающими свойствами, продлевающими ресурс водяного насоса. Не повреждает резиновые детали системы охлаждения.

Аntifreeze Maxim, Россия

Температура начала кипения 70 ºС
Температура кипения 84 ºС
ГОРИТ!

Antifreeze G11 Pilot, Россия

Температура начала кипения 70 ºС
Температура кипения 84 ºС
ГОРИТ!

Unix Antifreeze, Россия

Температура начала кипения 79 ºС
Температура кипения 88 ºС
ГОРИТ!

Nord Antifreeze, Россия

Температура начала кипения 106 ºС
Температура кипения 109 ºС
НЕ ГОРИТ!

ТCL Anti-Freeze, Япония

Температура начала кипения 100 ºС
Температура кипения 105 ºС
НЕ ГОРИТ!

Антифриз CoolStream Standard —40 ºС, Россия

Температура начала кипения 105 ºС
Температура кипения 109 ºС
НЕ ГОРИТ!


Тосол —40 Felix, Россия

Температура начала кипения 99 ºС
Температура кипения 105 ºС
НЕ ГОРИТ!

Тосол Айсберг, Россия

Температура начала кипения 103 ºС
Температура кипения 106 ºС
НЕ ГОРИТ!

Тосол Аляsка, Россия

Температура начала кипения 91 ºС
Температура кипения 93 ºС
ГОРИТ!

Тосол Дзержинский Maxim, Россия

Температура начала кипения 70 ºС
Температура кипения 89 ºС
ГОРИТ!

Тосол-север 40, Россия

Температура начала кипения 103 ºС
Температура кипения 106 ºС
НЕ ГОРИТ!

Источник: www. zr.ru

antifreeze.oos.ru

Чем Тосол отличатся от антифриза?

По своей сути — ничем. Это разные названия одной и той же жидкости, предназначенной для заливки в систему охлаждения автомобиля. Но вот состав Тосола и современных антифризов заметно отличается

Станислав Шустицкий

Чтобы полностью ответить на волнующий многих автовладельцев вопрос, нужно, прежде всего, внести ясность в названия и классификации охлаждающих жидкостей. Тосол – это наименование конкретного отечественного антифриза, разработанного в начале 70-х годов в НИИ органической химии и технологии и предназначенного для автомобилей ВАЗ взамен итальянского аналога охлаждающей жидкости. Тосол по оригинальной технологии уже не производится и сегодняшние одноименные охлаждающие жидкости – это, как правило, силикатные антифризы. По классификации охлаждающих жидкостей Тосол относится к традиционным антифризам, большим минусом которых является недостаток неорганических присадок. В процессе работы Тосола пленка, препятствующая коррозии, покрывает всю внутреннюю часть системы охлаждения, что препятствует эффективному теблообмену. По этой причине в термонагруженных двигателях современных автомобилей Тосол практически не используются. И, конечно же, доливать Тосол в системы охлаждения, заполненные другими антифризами, нельзя.

Гораздо меньше недостатков у гибридных антифризов с пакетом присадок, в котором присутствует комбинация карбоксилатов и неорганических кислот. Такие антифризы широко применялись для автомобилей, выпущенных с начала 90-х до 2006 года. Тем не менее, ряд автопроизводителей и сегодня используют такие антифризы для первичной заливки на своих конвейерах. Срок службы таких антифризов разные производители определяют как 3 – 5 лет или ориентируют на пробег в 40 – 60 тыс. км. Вот примеры обозначения гибридных антифризов: HOAT, Hybrid Technology Coolants, NF, G11…

Позже были разработаны карбоксилатные антифризы, которые сегодня многие автопроизводители используют для первичной заливки. В этих антифризах карбоксилаты, которые являются отличными ингибиторами коррозии, способны точечно воздействовать на места появления ржавчины. При этом карбоксилаты сохраняют свою стабильность даже при температурах выше 130о C. Среди обозначений технологии гибридных антифризов можно встретить такие: OAT, LLC, Carboxylate coolants, XLC, SF, G12, G12+…

Не так давно появились лобридные антифризы, пакет присадок которых представлен не только карбоксилатами, но и имеет небольшой процент неорганических соединений. Одной из последних спецификаций таких антифризов стала G13, в которой небольшая часть базового этиленгликоля заменена глицерином. Свойство антифриза от этого не ухудшились, но с точки зрения экологии (включая и процесс производства) это решение оказалось предпочтительней.   

Хочу получать самые интересные статьи

Беспилотники «Яндекса» проехали 7 тысяч километров по дорогам Лас-Вегаса без водителя за рулём

{«id»:100927,»url»:»https:\/\/vc.ru\/transport\/100927-bespilotniki-yandeksa-proehali-7-tysyach-kilometrov-po-dorogam-las-vegasa-bez-voditelya-za-rulem»,»title»:»\u0411\u0435\u0441\u043f\u0438\u043b\u043e\u0442\u043d\u0438\u043a\u0438 \u00ab\u042f\u043d\u0434\u0435\u043a\u0441\u0430\u00bb \u043f\u0440\u043e\u0435\u0445\u0430\u043b\u0438 7 \u0442\u044b\u0441\u044f\u0447 \u043a\u0438\u043b\u043e\u043c\u0435\u0442\u0440\u043e\u0432 \u043f\u043e \u0434\u043e\u0440\u043e\u0433\u0430\u043c \u041b\u0430\u0441-\u0412\u0435\u0433\u0430\u0441\u0430 \u0431\u0435\u0437 \u0432\u043e\u0434\u0438\u0442\u0435\u043b\u044f \u0437\u0430 \u0440\u0443\u043b\u0451\u043c»,»services»:{«facebook»:{«url»:»https:\/\/www. facebook.com\/sharer\/sharer.php?u=https:\/\/vc.ru\/transport\/100927-bespilotniki-yandeksa-proehali-7-tysyach-kilometrov-po-dorogam-las-vegasa-bez-voditelya-za-rulem»,»short_name»:»FB»,»title»:»Facebook»,»width»:600,»height»:450},»vkontakte»:{«url»:»https:\/\/vk.com\/share.php?url=https:\/\/vc.ru\/transport\/100927-bespilotniki-yandeksa-proehali-7-tysyach-kilometrov-po-dorogam-las-vegasa-bez-voditelya-za-rulem&title=\u0411\u0435\u0441\u043f\u0438\u043b\u043e\u0442\u043d\u0438\u043a\u0438 \u00ab\u042f\u043d\u0434\u0435\u043a\u0441\u0430\u00bb \u043f\u0440\u043e\u0435\u0445\u0430\u043b\u0438 7 \u0442\u044b\u0441\u044f\u0447 \u043a\u0438\u043b\u043e\u043c\u0435\u0442\u0440\u043e\u0432 \u043f\u043e \u0434\u043e\u0440\u043e\u0433\u0430\u043c \u041b\u0430\u0441-\u0412\u0435\u0433\u0430\u0441\u0430 \u0431\u0435\u0437 \u0432\u043e\u0434\u0438\u0442\u0435\u043b\u044f \u0437\u0430 \u0440\u0443\u043b\u0451\u043c»,»short_name»:»VK»,»title»:»\u0412\u041a\u043e\u043d\u0442\u0430\u043a\u0442\u0435″,»width»:600,»height»:450},»twitter»:{«url»:»https:\/\/twitter. com\/intent\/tweet?url=https:\/\/vc.ru\/transport\/100927-bespilotniki-yandeksa-proehali-7-tysyach-kilometrov-po-dorogam-las-vegasa-bez-voditelya-za-rulem&text=\u0411\u0435\u0441\u043f\u0438\u043b\u043e\u0442\u043d\u0438\u043a\u0438 \u00ab\u042f\u043d\u0434\u0435\u043a\u0441\u0430\u00bb \u043f\u0440\u043e\u0435\u0445\u0430\u043b\u0438 7 \u0442\u044b\u0441\u044f\u0447 \u043a\u0438\u043b\u043e\u043c\u0435\u0442\u0440\u043e\u0432 \u043f\u043e \u0434\u043e\u0440\u043e\u0433\u0430\u043c \u041b\u0430\u0441-\u0412\u0435\u0433\u0430\u0441\u0430 \u0431\u0435\u0437 \u0432\u043e\u0434\u0438\u0442\u0435\u043b\u044f \u0437\u0430 \u0440\u0443\u043b\u0451\u043c»,»short_name»:»TW»,»title»:»Twitter»,»width»:600,»height»:450},»telegram»:{«url»:»tg:\/\/msg_url?url=https:\/\/vc.ru\/transport\/100927-bespilotniki-yandeksa-proehali-7-tysyach-kilometrov-po-dorogam-las-vegasa-bez-voditelya-za-rulem&text=\u0411\u0435\u0441\u043f\u0438\u043b\u043e\u0442\u043d\u0438\u043a\u0438 \u00ab\u042f\u043d\u0434\u0435\u043a\u0441\u0430\u00bb \u043f\u0440\u043e\u0435\u0445\u0430\u043b\u0438 7 \u0442\u044b\u0441\u044f\u0447 \u043a\u0438\u043b\u043e\u043c\u0435\u0442\u0440\u043e\u0432 \u043f\u043e \u0434\u043e\u0440\u043e\u0433\u0430\u043c \u041b\u0430\u0441-\u0412\u0435\u0433\u0430\u0441\u0430 \u0431\u0435\u0437 \u0432\u043e\u0434\u0438\u0442\u0435\u043b\u044f \u0437\u0430 \u0440\u0443\u043b\u0451\u043c»,»short_name»:»TG»,»title»:»Telegram»,»width»:600,»height»:450},»odnoklassniki»:{«url»:»http:\/\/connect. ok.ru\/dk?st.cmd=WidgetSharePreview&service=odnoklassniki&st.shareUrl=https:\/\/vc.ru\/transport\/100927-bespilotniki-yandeksa-proehali-7-tysyach-kilometrov-po-dorogam-las-vegasa-bez-voditelya-za-rulem»,»short_name»:»OK»,»title»:»\u041e\u0434\u043d\u043e\u043a\u043b\u0430\u0441\u0441\u043d\u0438\u043a\u0438″,»width»:600,»height»:450},»email»:{«url»:»mailto:?subject=\u0411\u0435\u0441\u043f\u0438\u043b\u043e\u0442\u043d\u0438\u043a\u0438 \u00ab\u042f\u043d\u0434\u0435\u043a\u0441\u0430\u00bb \u043f\u0440\u043e\u0435\u0445\u0430\u043b\u0438 7 \u0442\u044b\u0441\u044f\u0447 \u043a\u0438\u043b\u043e\u043c\u0435\u0442\u0440\u043e\u0432 \u043f\u043e \u0434\u043e\u0440\u043e\u0433\u0430\u043c \u041b\u0430\u0441-\u0412\u0435\u0433\u0430\u0441\u0430 \u0431\u0435\u0437 \u0432\u043e\u0434\u0438\u0442\u0435\u043b\u044f \u0437\u0430 \u0440\u0443\u043b\u0451\u043c&body=https:\/\/vc.ru\/transport\/100927-bespilotniki-yandeksa-proehali-7-tysyach-kilometrov-po-dorogam-las-vegasa-bez-voditelya-za-rulem»,»short_name»:»Email»,»title»:»\u041e\u0442\u043f\u0440\u0430\u0432\u0438\u0442\u044c \u043d\u0430 \u043f\u043e\u0447\u0442\u0443″,»width»:600,»height»:450}},»isFavorited»:false}

8222 просмотров

Производство антифриза, тосола, охлаждающей жидкости для автомобилей

Производство антифриза

Компания Спектропласт разработала и выпускает инновационные антифризы, охлаждающие жидкости для автомобильной техники. Марка Spektrogen Auto отличается пониженной вязкостью и не имеет аналогов на рынке России. Компания имеет собственную аккредитованную лабораторию и завод, на котором налажено серийное производство антифризов 3 марок: на основе пропиленгликоля, этиленгликоля и низковязкий антифриз. Вся продукция сертифицирована и прошла долговременные испытания на различной автомобильной технике.

Выбор антифриза

Наиболее часто встречающиеся вопросы при выборе или замене антифриза, предохраняющего систему охлаждения автомобильного двигателя внутреннего сгорания от коррозионных процессов: в чём разница между антифризом и тосолом? Какой антифриз в итоге заливать? Какой антифриз для авто лучше выбрать? Какой антифриз купить из такого разнооборазия предложений?

Нет смысла сравнивать традиционные антифризы для автомобилей и тосолы, мы акцентируем внимание на конкурентных преимуществах новой российской разработки — нетоксичном антифризе марки Spektrogen Auto для систем охлаждения двигателей автотранспорта. Данный продукт предлагается от производителя по контрактному производству.

Основные преимущества Spektrogen Auto:

  • Энергоэффективность
  • При производстве антифризов Spektrogen Auto серии LV используется инновационная технология снижения вязкости жидкости. Благодаря низкой вязкости повышается эффективность прогрева двигателя и его КПД, что позволяет запускать двигатель при низких температурах за более короткое время, что также лучше для автомобилей, не адаптированных для езды в суровых условиях климата.

  • Экологичность
  • Антифриз Spektrogen Auto серии S изготавливается на основе 1,2-пропиленгликоля (пищевая добавка Е-1520), что обеспечивает его низкую токсичность и высокую экологическую безопасность (ПДК в воздухе рабочей зоны не установлен в связи с низкой летучестью и слабой токсичностью, средняя смертельная доза LD50 составляет более 18 гр/кг).

  • Высокая надежность
  • В антифризах марки Spektrogen Auto для автомобилей применяется нетоксичный и экологически безопасный комплекс присадок, эффективно снижающих коррозию металлов, пенообразование, а также препятствующих набуханию и растворению уплотнителей. Антифриз Spektrogen Auto содержит антикавитационный пакет присадок и пакет гибридных ингибиторов коррозии, содержащий соли органических кислот и силикаты и обеспечивающий надежную защиту от коррозии систем охлаждения современных двигателей внутреннего сгорания.

  • Пожаровзрывобезопасность
  • Антифриз Spektrogen Auto является негорючим и взрывобезопасным веществом. Согласно ГОСТ 12.1.007-76 относится к 3 классу опасности (умеренно опасные вещества) (для серии ОЖ) и к 4 классу опасности (малоопасные вещества) (для серии LV). Допускает разбавление умягченной или дистиллированной водой, соответствующей требованиям ГОСТ 6709-72. Температура хранения: от минус 50 °С до плюс 60 °С.

Температурный диапазон эксплуатации антифризов Spektrogen Auto для автомобиля и автотранспорта находится в интервале от -65°С до +115 °С. На марки антифриза Spektrogen Auto серии S имеется санитарно-эпидемиологическое заключение № 77.01.12.249.П.067 899.09. 07, что подтверждает его соответствие санитарно-эпидемиологическим стандартам. Антифризы Spektrogen Auto S (S-LV) являются экологичными, токсикологически безопасными, негорючими и взрывобезопасными веществами. Производятся на основе пищевой добавки Е1520. Не рекомендуется смешивание антифриза Spektrogen Auto S-LV с другими видами антифризов. Производство антифризов ведется компанией с более чем 25-летним опытом в области изготовления технологических жидкостей.

Антифриз для автомобилей Spektrogen Auto разливается в пластиковые канистры по 1, 5, 10, 20 кг или металлические бочки по 220 кг.

При заказе от 15 тонн доставка по Московской области бесплатно!

Для региональных дилеров предоставляются специальные финансовые условия.

Мы имеем более чем 25-летний опыт производства антифризов (тосола) и охлаждающих жидкостей для промышленных нужд и готовы предоставить взаимовыгодные условия для региональных дилеров и представителей. Чтобы приобрести антифризы Spektrogen Auto для автомобиля или получить консультацию по вопросу о том, какой антифриз лучше заливать в Ваш автотранспорт, обращайтесь по тел.: +7 (495) 966-08-09, Пн-Пт с 9 до 17.

Необходимость антифриза в машине

От слова ANTIFREEZE мы узнаем что-то, что предотвращает замерзание, или что-то, препятствующее замерзанию, но зачем это нужно в вашей машине? а где он размещен?

Что такое антифриз:

Другой термин для обозначения антифриза — автомобильная охлаждающая жидкость, они оба служат для одной и той же цели. Вода в автомобилях смешивается с ярко-желтой или зеленой жидкостью, которая предотвращает замерзание или перегрев радиаторов автомобилей, грузовиков и других транспортных средств.

Система охлаждения предназначена для понижения точки замерзания, а также для управления повышением точки кипения системы. Охлаждающие жидкости поддерживают работу вашего автомобиля и двигателя в актуальном состоянии и поддерживают его в хорошем состоянии.

Антифриз — это химическое вещество на спиртовой основе, которое состоит из этиленгликоля. Добавляя охлаждающую жидкость или антифриз в систему охлаждения автомобиля, мы можем повысить температуру автомобиля до того, как закипит система охлаждения.

Зависимость точки кипения охлаждающей жидкости от давления:

Температура кипения воды составляет 100 градусов Цельсия, в то время как известный диапазон давления радиатора автомобиля составляет 75-100 кПа (или 11-15 фунтов на квадратный дюйм), а температура смеси антифриза и воды составляет 33%, а температура кипения повышается до 121. 125 ° C, следовательно, изменение соотношения поднимается до 55%, а это — до 125–129 ° C.

Опасности для антифриза:

У всего есть свои преимущества и недостатки, но с одной стороны, если охлаждающие жидкости эффективны, но есть и вред. Химическое вещество со слегка сладковатым запахом даже в небольших количествах вредно для домашних животных и людей,

1- Две унции этиленгликоля могут убить собаку.

2- Одна чайная ложка этиленгликоля может быть смертельной для кошки.

3- Две столовые ложки этиленгликоля могут быть опасны для детей.

4- Проглоченный этиленгликоль превращается в кислоту, которая напрямую вызывает повреждение почек и их отказ, даже смерть.

Использование воды против охлаждающей жидкости:

Идея использования воды стала больше, чем обычно, потому что цель охлаждающей жидкости в двигателе вашего автомобиля состоит в основном в повышении температуры кипения воды, которая нагнетается вокруг двигателя вашего автомобиля и с помощью которой охлаждается радиатор, по которому течет воздух. воду и приводит ее в норму.

Очень важно проверять и балансировать охлаждающую жидкость или антифриз в вашем автомобиле. Его следует утилизировать для охлаждения и время от времени заменять. Очень важно регулярно проверять уровень жидкости в радиаторах. С помощью ареометра мы проверяем состояние охлаждающей жидкости и узнаем минимальную и максимальную температуру антифриза или охлаждающей жидкости на разных уровнях.

Если жидкость выглядит немного молочной, значит, масло попало в систему из-за утечки, это может привести к заглушению двигателя вашего автомобиля.Немедленно доставьте машину к механику. Никогда не сливайте антифриз или охлаждающую жидкость в канализацию или на землю, так как это приведет к загрязнению рек и ручьев, протекающих по земле. Вместо того, чтобы утилизировать, храните его в четко маркированных контейнерах и отдавайте любому переработчику антифриза, они могут повторно использовать его, перегонять или фильтровать, удаляя опасные химические вещества и настраивая его для повторного использования.

Проверка охлаждающей жидкости | MotorWeek

Удивительно, но в моей ремонтной мастерской мы обнаруживаем, что одна из самых запущенных частей среднего автомобиля — это система охлаждения.Кажется, что многие люди смотрят на это как на «вне поля зрения; из виду. » Они не обращают на это внимания. Но на самом деле вы должны проверять охлаждающую жидкость в системе охлаждения не реже одного раза в год. Теперь, независимо от того, что говорится в руководстве пользователя — в руководстве пользователя может быть сказано, что охлаждающая жидкость рассчитана на срок службы до 5 лет или 150 000 миль — это не значит, что так и будет. Так что будьте активны и проверьте его, прежде чем он нанесет ущерб.

Вы можете использовать простой ареометр, подобный этому; всасывает охлаждающую жидкость внутрь и сообщает о защите от замерзания.Он также сообщает вам, прозрачная ли охлаждающая жидкость и имеет ли она нужный цвет. Но лучший способ сделать это прямо здесь. Это тест-полоски для охлаждающей жидкости. Вы окунаете их в охлаждающую жидкость, затем маленькие подушечки на них меняют цвет, и вы сравниваете этот цвет с цветами на боковой стороне бутылки, и это говорит вам, хорошая или плохая охлаждающая жидкость.

Теперь еще один тест, и этот немного сложный, но вам нужен простой вольтметр — вы устанавливаете его на постоянное напряжение — вы подключаете отрицательный кабель к отрицательной клемме батареи; затем при снятой крышке радиатора и скорости вращения автомобиля около 2000 об / мин, вы погружаете красный зонд в охлаждающую жидкость, вот так, и вы получаете здесь показания. Это показание должно быть меньше 4/10 одного вольт. Если оно превышает 4/10 вольт, охлаждающая жидкость должна выйти, система будет промыта, а если это не поможет, вам нужно проверить электрическую систему в автомобиле.

Кроме того, охлаждающая жидкость, которую вы в него залили? Что ж, один размер не обязательно подходит всем. Итак, вы хотите заглянуть в руководство по эксплуатации. Должен быть номер, квалификационный номер или номер ASTM; убедитесь, что этот номер указан на емкости с охлаждающей жидкостью, которую вы покупаете.Всегда используйте дистиллированную воду. Вы смешиваете охлаждающую жидкость и воду в соотношении 50/50 вне автомобиля, затем заливаете смесь в машину. Никогда не используйте простую воду и обычную охлаждающую жидкость, потому что это может нарушить баланс в системе. Позаботьтесь об этом, и у вас никогда не будет проблем с системой охлаждения. Мы надеемся.

И если у вас есть вопрос или комментарий, напишите мне прямо здесь: MotorWeek .

Купить антифриз и охлаждающие жидкости для легковых и грузовых автомобилей

Купить антифризы и охлаждающие жидкости для легковых и грузовых автомобилей — AMSOIL

U.Доставка S.O. может быть отложена до 3-5 дней из-за суровой погоды во многих регионах.

  1. Дома
  2. Товары
  3. Антифриз и охлаждающая жидкость
  4. Антифризы и охлаждающие жидкости для легковых и грузовых автомобилей

Разработан , чтобы превосходить требования производителей оригинального оборудования (OEM)
Защита до 150 000 миль или 5 лет, в зависимости от того, что наступит раньше
Этиленгликоль 50/50 состав предварительной смеси
Защищает все охлаждение Металлы системы, прокладки и шланги
Фосфат — , не содержит нитратов, нитритов, силикатов, боратов и аминов
Совместим с со всеми другими цветами антифриза и охлаждающей жидкости

23 доллара.50

Розничная цена (долл. США)

Предпочтительная цена: 15,20 $
Сохранять: 8,30 долл. США Узнать больше

Размер упаковки

Выберите размер Галлонный кувшин Бочка на 55 галлонов Сумка на 264 галлона

Единица измерения

Выберите размер Каждый Корпус 4

Антифриз и охлаждающая жидкость для тяжелых условий эксплуатации
Код продукта: ANTHD1G-EA

Полная формула : НЕ требует использования дополнительных присадок к антифризу / охлаждающей жидкости (SCA) или расширителей.
Защита до 1 000 000 миль, 20 000 часов или 8 лет, в зависимости от того, что наступит раньше
Этиленгликоль 50 / 50 состав предварительной смеси
Запатентованная рецептура гибридной технологии органических кислот (HOAT) дополнительно улучшена за счет добавок против накипи, против обрастания и смазочных присадок
Фосфат- , нитрат-, нитрит-, силикат- , не содержит боратов и аминов
Совместим с со всеми другими антифризами и охлаждающими жидкостями

Просмотреть страницу сведений Предупреждение — Щелкните здесь, чтобы получить информацию о предложении 65 для этого продукта

Товар не продается

Полная формула : НЕ требует использования дополнительных присадок к антифризу / охлаждающей жидкости (SCA) или расширителей.
Защита до 1 000 000 миль, 20 000 часов или 8 лет, в зависимости от того, что наступит раньше
Этиленгликоль 50 / 50 состав предварительной смеси
Запатентованная рецептура гибридной технологии органических кислот (HOAT) дополнительно улучшена за счет добавок против накипи, против обрастания и смазочных присадок
Фосфат- , нитрат-, нитрит-, силикат- , не содержит боратов и аминов
Совместим с со всеми другими цветами антифриза и охлаждающей жидкости

25 долларов.05

Розничная цена (долл. США)

Предпочтительная цена: 16,45 $
Сохранять: 8,60 долл. США Узнать больше

Размер упаковки

Выберите размер Галлонный кувшин Бочка на 55 галлонов Сумка на 264 галлона

Единица измерения

Выберите размер Каждый Корпус 4

Низкотоксичный антифриз и охлаждающая жидкость двигателя
Код продукта: ANT1G-EA

Низкотоксичный, биоразлагаемый

AMSOIL Антифриз и охлаждающая жидкость на основе пропиленгликоля (ANT) разработан для обеспечения преимуществ, превосходящих те, которые обеспечивают современные традиционные антифризы и охлаждающие жидкости.В отличие от обычных продуктов на основе этиленгликоля, которые являются остро токсичными, AMSOIL Propylene Glycol Antifreeze & Coolant является биоразлагаемым, а его низкая токсичность снижает опасность для детей и животных. Его уникальный состав обеспечивает максимальную защиту системы охлаждения при экстремальных температурах и условиях эксплуатации.

Просмотреть страницу сведений

Товар не продается

AMSOIL Антифриз и охлаждающая жидкость на основе пропиленгликоля (ANT) разработан для обеспечения преимуществ, превосходящих те, которые обеспечивают современные традиционные антифризы и охлаждающие жидкости.В отличие от обычных продуктов на основе этиленгликоля, которые являются остро токсичными, AMSOIL Propylene Glycol Antifreeze & Coolant является биоразлагаемым, а его низкая токсичность снижает опасность для детей и животных. Его уникальный состав обеспечивает максимальную защиту системы охлаждения при экстремальных температурах и условиях эксплуатации.

$ 41,65

Розничная цена (долл. США)

Предпочтительная цена: 29,95 долл. США
Сохранять: 11 долларов.70 Узнать больше

Размер упаковки

Выберите размер Галлонный кувшин Бочка на 55 галлонов Сумка на 264 галлона

Единица измерения

Выберите размер Каждый Корпус 4

Dominator® Coolant Boost (усилитель охлаждающей жидкости)
Код продукта: RDCBCN-EA

Обеспечивает эффективную теплопередачу и улучшенную защиту от коррозии

Создан с использованием запатентованной многоуровневой технологии поверхностно-активных веществ, обеспечивающей быструю и эффективную передачу тепла внутри радиатора и головок цилиндров, что приводит к снижению рабочих температур, более эффективной работе, увеличению мощности и значительному сокращению времени прогрева двигателя в холодную погоду.

Просмотреть страницу сведений

Товар не продается

Создан с использованием запатентованной многоуровневой технологии поверхностно-активных веществ, обеспечивающей быструю и эффективную передачу тепла внутри радиатора и головок цилиндров, что приводит к снижению рабочих температур, более эффективной работе, увеличению мощности и значительному сокращению времени прогрева двигателя в холодную погоду.

13,00 $

Розничная цена (долл. США)

Предпочтительная цена: 9 долларов.95
Сохранять: 3,05 доллара США Узнать больше

Размер упаковки

Выберите размер 16 унций. Бутылка

Единица измерения

Выберите размер Каждый Корпус 6

Тест-полоски антифриза
Код продукта: G1164-EA

Доступен в упаковках по 6 или 70 штук.Используйте раскрывающийся список РАЗМЕР УПАКОВКИ, чтобы выбрать количество. Тест-полоски антифриза AMSOIL имеют две разные прокладки и позволяют тестировать ВСЕ цвета охлаждающей жидкости. Самый низкий сообщает об уровне защиты от замерзания и кипения (под давлением). Верхняя панель сообщает об уровне pH и показывает зону годен / не прошел. Никакого другого тестирования не рекомендуется и не требуется. Тест-полоски — единственный способ проверить смесь этиленгликоля и пропиленгликоля на предмет защиты от замерзания.

Просмотреть страницу сведений

Товар не продается

Доступен в упаковках по 6 или 70 штук.Используйте раскрывающийся список РАЗМЕР УПАКОВКИ, чтобы выбрать количество. Тест-полоски антифриза AMSOIL имеют две разные прокладки и позволяют тестировать ВСЕ цвета охлаждающей жидкости. Самый низкий сообщает об уровне защиты от замерзания и кипения (под давлением). Верхняя панель сообщает об уровне pH и показывает зону годен / не прошел. Никакого другого тестирования не рекомендуется и не требуется. Тест-полоски — единственный способ проверить смесь этиленгликоля и пропиленгликоля на предмет защиты от замерзания.

13,30 долл. США

Розничная цена (долл. США)

Предпочтительная цена: 12 долларов.10
Сохранять: 1,20 $ Узнать больше

Размер упаковки

Выберите размер Упаковка из 6 шт. Упаковка 70 шт.

Нужна помощь? Мы можем помочь вам найти нужный продукт, ответить на технические вопросы, сбросить пароль или что-то еще.Просто дайте нам знать, что вам нужно.

СВЯЗАТЬСЯ С НАМИ

Ваш браузер устарел.

Спасибо, что посетили www.amsoil.com. Некоторые части этого веб-сайта могут не работать с Internet Explorer.
Чтобы продолжить покупки у нас, обновите свой браузер, используя один из способов ниже.

Проверьте антифриз, чтобы избежать зимней хандры Совет по уходу за автомобилем предлагает советы по зимнему профилактическому обслуживанию

Проверка уровня антифриза в вашем автомобиле в холодные зимние месяцы — простой и недорогой способ убедиться, что ваш автомобиль не оставит вас в затруднительном положении.Антифриз важен, потому что он используется для охлаждения двигателя автомобиля, а также для защиты его от замерзания в холодную погоду. Он также действует как ключевой фактор в обеспечении тепла внутри кабины автомобиля.

«Осмотр и обслуживание системы охлаждения вашего автомобиля занимает всего несколько минут вашего времени, но оно того стоит, если учесть, что может случиться», — сказал Рич Уайт, исполнительный директор Совета по уходу за автомобилями. «Отказ системы охлаждения — основная причина поломок двигателя, которая может стоить тысячи долларов и оставить вас и вашу семью в затруднительном положении в самый неподходящий момент.”

Система охлаждения защищает от повреждений, поддерживая работу двигателя в правильном температурном диапазоне. Более 27 процентов автомобилей, проверенных в течение Национального месяца ухода за автомобилями, имели низкий уровень охлаждающей жидкости, в то время как 20 процентов нуждались в промывке охлаждающей жидкости, которая необходима для защиты системы от ржавчины, грязи и минеральных отложений.

Чтобы проверить уровень антифриза / охлаждающей жидкости, вам понадобятся несколько основных инструментов для обслуживания и тестер антифриза, который можно приобрести в местном магазине автозапчастей.Перед началом работы убедитесь, что двигатель и система охлаждающей жидкости остыли. Открытие горячего радиатора или бачка охлаждающей жидкости / переливного бачка может вызвать серьезные ожоги. Если уровень антифриза / охлаждающей жидкости низкий, добавьте смесь одобренного антифриза и дистиллированной воды в соотношении 50/50. Если вы недавно меняли антифриз, но его уровень низкий, используйте тестер антифриза, чтобы убедиться, что соотношение незамерзающей жидкости и воды правильное.

Это также хорошее время, чтобы проверить и заменить любые протекающие, хрупкие, пористые или потрескавшиеся шланги, а также убедиться, что хомуты шлангов радиатора затянуты, чтобы предотвратить утечки в соединениях.

Совет по уходу за автомобилем также рекомендует самостоятельно или профессиональным автомехаником выполнить следующие задачи, чтобы убедиться, что автомобиль готов к зимнему вождению:

Замена масла каждые 3000 км; подумайте о переходе на масло «зимнего веса», если вы живете в холодном климате. Для меньшего износа двигателя водители при отрицательных температурах вождения должны снизить вес масла с 10-W30 до 5-W30.
Проверить аккумулятор на наличие видимых следов коррозии. Перед поездкой на большие расстояния рекомендуется проверить срок службы аккумулятора у профессионального специалиста по обслуживанию.
Убедитесь, что нагреватель / дефростер работает должным образом, и держите бензобак полным, чтобы минимизировать риск замерзания газопровода.
Проверить глубину протектора шины и давление в шинах; подумайте о специальных шинах, если в вашем районе есть проблемы со снегом и льдом.
Убедитесь, что фары работают и фары правильно направлены.
Заменяйте щетки стеклоочистителя каждые шесть месяцев; подумайте о специальных снежных ножах, если того требует погода.

Совет по уходу за автомобилем является источником информации для кампании по обучению потребителей «Будьте внимательны к уходу за автомобилем», продвигая для потребителей преимущества регулярного ухода за автомобилем, технического обслуживания и ремонта.Чтобы получить бесплатный график интервалов обслуживания, посетите www.carcare.org.

Антифриз охлаждающей жидкости — охлаждение двигателя и тестирование стали проще

Охлаждающая жидкость Антифриз — охлаждение двигателя и тестирование стали проще

Антифриз для охлаждающей жидкости предназначен для защиты двигателя от высоких температур и коррозии.

Всегда промывайте и заменяйте антифриз охлаждающей жидкости через рекомендуемые интервалы, иначе он выйдет из строя и не сможет выполнять свою работу.

Преждевременное ухудшение качества антифриза может привести к серьезному повреждению двигателя и системы охлаждения.

Антифриз охлаждающей жидкости — для чего он нужен

Проверка прочности охлаждающей жидкости-антифриза так же важна при вождении в жаркую погоду, как и в холодную погоду.

Смесь 50/50 этиленгликоля (EG) антифриза и воды даст:

  • Защита от кипения до примерно 255 градусов по Фаренгейту с крышкой 15 фунтов на кв. Дюйм.
  • Вместе с защитой от замерзания до -34 градусов F.

Для сравнения, смесь антифриза на основе пропиленгликоля (PG) и воды в соотношении 50/50 даст:

  • Защита от кипения до 257 градусов F.
  • Вместе с защитой от замерзания до -26 градусов F.

Увеличение концентрации антифриза в охлаждающей жидкости приведет к повышению ее температуры кипения и снижению точки замерзания.

Коррозия, вызванная электролизом

Охлаждающая жидкость также играет важную роль в предотвращении коррозии, вызванной электролизом.

Коррозия, вызванная электролизом

Электролиз происходит, когда два разнородных металла начинают обмениваться электронами, вызывая коррозию металлов.

Сегодняшние двигатели имеют:

  • Алюминий
  • Медь
  • Чугун
  • Сталь
  • Магниевые сплавы

Электролиз медленно разъедает все внутренние части.

Фактически, старая охлаждающая жидкость становится довольно хорошим проводником электричества, ускоряя внутренний электролиз. Хорошей новостью является то, что проверить проводимость охлаждающей жидкости с помощью цифрового мультиметра довольно просто. В результате, если проводимость высокая, самое время промыть и залить охлаждающую жидкость.

Антифриз охлаждающей жидкости — зачем его промывать и заменять

Промывка системы охлаждения помогает удалить старую окалину и ржавчину. Следовательно, ингибиторы коррозии в новом антифризе помогают предотвратить образование накипи и ржавчины в системе охлаждения. Новый антифриз помогает поддерживать работу двигателя при наиболее эффективной температуре независимо от условий эксплуатации и температуры наружного воздуха.

Если не выполнять промывку антифриза через рекомендуемые интервалы, химические вещества начинают разрушаться и не могут выполнять свою работу.Нарастание ржавчины и накипи может привести к выходу из строя шланга радиатора, водяного насоса или термостата.

Антифриз охлаждающей жидкости — Проверка на электролиз (с помощью цифрового мультиметра):
  • Коррозия может преждевременно способствовать отказу системы охлаждения. Обычно все начинается с водяного насоса, радиатора, сердечника нагревателя и, наконец, термостата.
  • Чтобы использовать цифровой мультиметр, начните с холодного двигателя и откройте крышку радиатора.
  • Никогда не открывайте крышку радиатора при горячем двигателе во избежание ожогов.
  • С снятой крышкой запустите двигатель и дайте ему поработать на холостом ходу, пока он не достигнет рабочей температуры.
  • Возьмите мультиметр, установите его на постоянное напряжение и поместите отрицательный щуп на отрицательную клемму аккумулятора.
  • Что касается плюсового щупа, окуните его прямо в охлаждающую жидкость.
  • Если счетчик показывает 0,4 В или меньше, охлаждающая жидкость в порядке.
  • Если он показывает более 0,4 В, значит, в охлаждающей жидкости вышли из строя добавки, предотвращающие электролиз.

Тестирование только двигателя вентилятора

Отсоедините разъем проводки от вентилятора. Используйте перемычку от аккумулятора, чтобы направить питание непосредственно на вентилятор. Если двигатель вентилятора исправен, вентилятор должен вращаться с нормальной скоростью при напряжении 12 В. Шум в подшипниках или более низкая, чем обычно, скорость указывают на износ двигателя.

Проверка датчика температуры охлаждающей жидкости (ECT) Проверка датчика температуры охлаждающей жидкости (ECT)
  • Сначала убедитесь, что на жгут проводов подается напряжение.Плохой ремень безопасности или перерыв в проводе могут вызывать те же симптомы, что и плохой (ЭСТ).
  • Чтобы проверить жгут на напряжение, поверните ключ в положение ON. Используйте мультиметр для измерения 5 В постоянного тока.
  • Напряжение может незначительно отличаться, но должно быть около 5 вольт.
  • Если у вас хорошее чтение, значит, подвеска в порядке. Переходим к проверке самого датчика охлаждающей жидкости двигателя.
  • Если вы не получаете хорошие показания 5 вольт, значит, жгут неисправен.
  • Датчик можно проверить, находясь в автомобиле.Вам нужно будет добраться до него с помощью щупов мультиметра.
  • Первое испытание следует проводить при холодном двигателе.
  • Чтобы проверить его с помощью измерителя, установите его на Ом, чтобы получить показание сопротивления.
  • Используйте провода мультиметра, чтобы прикоснуться к двум контактам на ECT, где были вставлены разъемы жгута проводов.
  • Хорошее показание при холодном двигателе должно быть между 1,5 и 2 Ом. Если показаний не видно, значит датчик неисправен.
  • Иногда датчик все еще может давать показания, но работает неправильно.Чтобы проверить это, выполните те же считывания, но с прогретым двигателем.
  • Когда двигатель прогрет, сопротивление должно падать. Если он не падает, значит датчик неисправен и требует замены.

Заключение

Следуйте инструкциям в руководстве по эксплуатации вашего автомобиля по промывке и замене охлаждающей жидкости-антифриза. Также убедитесь, что вы используете антифриз соответствующего типа, рекомендованный для вашего автомобиля.

Поделитесь новостями портала DannysEngine

Проверьте термостат автомобиля | Страхование Desjardins

Термостат в вашем автомобиле отвечает за регулирование потока охлаждающей жидкости в двигателе вашего автомобиля.Он реагирует на изменение температуры небольшим открытым или закрытым клапаном. Например, когда вы заводите машину утром, термостат находится в закрытом положении. Автомобильный термостат срабатывает, когда двигатель автомобиля достигает своей полной рабочей температуры; позволяя охлаждающей жидкости течь в радиатор. Это помогает предотвратить перегрев двигателя. Неисправный термостат может повредить ваш двигатель, и другие механизмы системы охлаждения не смогут выполнять свою работу. Чтобы убедиться, что термостат вашего автомобиля находится в рабочем состоянии, важно его проверить.Выполните следующие действия, чтобы узнать, как проверить автомобильный термостат.

Проверка термостата вашего автомобиля

  1. Установите машину на ровной поверхности и убедитесь, что двигатель и радиатор остыли.
  2. Откройте капот автомобиля и найдите термостат. Вы можете сделать это, проследив за верхним шлангом радиатора по направлению к двигателю. Конец этого шланга будет прикреплен к корпусу термостата. В большинстве автомобилей термостат можно найти в корпусе. В некоторых автомобилях корпус термостата крепится к нижнему шлангу радиатора.Если вам нужны дополнительные инструкции по поиску термостата, просмотрите руководство по обслуживанию вашего автомобиля.
  3. Далее вам нужно будет проверить термостат вашего автомобиля. Вы можете сделать это, сняв крышку радиатора вашего автомобиля, чтобы проверить поток охлаждающей жидкости. Запустите двигатель вашего автомобиля и дайте ему поработать на холостом ходу. Посмотрите через заливную горловину радиатора, чтобы проверить, течет ли охлаждающая жидкость. В это время он не должен течь, поскольку ваша машина не достигла рабочей температуры, которая могла бы вызвать открытие термостата.
  4. Если вы обнаружите, что охлаждающая жидкость течет, это означает, что клапан термостата открыт.Обычно это хороший показатель того, нужно ли заменить термостат в автомобиле.
  5. Если вы заметили, что охлаждающая жидкость не течет, подождите, пока двигатель вашего автомобиля не достигнет рабочей температуры. Еще раз посмотрите через наливную горловину радиатора, чтобы увидеть, не потекла ли охлаждающая жидкость. Если вы по-прежнему не видите, как течет охлаждающая жидкость, а датчик температуры вашего автомобиля начинает повышаться, это означает, что термостат вашего автомобиля находится в закрытом положении.

Если крышка радиатора недоступна в вашем автомобиле, выполните следующие действия:

  1. Установите машину на ровной поверхности и убедитесь, что двигатель и радиатор остыли.
  2. Запустите двигатель вашего автомобиля и дайте ему поработать на холостом ходу.
  3. Используйте термометр для готовки или ареометр, чтобы получить показания температуры блока цилиндров (также известного как блок цилиндров) и верхнего шланга радиатора. Конец этого шланга будет прикреплен к корпусу термостата.
  4. Подождите примерно 10 минут и выполните еще одно считывание с обеих частей. Сравните эти результаты с предыдущими.
  5. Подождите еще 10 минут и снова снимите показания обеих частей.Вы должны заметить повышение температуры блока цилиндров, при этом температура верхнего шланга радиатора должна остаться прежней. Если температура блока цилиндров не меняется, это означает, что термостат вашего автомобиля застрял в открытом положении. Это означает, что вам, скорее всего, потребуется заменить термостат вашего автомобиля, поскольку двигатель не может достичь определенной температуры.

Как заменить термостат в автомобиле

  1. Установите машину на ровной поверхности и убедитесь, что двигатель и радиатор остыли.
  2. С помощью отвертки снимите шланг радиатора, сняв зажим.
  3. Далее снимаем шланг. Будьте готовы к тому, что некоторое количество охлаждающей жидкости может вытечь.
  4. Используйте гаечный ключ, чтобы ослабить болты, удерживающие термостат на месте. Снимите крышку и старый термостат.
  5. Удалите старую прокладку с корпуса.
  6. Вставьте новый термостат пружиной вниз. Снова прикрепите болты, шланг и хомут. Возможно, вам также понадобится добавить охлаждающую жидкость двигателя.
  7. Запустите двигатель автомобиля и подождите, пока он достигнет рабочей температуры.
  8. Выйдите на тест-драйв своей машины. Следите за стрелкой указателя температуры, чтобы увидеть, остается ли она в норме при полной рабочей температуре автомобиля.

Похожие сообщения

Как проверить уровни антифриза в автомобиле и проверить отсутствие утечек

Подготовка к зиме: как подготовить машину к холоду

5 хитрых вопросов, которые можно задать своему механику

Весеннее обслуживание автомобиля: 7 советов по подготовке автомобиля к весне

15 февраля 2018

В рамках планового технического обслуживания автомобиля рекомендуется проверять автомобильный термометр, чтобы регулировать рабочую температуру двигателя.Чтобы узнать, как проверить термометр вашего автомобиля, прочитайте наше полезное руководство.

Антифриз Охлаждающая жидкость

Проверка и замена охлаждающей жидкости в современных автомобилях стало намного сложнее из-за различных требований к охлаждающей жидкости, поэтому вот несколько советов, о которых следует помнить:

СИЛА ОХЛАЖДАЮЩЕЙ ЖИДКОСТИ

Для всех легковых и легких грузовиков требуется смесь антифриза и воды не менее 50/50 для обеспечения адекватной защиты от замерзания, кипения и коррозии.Смесь 50/50 обеспечит защиту от замерзания до -34 градусов F с антифризом на основе этиленгликоля (EG) и -26 градусов F с антифризом на основе пропиленгликоля (PG).

Если вы используете ареометр для проверки концентрации антифриза в охлаждающей жидкости, имейте в виду, что удельный вес обычного антифриза EG отличается от удельного веса PG (который имеет плотность, близкую к воде). Поэтому, если кто-то добавил в систему охлаждения «менее токсичный» антифриз PG, вам понадобится либо специальный ареометр для измерения концентрации охлаждающей жидкости, либо рефрактометр (который работает с любым типом антифриза).

Повышение концентрации антифриза в охлаждающей жидкости может обеспечить дополнительную защиту от замерзания в очень холодном климате, но максимальная смесь для антифриза EG составляет 70%. Когда возвращается жаркая погода, охлаждающую жидкость следует снова разбавить до 50/50, потому что антифриз охлаждается менее эффективно, чем вода.


Прочность охлаждающей жидкости можно проверить с помощью недорогого
тестер антифриза нравится.

СОСТОЯНИЕ ОХЛАЖДАЮЩЕЙ ЖИДКОСТИ

Следующее, что нужно проверить — это состояние охлаждающей жидкости.Основной причиной выхода из строя радиатора является коррозия из-за недостаточного обслуживания системы охлаждения. Ингибиторы коррозии в обычных антифризах со временем постепенно истощаются, поэтому рекомендуемый интервал замены охлаждающей жидкости традиционно составлял каждые два года или от 24 000 до 30 000 миль для профилактического обслуживания.

Новые формулы антифриза «расширенного обслуживания», которые могут пройти 5 лет или 150 000 миль между заменами, уменьшают потребность в обслуживании системы охлаждения и могут снизить риск преждевременного выхода из строя радиатора.Но некоторые старые автомобили все еще имеют антифриз с обычными присадками в системе охлаждения. Таким образом, если пренебречь регулярными проверками и заменой охлаждающей жидкости, риск коррозии резко возрастает с каждым годом.

Проверка уровня pH охлаждающей жидкости с помощью химически обработанных тест-полосок может помочь вам определить состояние охлаждающей жидкости. Но имейте в виду, что современные охлаждающие жидкости обычно работают с более низким уровнем резервной щелочности (RA), чем раньше. Щелочность типичной смеси антифриз / вода будет варьироваться в зависимости от пакета присадок в антифризе и может варьироваться от 8 до 14.Среднее значение раньше составляло около 10,5, но некоторые охлаждающие жидкости с увеличенным сроком службы теперь поддерживают защиту от коррозии при pH всего 8,3. В зависимости от используемых тест-полосок pH можно получить ложное указание на плохую охлаждающую жидкость. Следовательно, чтобы получить точные показания, вы должны знать, какой тип охлаждающей жидкости находится в системе, и использовать соответствующие тест-полоски для проверки.

РАЗЛИЧНЫЕ ВИДЫ АНТИФРИЗА

Какой тип антифриза следует использовать в системе охлаждения? К сожалению, определить тип охлаждающей жидкости, которая есть в системе охлаждения, не всегда просто.Вы не можете выбрать цвет, потому что он варьируется. Большинство обычных двухлетних антифризов для отечественных автомобилей Северной Америки окрашены в зеленый цвет. Но Saturn и некоторые европейские производители также используют зеленый краситель для охлаждающих жидкостей с увеличенным сроком службы. Охлаждающая жидкость с увеличенным сроком службы DEX-COOL, которая используется в автомобилях GM 1996 года и более новых, окрашена в оранжевый цвет, чтобы отличить ее от обычного антифриза. При смешивании с другими типами антифриза цвет может измениться или не измениться в зависимости от дозировки. GM предупреждает, что смешивание охлаждающей жидкости с увеличенным сроком службы с обычной охлаждающей жидкостью сокращает срок службы охлаждающей жидкости до срока службы обычной охлаждающей жидкости.

Чтобы добавить цветовую путаницу, некоторые европейские и корейские охлаждающие жидкости окрашены в синий цвет. В своей охлаждающей жидкости Mercedes использует желтый краситель. Новая охлаждающая жидкость Toyota с увеличенным сроком службы окрашена в красный цвет. Смешайте что-либо из вышеперечисленного, и трудно сказать, какого цвета может стать охлаждающая жидкость.

Настоящая разница между всеми этими охлаждающими жидкостями EG заключается в пакете присадок. Традиционные формулы антифризов для бытовых применений с алюминиевыми радиаторами, головками цилиндров или блоками цилиндров содержат высокий уровень силикатов (что на самом деле не требуется в чугунных двигателях с медными / латунными радиаторами).Силикаты образуют на алюминии защитный барьер, который защищает металл. Однако присадки в антифризы европейских и азиатских производителей, как правило, содержат меньше силикатов и в основном используют бораты для подавления коррозии. Европейские антифризы также не содержат фосфатов, поскольку жесткая вода может реагировать с фосфатами с образованием отложений кальция и магния. Азиаты используют фосфаты, но без боратов и с низким содержанием силикатов или без них, потому что они говорят, что бораты могут фактически усугубить проблему коррозии алюминия, если пренебречь охлаждающей жидкостью.

В охлаждающих жидкостях с увеличенным сроком службы используется совершенно другой химический состав, ингибирующий коррозию, в котором вместо силикатов, фосфатов или боратов используются карбоксилатные органические кислоты.


Схемы применения антифриза:


Щелкните изображение выше, чтобы просмотреть диаграмму в полном размере.


Щелкните изображение выше, чтобы просмотреть диаграмму в полном размере.

Охлаждающие жидкости Euro Spec

Охлаждающие жидкости, соответствующие спецификациям европейских производителей автомобилей, включают G-05, G-30, G-40 и G-48.Вот описание каждого рейтинга:

G-05: Гибридный антифриз HOAT, соответствующий спецификациям Ford и Chrysler (с 2002 по 2012 год), а также Mercedes (2011 года и старше), Audi, VW и Porsche (2004 года и старше).

G-30: Формула ОАТ без силикатов и фосфатов. Соответствует требованиям BS 6580: 2010 для европейских приложений.

G-40: Формула OAT с силикатами для Mercedes, VW и других европейских производителей. Соответствует ASTM D3306, D4985 и D6210, VW TL 774G и TL774J, а также грузовикам Mercedes и Mercedes 325.5 спецификаций.

G-48: Формула OAT с низким содержанием силикатов, низким pH и без фосфатов для Volvo, BMW и MINI. Соответствует ASTM D3306 и D4985, GM 1988M и 1825M, Tesla (2013 г. и новее), Mercedes 325.0 до 2017 г. и BMW GS9400.


Универсальные охлаждающие жидкости

Чтобы избежать путаницы, несколько поставщиков антифризов (Peak и Prestone) продают антифриз «универсального» типа, который совместим со всеми типами антифризов, а также всеми марками и моделями автомобилей.Некоторые утверждают, что это не совсем так, но, похоже, все работает нормально и широко используется на вторичном рынке.

ДОБАВИТЬ ИЛИ ИЗМЕНИТЬ АНТИФРИЗ

При добавлении или замене охлаждающей жидкости лучше всего следовать рекомендациям производителя транспортного средства. Замените то же самое на такое же, если возможно, используя тот же тип антифриза, который был в системе. Эту информацию можно найти в вашем руководстве по эксплуатации, или она может быть нанесена на крышку радиатора или резервуар охлаждающей жидкости.Или используйте универсальную охлаждающую жидкость , одобренную для ВСЕХ применений.

Какая охлаждающая жидкость лучше всего подходит для старых автомобилей? Если вашему автомобилю или грузовику более 15 лет, и он оснащен биметаллическим двигателем (железный блок и алюминиевые головки) и алюминиевым радиатором, наилучшую защиту, вероятно, будет обеспечивать охлаждающая жидкость HOAT или G-05, содержащая силикаты. Силикаты помогают защитить алюминий от коррозии. С другой стороны, если у вас старый автомобиль с железным или биметаллическим двигателем и радиатором из меди / латуни , используйте охлаждающую жидкость с традиционной ЗЕЛЕНОЙ формулой.Добавки в традиционные охлаждающие жидкости обеспечивают лучшую защиту медных / латунных радиаторов.

Добавить комментарий

Ваш адрес email не будет опубликован. Обязательные поля помечены *